X



トップページ数学
1002コメント363KB
大学学部レベル質問スレ 15単位目
レス数が1000を超えています。これ以上書き込みはできません。
0001132人目の素数さん
垢版 |
2020/12/28(月) 18:05:49.69ID:YO0EdTIr
大学で習う数学に関する質問を扱うスレ

・質問する前に教科書や参考書を読むなりググるなりして
・ただの計算は
http://wolframalpha.com
・数式の表記法は
http://mathmathmath.dote ra.net
・質問のマルチポストは非推奨
・煽り、荒らしはスルー

関連スレ
分からない問題はここに書いてね478
http://rio2016.5ch.net/test/read.cgi/math/1511604229/

※前スレ
大学学部レベル質問スレ 14単位目
https://rio2016.5ch.net/test/read.cgi/math/1594758474/
0002132人目の素数さん
垢版 |
2020/12/28(月) 18:06:22.71ID:YO0EdTIr
建てました
0003132人目の素数さん
垢版 |
2020/12/28(月) 18:41:21.10ID:YO0EdTIr
保守
0004132人目の素数さん
垢版 |
2020/12/29(火) 07:38:32.84ID:EQyXg2tf
保守
0007ID:1lEWVa2s
垢版 |
2020/12/29(火) 13:05:41.74ID:gUYhfs9N
>>5
逆から計算すると底辺かける高さで表せるとしたら三角錐の底辺は1/2or(又は)0だから。
どちらかしか成り立たないのはおかしい。
以上証明終わり。
具体的値は未解決問題(自分の中では)。
0010132人目の素数さん
垢版 |
2020/12/29(火) 14:54:21.60ID:ugO5JFyV
「全順序部分集合」を「鎖」と言うようですが、これの読み方は「くさり」ですか?それとも「さ」ですか?
どっちでもいいですか?
0014132人目の素数さん
垢版 |
2020/12/31(木) 13:56:09.88ID:iSEgwVhM
(x+y)^4=x^2y の囲む面積を求める問題で、本にある答えは(極座標変換して)θが0からπ/2までで積分しています
rの符号を見てθの積分範囲は0からπになりそうな気がするんですが、π/2からπの範囲は足さなくていいんでしょうか?それとも常にx≧0ですか?
0015132人目の素数さん
垢版 |
2020/12/31(木) 14:38:14.50ID:2YT2SI7j
>>14
x/y=tとおいて
与式
⇔x(1+t^4)=t^3
⇔x=t^3/(1+t^4), y=t^2/(1+t^4)
でy軸対称みたいだから後で2倍してるんじゃない?
0016132人目の素数さん
垢版 |
2020/12/31(木) 14:43:27.51ID:2YT2SI7j
極座標でもわかるな
r = sinθcos^2θ/(sinθ+cosθ)^2 (-π/2<θ<π/2)
において変換(r→-r, θ→-θ)で不変だからy軸対称
0017132人目の素数さん
垢版 |
2020/12/31(木) 15:43:45.40ID:iSEgwVhM
>>15
この(iii)です、2倍されてなさそうですが後で2倍し忘れただけなのかな
ありがとうございます
https://i.imgur.com/4v0yZvs.jpg
0019132人目の素数さん
垢版 |
2020/12/31(木) 16:09:09.25ID:2YT2SI7j
>>17
wolfram大先生に教えてもらった
θ=3π/2で切れるから第二象限では切れてる
第三、第四象限は変換
θ→π+θ、r→-rでの不変だからこない
0021132人目の素数さん
垢版 |
2020/12/31(木) 16:12:38.65ID:iSEgwVhM
>>19-20
ありがとうございます
実際にπ/2からπまでの積分も加えようとしたら0になってしまいました……
やっぱりグラフ書いてみないとダメですね
0023132人目の素数さん
垢版 |
2021/01/03(日) 16:27:58.70ID:78obLKO4
すみません、この5.の説明をして欲しいです
https://i.imgur.com/ACbmQdP.jpg
https://i.imgur.com/AJAhe7g.jpg

問題は「A!∈M(!は否定) 、したがって∃λ,O_λ⊂A」としている部分で
EとしてR^2の原点中心の閉球を考えて、開被覆を原点を少しだけ越えたところの半平面(とEとの共通部分)、A=E-{(0,1)}とすれば反例になってしまうかと思います
0025132人目の素数さん
垢版 |
2021/01/03(日) 18:37:46.64ID:KwDFZq1H
>>23
問題の誤字で
\mathfrak{M} = { M | ∀λ(λ∈Λ) [M ⊂ O_λ] }
ではなくて
\mathfrak{M} = { M | ∀λ(λ∈Λ) [M \not⊂ O_λ]
なのでは?
0026132人目の素数さん
垢版 |
2021/01/03(日) 20:01:14.26ID:78obLKO4
>>25
あーなるほど……
ずっと解答が誤字ってるのかと思ってて、ただそうすると今度はA以降がおかしくなるのでなんか変だと思いましたが、そうか問題が誤植か……ありがとうございます
0028132人目の素数さん
垢版 |
2021/01/03(日) 22:21:17.74ID:s/Gx7GCJ
>>23
書名プリーズ
0030132人目の素数さん
垢版 |
2021/01/04(月) 08:11:50.28ID:yvHLV+bw
Rings of Differential Operators
はMSCのどこに分類されるべきだと思いますか?
0034132人目の素数さん
垢版 |
2021/01/05(火) 18:58:26.60ID:1s28U22F
>>33
なるほど...さわりだけでも構わないのですが具体的にどんな感じになりますか?
不勉強でごめんなさい
0036132人目の素数さん
垢版 |
2021/01/06(水) 18:09:52.85ID:ViuKwMX7
M:コンパクトなn-dim可微分多様体でN=S^(n-4i)を球面として次元は4i<(n-1)/2を満たすとする
このとき可微分写像f:M→Nに対して
y∈Nが正則値ならf^-1(y)は4i次元の可微分多様体になりますが
その法束が自明である事はどのようにして言えるのでしょうか

具体的な本の中で使われていた事実なので,いろいろ仮定がついていますが
どれが必要なのかはわからないので全て書きました
0037132人目の素数さん
垢版 |
2021/01/06(水) 20:44:28.42ID:zmxgu6cw
本の記述がわからないという質問をする人は書名とページを明記してほしい
0038132人目の素数さん
垢版 |
2021/01/06(水) 22:41:46.38ID:ViuKwMX7
すみません
ミルナー・スタシェフの特性類講義p.224のLem20.1の証明の最後のステップになります
0040132人目の素数さん
垢版 |
2021/01/06(水) 23:25:32.70ID:ViuKwMX7
向き付け不可能な空間をユークリッド空間に埋め込むと
法束が向き付け可能であればユークリッド空間の向きと法束の向きから
元の空間の向きも定まってしまって矛盾するので法束は向き付け不可能
よって法束は自明ではないことが言えるようです
なので何らかの仮定は必要なはずです
0041132人目の素数さん
垢版 |
2021/01/06(水) 23:51:20.82ID:ylqBDucg
>>36
そのノーマルバンドルは1点の周りのチャートの中のR^(n-4i)引き戻したものになるから自明じゃないの?
>>40
>向き付け不可能な空間をユークリッド空間に埋め込むと
これと元の話とはどう関係してるの?
0042132人目の素数さん
垢版 |
2021/01/06(水) 23:56:43.00ID:/IhTtVzQ
>>38
本を見たけど>>41さんの言う通りのことが書いてあった。
ベクトル束の引き戻しの理解があやふやなんだと思う。
0043132人目の素数さん
垢版 |
2021/01/07(木) 00:14:59.69ID:8tQ6h1/P
>>41
引き戻しになるという事がなぜ言えるのか分からないのですが
引き戻しについてもう一度勉強し直すしかないでしょうか

>>40>>39へのアンカーが抜けていました
0046132人目の素数さん
垢版 |
2021/01/08(金) 20:34:31.50ID:o8jNRgcg
>>45
え?
引き戻しになるの?
f:M→NでNの接束はm-4i次元のバンドル
その引き戻しf*T(N)はM上のランクn-4iのバンドルで接束はcovariantだから自然な射T(M)→f*T(N)があってランクがそれぞれn,n-4iでこの射のkernelがf^-1(y)の接束だと思うんだけど
0047132人目の素数さん
垢版 |
2021/01/08(金) 20:52:48.61ID:b2lOJmvC
>>46
次元についてはM:n次元,N:n-4i次元でf^-1(y):4i次元なので
Mでのf^-1(y)の法束はn-4i次元となって
Nでのyの法束つまりyでの接束の次元n-4iと一致しています

証明はfの微分
Df_x:DM_x→DN_yの核にDf^-1(y)_xが含まれるので
F:DM_x/Df^-1(y)_x→DN_yが誘導されて
これはそれぞれの法束のファイバーの間の線形写像
正則性より元の微分Df_xが全射なので誘導された写像も全射であって
domainと行き先の次元が等しいのでFは結局同型
よってこれが法束の間の束写像を定めているので〜と考えました
0048132人目の素数さん
垢版 |
2021/01/08(金) 21:38:35.09ID:o8jNRgcg
>>47
そう、君の書いてるDN_yを張り合わせたものがNの接束T(N)の引き戻し
つまり引き戻しf^*(T(N))はT(M)の商バンドルであって部分バンドルではない
でバンドルの完全列
0→T(f^(-1)(y))→T(M)→f^*(T(N))→0ができる
問題は第3項が自明なら第1項が自明か?
でそんなに自明でないし一般には成り立たないと思う
他の次元差がどうこう使わないと出ない希ガス
0049132人目の素数さん
垢版 |
2021/01/08(金) 22:13:49.02ID:o8jNRgcg
イヤ、わかった
法バンドルが余接バンドルじゃなくてリーマン計量がなんか入っててその意味での直交補空間バンドルって意味なら通る
そう言う意味?
0050132人目の素数さん
垢版 |
2021/01/09(土) 00:19:25.58ID:KuBWF/Pz
>>49
はい、確かに本ではリーマン計量は常に仮定していました

まだ>>48の内容がよく理解できていないのですが
第1項T(f^-1(y))の自明性はなぜ必要なのでしょう?
欲しいのはf^-1(y)の法束の自明性なですが
これは第3項の自明性のことではないのでしょうか
0051132人目の素数さん
垢版 |
2021/01/09(土) 06:30:41.68ID:0ZtIVcdq
>>49
なんでデュアルと思うかなあ
法は直交つまり軽量入ってる場合の用語
だいたい
デュアルが自明かどうかは
君書いてるようにT(f^-1(y))が自明かどうかなんだから
わざわざ(君の言う)法バンドルが自明とか問うわけないじゃん
0052132人目の素数さん
垢版 |
2021/01/09(土) 07:55:29.87ID:m8iZn/gm
>>50-51
ホント失礼しました
畑が代数系であんまりリーマン計量とかやらないので反射的にcotagentと脳内変換してた
お騒がせしました
0053132人目の素数さん
垢版 |
2021/01/11(月) 15:23:23.78ID:x7bI62AW
Σ[n=1,...,∞]1/(n^k) = 1/a_n * π^k
と表せる?(kは自然数≧2)
kが偶数の時は簡潔な自然数列が定まる?
0055132人目の素数さん
垢版 |
2021/01/12(火) 00:38:12.24ID:sViBVPi/
「(可微分)多様体Mの弧状連結性により座標近傍Uも弧状連結であるから、」
という記述があるんですが本当ですか?M=R^nのときですら言えないような気が……

ちなみに、上の記述はdf=0ならfがM上で定数であることの証明中に出てきます
直観的には、U(と同相なR^nの開集合)の連結成分上では定数だから、後は別の弧状連結な座標近傍をくっつけて局所定数fの定義域を広げていく(Mの弧状連結性からM全体に広げられる、したがってM全体で定数)やり方で示せると思いますが、この方針だとどこかで詰まりますか?
0056132人目の素数さん
垢版 |
2021/01/12(火) 01:35:55.22ID:tpqD8OxO
>>55
最初の文はそのまま読めば当然ウソだけど
筆者は弧状連結な座標近傍が取れるという意味で書いてると思う

2つ目の疑問はその方針では言えていない
例えばR上の座標近傍で(0,1/2),(1/4,3/4),(3/8,7/8),…というものを使って拡張していっても
(0,1)についての結果しか言えずR全体の結果には到達できない、つまり
>Mの弧状連結性からM全体に広げられる
という部分を示すために具体的に2点の間の道を取って
道に沿って拡張して一方から他方へとたどり着けることを示している
0057132人目の素数さん
垢版 |
2021/01/12(火) 04:04:02.21ID:GHL5aS+l
そういうの暗算みたいにできない?
Mがスムースならわざわざ弧状とか付けなくてよくない?
一体どこ大学だよ?どんな本で勉強してんの?
0058132人目の素数さん
垢版 |
2021/01/12(火) 07:52:35.26ID:T14+yp3e
>>55
都度広げていくのではなくて
全部の点の周りにそれぞれU取って
どこでも定数それがどこでも同じ値っていう方針じゃない?
0059132人目の素数さん
垢版 |
2021/01/12(火) 07:55:49.67ID:T14+yp3e
>>56
>>55はそのつもりで書いていたのかも
0060132人目の素数さん
垢版 |
2021/01/12(火) 09:27:52.56ID:m4PRHULg
そもそも
a∈im(f)に対してf^(-1)(a)は空でない開集合かつ閉集合を示す方が楽やろ
0061132人目の素数さん
垢版 |
2021/01/12(火) 17:02:12.95ID:iZi7xbpm
>>55
前も書いたが、本の記述がわからないという質問をする人は書名とページを明記してほしい
0062132人目の素数さん
垢版 |
2021/01/12(火) 20:17:04.22ID:sViBVPi/
>>56
ですよね、ありがとうございます
とりあえず弧状連結成分取って読み進めましたが、回りくどすぎた

>>57
学生ですらないです……
単なる趣味で数理物理の本を読んでます
0063132人目の素数さん
垢版 |
2021/01/12(火) 23:10:17.22ID:8FebxfZV
位相空間に関する性質が、積空間や部分空間に遺伝する性質であるか遺伝しない性質であるかって、何が要因で決まる?
0064132人目の素数さん
垢版 |
2021/01/12(火) 23:37:40.04ID:Q+7xixvm
一概には言えないだろうね
開や閉は部分空間で変化する(部分空間との共通部分をとる)から開や閉が関係してる条件は怪しくなってくる
でも条件が開や閉の共通部分で書けているものは大丈夫だったり
0066132人目の素数さん
垢版 |
2021/01/13(水) 23:37:01.25ID:cQ60IoU5
>>63
要因ってあるって思ってるの?
0069132人目の素数さん
垢版 |
2021/01/14(木) 15:29:57.06ID:IIffE6HS
順序数って集合論や基礎論の議論以外に登場することありますか?
知人と話してるとき、なんの役にたつの?と聞かれて困ってしまいました。
私としては順序数や集合論自体、大変おもしろく学べているのでそれで満足なんですが。
何か良い例がありましたらよろしくお願いします。
0071132人目の素数さん
垢版 |
2021/01/14(木) 15:56:34.49ID:8EXbbp8K
距離空間がパラコンパクト、の証明は順序数を使うと瞬殺
0073132人目の素数さん
垢版 |
2021/01/14(木) 18:13:11.45ID:1DsV0Lpv
>>69
任意のR-加群が入射加群に埋め込まれることを示す一般的アプローチのBaer's argumentとか
0074132人目の素数さん
垢版 |
2021/01/14(木) 18:15:20.91ID:1DsV0Lpv
上の一般的は多くの人にとって普通ということではなく数学的により広いということです
0075132人目の素数さん
垢版 |
2021/01/14(木) 19:24:39.34ID:SpshK7i6
複素解析の質問です。よろしくお願いします。
f(z)がz=∞を孤立特異点に持つとは、f(1/ζ)がζ=0を孤立特異点に持つことと定義します。
f(z)=1/(z-1)とします。
f(1/ζ)=1(1/ζ-1)=ζ+ζ^2+ζ^3+…より、ローラン展開の主要部が0であるためにz=∞を孤立特異点に持ちません。
f(z)のz=∞での留数は、z=∞が孤立特異点であるときに限り定義されるはずなのですが、無限遠点含めて留数の和を取ると0になるため1/(z-1)のz=∞での留数は-1と分かります。
実際に調べてみるとやはり1/(z-1)のz=∞での留数は-1で間違いないらしいです。
z=∞が孤立特異点でないのに留数が存在しているのですが、これはおかしくないのでしょうか?
0077132人目の素数さん
垢版 |
2021/01/14(木) 23:18:51.47ID:h0uGq2m0
1/(z-1) を展開した事ないのか?
級数展開の証明では 1/(1-z) の展開を使ってるのにな
0078132人目の素数さん
垢版 |
2021/01/14(木) 23:40:46.75ID:UO8qfSKq
自己解決しました。
f(z)=把_n z^-nとローラン展開した時にRes[z=∞]f(z)=-c_1となるのでこのローラン展開主要部が0であったとしても(つまりz=∞が孤立特異点でなくても)留数は存在して0以外の値を取りうるのですね……。
無限遠点に限っては極でなくても留数が定まるようです、お目汚し失礼しました。
0079132人目の素数さん
垢版 |
2021/01/15(金) 01:42:18.63ID:K2CvppaW
何を馬鹿なことを言ってんだ
1/(z -1) = -1 - z - z^2 - z^3 - …
1/(1/ζ -1) = -1 - 1/ζ - 1/ζ^2 - 1/ζ^3 - …
というだけだろ
0080132人目の素数さん
垢版 |
2021/01/15(金) 02:19:01.95ID:dHJctuuT
コレはresという記号がちょっと誤解を与える記号である事が要因
wikipediaにも書いてあるけどresはあくまでスカラーに対して定義されるものでなく1-formに対して定義されるもの
本来はres(1/(z-1)dz, z=∞)のように表すべきものなのを雑に表してるから間違いやすい
そこは最初にこの記号作った人がそういうふうに決めてしまったので脳内で変換して読まないといけない
平面上の話だけしてるならそれでもいいが、z=∞とか出てくると話が狂う
res(1/(z-1)dz, z=∞)をwで表示して計算するなら
res(1/(1/w-1) (-dw/w^2), w=0)
= res(-1/(w-w^2)dw, w=0)
= res(1/(w-1)-1/w,w=0)
=-1
コレでres(1/(z-1)dz,z=1)=1と話の辻褄が合う
0081132人目の素数さん
垢版 |
2021/01/15(金) 16:44:58.36ID:OPBgJ08z
>>80
なるへそ
素晴らしい
0082132人目の素数さん
垢版 |
2021/01/16(土) 09:45:30.27ID:w1RIfh/2
・a[i,j] は直交行列。(Lorentz変換)
γ1, γ2, γ3, γ4 は 4次のエルミート行列で
・γi γj + γj γi = 0 (i≠jのとき)   [反交換関係]
・γi γi = I (単位行列)
を満たすものとします。(Dirac行列)
この時、
 ΣΣΣΣ{i,j,k,m} a[1,i]a[2,j]a[3,k]a[4,m] γi γj γk γm = det(a) γ1 γ2 γ3 γ4
となる事を示してください。
物理の教科書的には 具体的な Dirac行列 と 微小Lorentz変換を与えて
計算するのが定番みたいなんですが、前提として挙げた代数関係だけを使って解けませんかね?
0083132人目の素数さん
垢版 |
2021/01/16(土) 13:14:09.57ID:FUEXUGEa
twitterでlim x→0 logxとlim x→+0 logxが同じかどうかという問題があったのですが、
lim x→a f(x)=-∞の定義はfの定義域をDとして∀N>0∃δ>0∀x∈D 0<∣x-a∣<δ⇒f(x)<-Nで、
lim x→0 logx=-∞を考えると、δ=e^-Nとおくと
0<x<e^-N(x∈logの定義域=(0,∞)なので絶対値が外せる)⇒logx<-Nとなって成立し、
lim x→0 logxとlim x→+0 logxは同じになると思うのですが、おかしい部分などありますか?
0084132人目の素数さん
垢版 |
2021/01/16(土) 13:41:26.11ID:vDsxXk9V
>>82
反交換関係から γi γj γk γm = sign(i, j, k, m) γ1 γ2 γ3 γ4 を出して
あとは det の定義
0085132人目の素数さん
垢版 |
2021/01/16(土) 14:07:31.58ID:w1RIfh/2
>>84 和をとる時の添え字は各自独立に動くので
γ1 γ2 γ1 γ4 みたいな重複アリの項をどう処理したらいいのか分からんのです。

いくつかランダムな直交行列と、具体的な Dirac行列(Dirac表現) で数値計算してみたんですが、
・γi γi γj γk, γi γj γi γk, .... (3色: i,j,kは相異なる)
・γi γj γj γj, γi γj γi γi, ...., γi γi γj γj, ... (2色: i,jは相異なる)
・γi γi γi γi (1色)
この場合分けの総和でゼロになる事が(数値上で)確認できたんですが、其々の段の和はゼロにはならんのです。
どうやれば代数的に相殺できるのやら...といった感じなのです。
0086132人目の素数さん
垢版 |
2021/01/16(土) 14:22:37.39ID:0L2ZTQuB
>>83
そもそも実関数ならx<0で定義されてない、つまりx→0の極限操作そのものが定義されてないです
ただし本によってはx<0で定義されない場合は片側極限(x→+0)によってその極限(x→0)を定義するので、回答は「立場によって変わる」となります

「lim x→a f(x)=-∞の定義は…」とありますが、そこでのDはaの近傍、特にx<aであるようなある点xも含むことを仮定してませんか?もちろん、上で言ったように片側極限をもって定義することもありaが孤立点でなければいいと書いてるものもありますが、とにかく定義の確認をするべきです
0087132人目の素数さん
垢版 |
2021/01/16(土) 14:39:51.11ID:FUEXUGEa
>>86
すみません、定義に見落としがありました
lim x→a f(x)=-∞の定義はfの定義域をDとして、aは集積点で∀N>0∃δ>0∀x∈D 0<∣x-a∣<δ⇒f(x)<-Nとなります
ただ、結局0は(0,∞)のRの部分集合としての集積点であり、やはりlim x→0 logxとlim x→+0 logxは同じになるということで良いのですか?
0090132人目の素数さん
垢版 |
2021/01/16(土) 16:02:44.38ID:w1RIfh/2
>>85 (追加)
別の場合分けで部分的にゼロになるのは分かるんです。
ΣΣΣ{i,k,m} a[1,i]a[2,i]a[3,k]a[4,m] γi γi γk γm
  = Σ{i} a[1,i]a[2,i] ΣΣ{k,m} a[3,k]a[4,m] γk γm = 0 (aの直交性)
ΣΣΣ{i,j,k かつ i≠j} a[1,i]a[2,j]a[3,k]a[4,k] γi γj γk γk
  = ΣΣ{i,j かつ i≠j} a[1,i]a[2,j] γi γj Σ{k} a[3,k]a[4,k] = 0 (aの直交性)
しかしこの先が続かない... 残りの場合分けは簡単になるようには見えません。

>>88
i,j,k,m が全て相異なるパターンの和がそうなるのは分かります。
そうでないパターンの総和がゼロになる事を示したいのです。
本当に代数関係だけで示せるのかは知りません。
0092132人目の素数さん
垢版 |
2021/01/16(土) 16:46:17.21ID:FUEXUGEa
>>89
ありがとうございます、答えがはっきり分かりスッキリしました
0094132人目の素数さん
垢版 |
2021/01/16(土) 17:55:16.38ID:w1RIfh/2
>>93
PARI/GPによる数値計算の一部を載せときます。
この種の計算に向いてる言語とは思いませんが、ある程度は何をしたか伝わるかと思います。

X = matrix(4); \\ 4次ゼロ行列
\\ 3色
X += sum(i=1,4,sum(j=1,4,sum(k=1,4, (i!=j)*(j!=k)*(k!=i)*( \
a[1,i]*a[2,i]*a[3,j]*a[4,k] *G[j]*G[k] + \
a[1,i]*a[2,j]*a[3,i]*a[4,k] *G[k]*G[j] + \
a[1,i]*a[2,j]*a[3,k]*a[4,i] *G[j]*G[k] + \
a[1,j]*a[2,i]*a[3,i]*a[4,k] *G[j]*G[k] + \
a[1,j]*a[2,i]*a[3,k]*a[4,i] *G[k]*G[j] + \
a[1,j]*a[2,k]*a[3,i]*a[4,i] *G[j]*G[k] ) )));

\\ 2色
X += sum(i=1,4,sum(j=1,4, (i!=j)*( \
a[1,i]*a[2,i]*a[3,j]*a[4,j] *+matid(4) + \
a[1,i]*a[2,j]*a[3,i]*a[4,j] *-matid(4) + \
a[1,i]*a[2,j]*a[3,j]*a[4,i] *+matid(4) + \
a[1,j]*a[2,i]*a[3,i]*a[4,i] *G[j]*G[i] + \
a[1,i]*a[2,j]*a[3,i]*a[4,i] *G[i]*G[j] + \
a[1,i]*a[2,i]*a[3,j]*a[4,i] *G[j]*G[i] + \
a[1,i]*a[2,i]*a[3,i]*a[4,j] *G[i]*G[j] ) ));

\\ 1色
X += sum(i=1,4, a[1,i]*a[2,i]*a[3,i]*a[4,i])*matid(4);

これでゼロ行列になりました。
(ランダム直交行列: a[i,j]と Dirac行列: G[i] を用意する部分は省略)
0095132人目の素数さん
垢版 |
2021/01/16(土) 22:41:50.26ID:w1RIfh/2
>>82 (改)
もしかしたら当初の代数関係のみを用いて示すのは無理があるのかもしれません。

Sを4次の変換行列として、
物理的要請 S⁻¹γᵢS = Σ{j} aᵢⱼγⱼ を加えます。
本来示したかったのは S⁻¹γ₅S = det(a) γ₅ の等式でした。(γ₅:= γ₁γ₂γ₃γ₄)

S⁻¹γ₅S = (S⁻¹γ₁S)(S⁻¹γ₂S)(S⁻¹γ₃S)(S⁻¹γ₄S)
= Σ{ijkm} a₁ᵢ a₂ⱼ a₃ₖ a₄ₘ γᵢ γⱼ γₖ γₘ
一方で γ₅ = 1/4! *Σ{ijkm} ε[ijkm] γᵢ γⱼ γₖ γₘ (ε[ijkm]は完全反対称テンソル)
と表せるので、
S⁻¹γ₅S = 1/4! *Σ{ijkm} ε[ijkm] (S⁻¹γᵢS)(S⁻¹γⱼS)(S⁻¹γₖS)(S⁻¹γₘS)
= 1/4! *Σ{ijkm} Σ{stuv} ε[ijkm] aᵢₛ aⱼₜ aₖᵤ aₘᵥ γₛ γₜ γᵤ γᵥ
= det(a) γ₁γ₂γ₃γ₄ {∵ εの反対称性より s,t,u,vの重複項は消える}
これより >>82 の等式が示せました。
0098132人目の素数さん
垢版 |
2021/01/17(日) 14:13:12.87ID:lQ96Dcp/
物理的要請と書いてるけど、そういうSはいつでも取れるはずなんじゃないっけ
直交群の被覆であるスピン群からそういう元を取ったと思えば
0099132人目の素数さん
垢版 |
2021/01/17(日) 22:07:32.73ID:lQ96Dcp/
だからスピン群を経由しない方法でも示せるはずだけど、和を包除原理や対称反対称分解使って計算するだけだと上手く示せない…
すごくモヤモヤする
0100132人目の素数さん
垢版 |
2021/01/17(日) 22:30:36.66ID:GLQn0cgY
>>98
> そういうSはいつでも取れる
空間回転とLorentzブーストに関しては無限小変換の生成子を構成する。空間反転に関しては S=γ₄ が条件を満たす事を確認する。
物理の教科書的にはそういう流れになります。本を読み返したらDirac行列の表現によらない記述になってました。
a の中には物理的に無意味なのもありますが、虚数の空間回転とか加えれば たぶん網羅するのでしょう。
そういう意味では >>82 は「代数関係のみを使って示せる」と言えるのかも。

> 直交群の被覆であるスピン群
実験物理出身の自分には高度過ぎるようです。
0101132人目の素数さん
垢版 |
2021/01/17(日) 23:48:31.71ID:lQ96Dcp/
>>100
ローレンツ計量の場合の証明はちゃんと読んだことないけど、O(3,1)の場合もPin(3,1)からの全射があったはず
O(4)のの場合、回転は必ずいくつかの鏡映の積で書けるから、その鏡映を表現するPin(4)の元の積をSとすればそれでok
(ただ詳しくみると正確にはS^(-1)vSはvの鏡映の-1倍になってしまうので、問題の式もS^(-1)γiS=det(a)Σaijγjということになる
しかし定数倍det(a)は>>95の計算に影響ないので同じ結果を得る)
0102132人目の素数さん
垢版 |
2021/01/18(月) 00:06:47.25ID:BNPNigpN
開区間の重積分って閉空間と同じように計算していいの?
0103132人目の素数さん
垢版 |
2021/01/18(月) 01:04:26.10ID:TCD9nCM7
そもそも一般のaijでは言えないの当たり前じゃないの?
a1=a2=a3=a4=(1,0,0,0)のとき左辺はγ1γ1γ1γ1=Iだけど右辺0やん
0105132人目の素数さん
垢版 |
2021/01/18(月) 02:14:44.17ID:TSsLg4J/
あんま美しくないけど気合いで示せたわ

どこかが重複する和のタイプは包除原理より
(2,1,1)-(2,2)-2(3,1)+6(4)

これを具体的に書くと
((abxx)+(xxcd)+(axxd)+(xbcx)+(xbxd)+(axcx))
-((xxyy)+(xyyx)+(xyxy))
-2((axxx)+(xbxx)+(xxcx)+(xxxd))
+6(xxxx)

交換関係を使って得られる関係式
(xbcx)= -(xbxc)+2(xbxx)=(xxbc)-2(xxxc)+2(xbxx)
(xbxd)= -(xxbd)+2(xxxd)
(axcx)= -(axxc)+2(axxx)
(xyxy)= -(xxyy)+2(xxxx)
(xxcx)= -(xxxc)+2(xxxx)
を上に代入すると

((abxx)+(xxcd)+(axxd)+(xxbc)-(xxbd)-(axxc))
-((xxyy)+(xyyx)-(xxyy))
これはペアで和を取っている部分があるものばかりなので直交性によりゼロ

どういう仕組みでこうなってるのか解明しないと一般次元で示せないけど…
0106132人目の素数さん
垢版 |
2021/01/18(月) 02:19:04.09ID:TSsLg4J/
多分、形的に一般の次元ではWickの定理のように
1ペア縮約、2ペア縮約、3ペア縮約…の形が1項ずつ出てきて
縮約の形の交差や次数で符号がつくと思われる
0107132人目の素数さん
垢版 |
2021/01/18(月) 02:44:24.76ID:TSsLg4J/
>>105
記号の説明

例えば
(axcx)= -(axxc)+2(axxx)
という式は
Σ[i,k,x(=j,l)]a(1,i)a(2,x)a(3,k)a(4,x)γiγxγkγx
= Σ[i,k,x(=j,l)]a(1,i)a(2,x)a(3,k)a(4,x)γiγx(-γxγk+2δkx)
= -Σ[i,k,x(=j,l)]a(1,i)a(2,x)a(4,x)a(3,k)γi(γxγx)γk
+ 2Σ[i,x(=j,k,l)]a(1,i)a(2,x)a(3,x)a(4,x)γiγxγxγx
に対応している
0108132人目の素数さん
垢版 |
2021/01/18(月) 05:53:21.44ID:TSsLg4J/
つまりだ、直交行列は忘れてγ行列の恒等式

γaγbγcγd
=ε(abcd)γ1γ2γ3γ4+δ(ab)γcγd+δ(cd)γaγb
+δ(bc)γaγd+δ(ad)γbγc-δ(ac)γbγd-δ(bd)γaγc
-δ(ab)δ(cd)-δ(ad)δ(bc)+δ(ac)δ(bd)

を示せばいいわけだ…
0109132人目の素数さん
垢版 |
2021/01/18(月) 06:40:18.49ID:ya0zRNfP
>>102
>区間の重積分
0110132人目の素数さん
垢版 |
2021/01/18(月) 07:06:32.66ID:TSsLg4J/
4^4=256パターンの確かめは大変そうだけど重複のタイプ別に調べれば意外と簡単か

九後をカンニングしたら帰納的にも示せるっぽい
交換関係から
γaγb=1/2!(γaγb-γbγa)+δ(ab)
さらに反対称積を
γ(a(1),a(2),…,a(n))=1/n!Σsgn(σ)γa(σ1)γa(σ2)…γa(σn)
と定義すると一般に帰納的な関係式

γbγ(a(1),a(2),…,a(n))=γ(b,a(1),a(2),…,a(n))
+Σ[i=1,n](-1)^(i-1)δ(b,a(i))γ(a(1),a(2),…a(i-1),a(i+1),…,a(n))

が言えて、これらを使って順次計算できる
γcγd=γ(c,d)+δ(cd)

γbγcγd=γbγ(c,d)+γbδ(cd)
=γ(b,c,d)+δ(bc)γd-δ(bd)γc+γbδ(cd)

γaγbγcγd=γaγ(b,c,d)+γaδ(bc)γd-γaδ(bd)γc+γaγbδ(cd)
=γ(a,b,c,d)+δ(ab)γ(c,d)-δ(ac)γ(b,d)+δ(ad)γ(b,c)
+γaδ(bc)γd-γaδ(bd)γc+γaγbδ(cd)
=γ(a,b,c,d)+δ(ab)γcγd-δ(ac)γbγd+δ(ad)γbγc
+δ(bc)γaγd-δ(bd)γaγc+δ(cd)γaγb
-δ(ab)δ(cd)-δ(ad)δ(bc)+δ(ac)δ(bd)

最後にγ(a,b,c,d)=ε(abcd)γ1γ2γ3γ4に注意すれば>>108を得る
0111132人目の素数さん
垢版 |
2021/01/18(月) 11:57:23.08ID:TCD9nCM7
わかった
a11 = a22 = cosθ、a21 = - a12 = sinθ、a33 = a44 = 0、
aij = 0 ( otherwise )
のとき
δi = Σj aij γj
で定めるときδiもγiと同じ交換関係を満たす
universarityからこの場合にはSがとれる
a12 = a21= = a33 = a44 = 1、
aij = 0 ( otherwise )
のときも同様
結局aijが直交行列の時は上の2タイプの積でかけるのだからいつでもSがとれる
以下>>95
0112132人目の素数さん
垢版 |
2021/01/18(月) 12:13:41.06ID:TCD9nCM7
そうか、さらにわかった
QVをベクトル空間の2次形式Qのなす圏、Algを代数のなす圏とするときクリフォード代数を対応させる対応は自然変換でQVの射A:(V,Q)→(W,R)は必ずクリフォード代数の射S:C(V,Q)→C(W,R)にliftするんだ
しかもuniversalityからAが同型ならSも自動的に同型になる
0114132人目の素数さん
垢版 |
2021/01/18(月) 12:22:50.79ID:Xvy2vIAP
本を前に1時間考え込んで「なるほど…自明だ…」
数学あるある
0115132人目の素数さん
垢版 |
2021/01/18(月) 13:31:20.56ID:EyIEbFkw
x^4+y^4-2x^2の極値を求めたいんですがDが0になってしまって困ってます。y=0とかで固定して考えようと思ったりしたんですがよくわかりません。
0116132人目の素数さん
垢版 |
2021/01/18(月) 13:31:39.36ID:4M+e5aOM
単に無作為抽出って言った場合は復元抽出(同じ標本が何度も選ばれうる)をさすと考えていいですか?
0117132人目の素数さん
垢版 |
2021/01/18(月) 14:19:12.38ID:TSsLg4J/
>>113-114
いや、だからS経由は簡単なんだけど
それを経由させずに示せるかが気になったんよ>>99
S使えば自明なのに使わないと和の打ち消しが非自明なのが面白いと思った
0119132人目の素数さん
垢版 |
2021/01/18(月) 14:19:57.32ID:ya0zRNfP
だめ
0121132人目の素数さん
垢版 |
2021/01/18(月) 18:40:15.24ID:TSsLg4J/
母集団が大きいときはどちらで計算しても統計的な量はほとんど変わらないから計算が圧倒的に楽な復元抽出を暗黙に仮定することが多いようだね
0122116
垢版 |
2021/01/19(火) 00:21:27.20ID:ffU2r4h4
>>121
ありがとうございます
大変納得いたしました
0123132人目の素数さん
垢版 |
2021/01/19(火) 11:48:06.31ID:eAGJZUMW
一次分数変換の分類の話で双曲的/放物的/楕円的という分類が出てきますが
これらの定義をなぜ双曲とか放物とか楕円という名前で呼ぶのかがピンと来ません
双曲線などとどう関係があるのでしょうか
0124132人目の素数さん
垢版 |
2021/01/19(火) 11:53:53.15ID:HvIlgi0N
楕円、放物、双曲の三分類は数学の色んなところに出てきて慣れてきたら円錐曲線との関係なんか気にしなくなる
0126132人目の素数さん
垢版 |
2021/01/19(火) 12:48:46.70ID:6pofGOpJ
力学系と保型関数論で2次正方正則行列が双曲的放物的楕円的に分類されるがだいたい一致する
0128132人目の素数さん
垢版 |
2021/01/19(火) 16:46:31.76ID:WumoAitc
どんな証明にも証明の長さが最小の証明って存在するわけだが、それが一体どんなものかって気になるよなw
0132132人目の素数さん
垢版 |
2021/01/19(火) 20:02:22.74ID:eAGJZUMW
>>126
力学系ってのは楕円型微分方程式とかそういうやつですか
双曲型のWiki眺めてみたけど確かに似てますね

>>127
楕円的は回転になってるので確かにぽいですが
放物的と双曲的がどっちも発散してる感じで
出るとこと入るとこが同じか違うかをなぜ放物と双曲と言うのかと悩んでました
0135132人目の素数さん
垢版 |
2021/01/19(火) 22:25:34.76ID:VmWF8uxU
>>131
https://www.ms.u-tokyo.ac.jp/~ito/notes_functional_analysis_20180511.pdf
(1)は84〜85ページを少し変えればよい
B(X)が作用素ノルムに関して完備となるのは71ページ
(2)が(1)から導けないようではお先真っ暗
0136132人目の素数さん
垢版 |
2021/01/19(火) 23:07:01.96ID:eAGJZUMW
>>133
なるほど!
そう考えると確かに納得できる
放物は楕円に近い運動してますね
ありがとう
0137132人目の素数さん
垢版 |
2021/01/20(水) 00:06:24.17ID:Ei5u5h/+
二次以下の実数多項式全体のなすベクトル空間をP(2;R)における線形変換T(f)= ∫[-1.1]2f(t)×(x-t)^2dtとして基底[1.x.x^2]に関する表現行列を求める問題なんだけどやり方わかる人いる?
定積分をそれぞれa,b,cとおいてもうまくいかない😭
0138132人目の素数さん
垢版 |
2021/01/20(水) 00:28:08.27ID:Ei5u5h/+
∫[-1.1]2f(t)×(x-t)^2dtじゃなくて∫[-1.1]f(t)×(x-t)^2dt
0140132人目の素数さん
垢版 |
2021/01/20(水) 00:39:06.37ID:Ei5u5h/+
どうやって解けばこの解に辿り着くの?
0141132人目の素数さん
垢版 |
2021/01/20(水) 02:28:59.04ID:Ei5u5h/+
できたわ
>>139ありがとう😘
0142132人目の素数さん
垢版 |
2021/01/20(水) 02:40:15.81ID:H7yZoyvA
>>135
すげー分かりやすかった! ありがとうございました!
(2)は等比数列の和の公式使えばいい感じですよね? やってみます
0144132人目の素数さん
垢版 |
2021/01/21(木) 16:50:54.19ID:Wq5GA9tn
(i)はできそうなので大丈夫です
(ii)のコンパクトの方だけお願いいたします
0145132人目の素数さん
垢版 |
2021/01/21(木) 17:55:10.79ID:oVrgxGnu
>>144
>>135の193ページ
0146132人目の素数さん
垢版 |
2021/01/21(木) 21:30:02.61ID:pF58XMAc
複素数の間の共形的かつ全単射で滑らかな写像φ:C→Cは1次変換か1次変換の複素共役である
という定理の証明がわかりません

本ではその少し前に
ψ:D→C,(D⊂C:開集合)が共形的でヤコビ行列がいたるところ可逆ならψかその複素共役が
正則関数になるという結果があり、これとリウビルの定理なりピカールの定理なりを使えば示せると書いてありました
上の命題を使うとφかその共役がCの間の全単射な正則関数であることは言えますが、そこから先がわかりません
(本は深谷「双曲幾何」のp.65です)
0148132人目の素数さん
垢版 |
2021/01/21(木) 22:42:12.51ID:rn3WyCvo
>>146
コレが示せたらいいのでは

fがRiemann球の自己全単射で正則なら一次変換

∵ 一次変換は三重可積遷だからf(0)=0, f(1)=1, f(∞)=∞として良い
f(z)=z^n g(z)、g(z)は正則、g(0)≠0、とおける
f(z)は一対一だったから原点以外に零点を持たない
よってg(z)は零点を持たない
∴ g(z)は定数(∵リュービルの定理)
∴ f(z)=cz^n
さらに再び一対一性よりc≠0, n=1□
0151132人目の素数さん
垢版 |
2021/01/22(金) 01:09:42.81ID:c7pUyOSZ
統計学の独学でもわかりやすい参考書とかってありますか?
ちなみに国立文系で数三はノータッチです。
0153132人目の素数さん
垢版 |
2021/01/25(月) 00:32:13.87ID:MdB8SPaM
ちょっと躓いた

A,B整列集合、 A⊆B、A≠Bとする
m=min(B-A)とおく。
この時、A=B<m> (切片の意味) 
だが、A⊆B<m>の証明に躓いた
0154132人目の素数さん
垢版 |
2021/01/25(月) 00:46:31.94ID:ZfoS9OEC
以下Bを全集合としてBに含まれる事は一々述べない
m≦x,x∈Aとすればm∈Aとなりm∈B-Aに矛盾
∴x∈A→x<m
x<m, ¬x∈Aとすればx∈B-A,x<mとなるがm=min(B-A)に矛盾
∴x<m→x∈A
0155132人目の素数さん
垢版 |
2021/01/25(月) 01:24:13.49ID:cnOGloVb
>>153
mがAに入ってなくて最小なんだからほぼ自明でしょ
証明は背理法で
0158132人目の素数さん
垢版 |
2021/01/25(月) 07:42:42.81ID:cnOGloVb
>>156
>A⊆B<m>は無条件では無理
??
0160132人目の素数さん
垢版 |
2021/01/25(月) 19:41:39.71ID:cnOGloVb
あそっか
0161132人目の素数さん
垢版 |
2021/01/26(火) 11:55:28.00ID:VQBvG5PV
遺伝的有限集合全体をHFとして
関係R⊆HF^nが兩1であるとき、
兩0集合S⊆HF^n+1が
a∈R⇔∃x∈HF((a,x)∈S)
となるようにとれる

キューネンの基礎論p292です
さらっと書いてあるんで自明なんでしょうけど、わかりません

どなたか証明をつけていただけないでしょうか
よろしくお願いします
0163132人目の素数さん
垢版 |
2021/01/26(火) 13:18:26.79ID:tDKpMNKD
>>161
原本をみたら、それらしき箇所に、(see Lemma II.17.28)とあるのだが、訳書の方には書いてないの?
0164132人目の素数さん
垢版 |
2021/01/26(火) 14:11:12.31ID:VQBvG5PV
あ、自分のノートばっかり見てました。
訳書にもありますね。
なんてミスを。すみません。
補題2.17.28から導出するのは論理式の相対化を使うんでしょうか
0165132人目の素数さん
垢版 |
2021/01/26(火) 18:42:57.70ID:I4alVVeD
双曲幾何の上半平面モデル(もしくは円盤モデル)と双曲面モデルとでは角度は等しいのでしょうか
もし異なれば余弦定理などの角度の入っている公式がモデル毎に異なることになると思うのですが
0167132人目の素数さん
垢版 |
2021/01/26(火) 20:19:47.65ID:I4alVVeD
>>166
リーマン幾何やってないので計量が違うという意味がよく分からないのですが
上半平面モデルと円盤モデルは共形ですがこれらの計量は同じなのですか?
0169132人目の素数さん
垢版 |
2021/01/27(水) 01:33:24.46ID:a13HY8y0
>>167
通常、角度というものは、計量から定めるものである。
リーマン幾何やってないのに、角度だのなんだの言っているのがよくわからん。
0170132人目の素数さん
垢版 |
2021/01/27(水) 13:13:01.54ID:HylCjDki
>>168>>169
実質的には計量を考えていることになるのだと思いますが計量という言葉をあまり出さずに書いてる本を読んでいます
(深谷「双曲幾何」)
リーマン幾何ちゃんとやってないと理解が難しいようならそれまで棚上げにしときますが
結論としては余弦定理などの角度が出てくる公式はモデル毎に異なるという事でいいんでしょうか
0172132人目の素数さん
垢版 |
2021/01/27(水) 13:22:34.68ID:HylCjDki
>>171
たとえば余弦定理の公式
cos∠γ=(coshAcoshB-coshC)/(sinhAsinhB)
という式を本では双曲面モデルの上で示していましたが
この式は上半平面モデルや円盤モデルでは成り立たないのかが気になっています
長さが保たれる2つのモデルで保たれることは書いてあったものの
左辺の角度が2つのモデルで同じである保証はあるのかと
0173132人目の素数さん
垢版 |
2021/01/27(水) 13:22:37.39ID:uyFxPKru
>>170
そうじゃない?
でも多分曲率が負の一定値の定曲率空間は計量テンソルの正の定数倍で写り合う気はする
なので上半平面モデルと単位円モデルで差があっても定数倍の差しかない気はする
でもどっちも曲率-1/4とかだったような記憶が...
0174132人目の素数さん
垢版 |
2021/01/27(水) 13:29:53.33ID:HylCjDki
>>172
>長さが保たれる2つのモデルで保たれることは書いてあったものの
長さが2つのモデルで保たれることは書いてあったものの
の間違いです失礼しました

>>173
なるほどそういうリーマン幾何の深い結果があって
それを使うと角度も等しい事が言える的な感じなんでしょうか
本だとどうも角度もモデルによらず決まるような雰囲気で記述してあって混乱していましたが
0176132人目の素数さん
垢版 |
2021/01/27(水) 23:06:44.95ID:B9DozRHO
証明系に詳しい人が居たら聞きたいんだが、今後10年20年と見据えたとき、どのプルーフチェッカーが「勝ち」そう?
0178132人目の素数さん
垢版 |
2021/01/28(木) 16:14:44.20ID:DvPln0LA
「e^x=10である実数xは存在するか? 理由も含めて答えよ」
集合と位相の講義で出題されたんですが、よく分かりません
「log10は実数だから存在する」終わりじゃ駄目なんですかね?
0179132人目の素数さん
垢版 |
2021/01/28(木) 17:01:54.12ID:QHxiB+CU
講義中に何か説明あったんじゃないですか?
数学の問題というよりちゃんと授業を聞いているかどうかの問題な気がしますけど
0180132人目の素数さん
垢版 |
2021/01/28(木) 17:10:17.94ID:Uz7dgf6/
>>179
もし良ければ解法を教えていただけますか?
0181132人目の素数さん
垢版 |
2021/01/28(木) 17:20:28.93ID:Or24e5QC
>>178
存在すればその値をlog10と書くのではなくて?
0185132人目の素数さん
垢版 |
2021/01/29(金) 05:49:37.35ID:NpFiIfdz
>>170
> 実質的には計量を考えていることになるのだと思いますが計量という言葉をあまり出さずに書いてる本を読んでいます
> (深谷「双曲幾何」)
本見てみたら、3章の冒頭文で、双曲面モデルが上半平面モデルや円盤モデルと同じ幾何を定める旨が述べてあるし、
円盤モデルと双曲面モデルが等長的であることも書いてある(定理3.59)。

等長なんだから内積も対応するし、内積から定まる角度も対応するでしょ。
0186132人目の素数さん
垢版 |
2021/01/29(金) 07:39:30.29ID:QcgvcU0r
恥を忍んで質問させて下さい。
書籍のAbramowitz and Stegun(Handbook of Mathematical Functions with Formulas,
Graphs, and Mathematical Tables)の略称って、
・AS
・A&S
・そのほか(省略しないなど)

あと、上の略称は「解析概論」(もちろん高木先生の)の様に、
知らないとモグリ扱いなのでしょうか?
0188132人目の素数さん
垢版 |
2021/01/29(金) 17:57:56.54ID:DyprcR8u
z= √ (x^2+y^2)の1≦x^2+y^2≦9の範囲の曲面積を求める問題で答えが8 √ 2になるはずが円錐の側面積として考えると9 √ 2になってしまいます。どちらが正答でしょうか?理由もつけて教えて欲しいです。
0190132人目の素数さん
垢版 |
2021/01/29(金) 19:20:53.80ID:x2PNa3Xt
>>185
等長なら対応する内積が等しいことが言えるんですか
疑問が氷解しました
ちなみにそのことはすぐ言えるんでしょうか
0192132人目の素数さん
垢版 |
2021/01/29(金) 19:36:35.06ID:x2PNa3Xt
>>191
長さは多様体の上での距離で内積は接空間の内積ですので
何らかの局所と大域をつなげる議論が必要でしょう
その部分が疑問なのです

疑問が氷解したというのは角度が等しくなるのかどうかとか
等長群の作用について等しいという事をなぜ幾何が等しいと呼ぶのかとかが
分かったという意味です
0193132人目の素数さん
垢版 |
2021/01/29(金) 19:53:43.59ID:Q9OtMHs1
>>165
>双曲幾何の上半平面モデル(もしくは円盤モデル)と双曲面モデルとでは
>角度は等しいのでしょうか

等しい

ただ、モデルは「見え方」だから、
円盤でもポアンカレモデルとクラインモデルでは見え方が違う
つまりクラインモデルでは見た目違う角度が、モデル内の合同変換で写り合う
ポアンカレモデルでは見た目の角度も同じになる
(ただしポアンカレモデルのほうでは直線が円弧になったりする)

投影の仕方の違いだけなので、
それぞれのモデルの間で1対1の対応がつけられ
結局同型であることがわかる
0194132人目の素数さん
垢版 |
2021/01/29(金) 19:54:42.90ID:NpFiIfdz
>>190
191に答えがあるが、補足。
・長さというのは、接ベクトルのノルムを積分したもの。
・角度というのは、接ベクトルの内積から決まるもの。
・接空間において、ノルムと内積は表裏一体。(191)
あとは、まとめれば良い。
0195132人目の素数さん
垢版 |
2021/01/29(金) 19:55:34.61ID:+uIwD83a
>>192
接続とはまた別の話だよ
0196132人目の素数さん
垢版 |
2021/01/29(金) 19:56:49.07ID:NpFiIfdz
>>192
> 長さは多様体の上での距離で内積は接空間の内積ですので
> 何らかの局所と大域をつなげる議論が必要でしょう
> その部分が疑問なのです
曲線をパラメータ付けして、0からsまでの部分の長さl(s)をsで微分すれば、接空間でのノルムが出てくる。
0199132人目の素数さん
垢版 |
2021/01/29(金) 20:08:21.01ID:x2PNa3Xt
>>196のレスで長さが等しいことから接空間のノルムが、よって内積が等しい事が分かりました
その部分が理解できていなかったようです
いろいろとありがとうございました感謝です

見た目の長さや角度とは違ってくる事はクラインモデルをいじって理解できていましたが
>>197も読んでみます
0200132人目の素数さん
垢版 |
2021/01/29(金) 23:17:27.84ID:5poGgjWr
z=‪√‬(12-x^2-y^2)とz=x^2+y^2で囲まれる体積を求めたいのですが積分領域をどうすればいいのかいまいち分かりません。x=rcosθ y=rsinθとおいてrの範囲はわかったんですがθの範囲をどうすればいいのでしょうか??
0201186
垢版 |
2021/01/29(金) 23:25:44.50ID:QcgvcU0r
「A&Sのこの項目」という表現は、Maximaのソースで知りました。

>>187
本自体はこんなやつです。
ttps://ja.wikipedia.org/wiki/Abramowitz_and_Stegun

以前雑誌記事で、参考文献に著者名抜きで「解析概論」って書いてあるのを見た事があって、
著者が分からずに学校の先生に訊いたら、どうも知ってるのが常識らしく…。
いまはそれではacceptされないんだろうなぁ。
0203132人目の素数さん
垢版 |
2021/01/30(土) 17:40:11.41ID:lHyDsxQK
有向族についての質問

集合Xの元の列x_i, i∈Iが有向族であるとは、Iが有向集合であるってのが普通の定義だけど、
Xの部分集合AとAの二項関係≦のセット(A,≦)に対して有効集合を定義しちゃいけないのか?

何か不都合があるなら教えてほしい
0204132人目の素数さん
垢版 |
2021/01/30(土) 18:28:49.48ID:1+K/Bd+i
>>203
その定義でもいいと思うよ。フィルターはそうだし
ただ、有向族は数列の拡張で、数列の考え方や記法が使えるのがいいんじゃないかな
同じ添字集合の複数の有向列どうしなら収束の速さなんかの比較もできそう
0205132人目の素数さん
垢版 |
2021/01/30(土) 19:06:27.24ID:lHyDsxQK
>>204
>同じ添字集合の複数の有向列
こういう概念も(A,≦_1)、(A,≦_2)として同様に扱えると思うんだが、

唯一の、(x_i|i∈I)と(A,≦)の違いは写像と集合の違いなんだが、これが各種議論でどういう影響を及ぼすのかが分からんから聞いてみたっていう趣旨。
0206132人目の素数さん
垢版 |
2021/01/30(土) 21:25:45.56ID:3AK59yWI
K, L をそれぞれ距離空間 (X, dX), (Y, dY ) のコンパクト部分集合とする.
K × L ⊂ O となる、直積空間 (X × Y, d) における任意の開集合 O に対して,
(X, dX) の開集合 U と (Y, dY ) の開集合 V s.t. K × L ⊂ U × V ⊂ O が存在することを示せ。


コンパクトの定義を使ってOを有限部分被覆に分解するのは分かるのですが、そこから先がいまいち分かりません...
どなたかお願いいたします...
0207132人目の素数さん
垢版 |
2021/01/30(土) 21:31:43.57ID:1+K/Bd+i
>>205
そりゃ考える数学的対象と何がしたいかによるだろう
有向族の途中の項に重複があるかどうかで違いがでるものを考えるのでなければ
どっちの定義でも同等なんじゃないの? ネットとフィルターのように
でもネットとフィルターで議論の仕方は結構違うよね
0208132人目の素数さん
垢版 |
2021/01/30(土) 23:20:18.76ID:RQ48F/1U
>>206
初手から間違っている。
一般に距離空間において、コンパクト集合KとKを含む開集合Oに対し、ε>0が存在して、Kのε近傍はOに含まれる。
問題に戻る。ε>0が存在して、K × Lの2ε近傍はOに含まれる。XにおけるKのε近傍をU、Lのε近傍をVとすると、U × VはK × Lの2ε近傍に含まれ、したがってOに含まれる。
これでわからなかったら、もっと前のところの理解が怪しくなっている。
0209132人目の素数さん
垢版 |
2021/01/30(土) 23:22:51.77ID:RQ48F/1U
K × L ⊂ U × Vを書き漏らしたが自明だから問題ないね。
0210132人目の素数さん
垢版 |
2021/01/31(日) 00:17:27.24ID:1GtbAOjB
>>208
それを使ったら簡単だよなー
まあ、その証明も簡単だが
ところで直積空間の距離は無数にあるけど、どう定義してんの?
0211132人目の素数さん
垢版 |
2021/01/31(日) 00:31:13.15ID:md3DB+Z5
私は各成分の距離の和なんだろうと補った
0212132人目の素数さん
垢版 |
2021/01/31(日) 00:32:28.92ID:y3ytn68g
>>208
ありがとうございます!
「コンパクト集合KとKを含む開集合Oに対し、ε>0が存在して、Kのε近傍はOに含まれる。」という定理を恥ずかしながら知らなかったのですが、どこかのサイトに証明が載ってたりしますか?
0213132人目の素数さん
垢版 |
2021/01/31(日) 01:01:48.30ID:md3DB+Z5
Oが全集合ならば自明。
そうでないならOの補集合は空でない閉集合。
コンパクト集合と、それと交わらない空でない閉集合との距離は正である。
以上により従う。
0216132人目の素数さん
垢版 |
2021/01/31(日) 20:33:48.81ID:WZTjWF7h
いやNG判定で弾かれるのを嫌ってURL表記を途中でブツ切りにしてあるだけだろ
今は何故かリンクを貼れるが前スレ当時はNGで弾かれてたんだのを
新スレ立てる時にリンクを貼れるか試さずに立てたんだろ
http://mathmathmath.dotera.net
0217132人目の素数さん
垢版 |
2021/01/31(日) 20:35:38.43ID:WZTjWF7h
ほれやっぱり貼れた。しかし何だって去年前半頃までNGワードだらけだったんだろうな
0218132人目の素数さん
垢版 |
2021/01/31(日) 20:39:22.59ID:FoAtuery
>>216
あーなるほど bb2cってブラウザで見てるとスペース以降見えなくて気づかなかった
ありがとう
0219132人目の素数さん
垢版 |
2021/01/31(日) 21:04:12.97ID:37sMitae
関数解析では線形写像を線形作用素と呼ぶのが普通とのこと

何でこんな無意味な言い換えってするの?
数学ではちらほらこういう無意味な言い換えあるよな
0220132人目の素数さん
垢版 |
2021/01/31(日) 21:47:42.24ID:mPPX04im
>>219
気分がそうさせるだけよ?何か問題なの?
0222132人目の素数さん
垢版 |
2021/02/01(月) 09:18:20.46ID:cq4NiitP
有界閉集合でジョルダン可測(面積確定)でないものってどんなものがあるんでしょうか。
本を読んでいたら「有界閉ジョルダン可測集合」と書かれていたものがあったので、有界閉集合ってジョルダン可測なんじゃないかと思ったのですが、判例などがあればよろしくお願いします。
0223132人目の素数さん
垢版 |
2021/02/01(月) 10:04:18.85ID:C7D672kN
なんかそういう判例集みたいな本があったような
0225132人目の素数さん
垢版 |
2021/02/01(月) 11:20:45.28ID:cq4NiitP
>>224
それって閉集合ではないですよね。
0<x<1となる無理数xは[0,1]∩Qの元ではないですが、[0,1]∩Qの集積点にはなっているので。
つまりxの近傍Vをどんなに小さく取ってもV∩([0,1]∩Q)が空集合にならないので。
0226132人目の素数さん
垢版 |
2021/02/01(月) 11:26:18.08ID:3VLHP/sk
またきやがった
質問してる風で偉そうな奴が
0227132人目の素数さん
垢版 |
2021/02/01(月) 11:31:13.20ID:cq4NiitP
>>224
すみません。勘違いしたかもしれませんが、
特性関数というのは確率論などで使うものでしょうか。
複素数値関数に見えるのですが。。。
0229132人目の素数さん
垢版 |
2021/02/01(月) 11:41:53.69ID:ylCDXE1x
ジョルダン可測ってのは境界のルベーグ測度が0って事だから
有開閉集合なら、内部とその集合自身のルベーグ測度が等しくないものがあるかどうか考えればいい
結局0かどうかで考えればいいから、内点を持たずに正のルベーグ測度を持つ集合であればよくて

普通にカントール集合でいいなこれは
0230132人目の素数さん
垢版 |
2021/02/01(月) 11:59:04.09ID:3VLHP/sk
>>227
特性関数は全然違う意味のが2つある
0232132人目の素数さん
垢版 |
2021/02/01(月) 13:00:20.60ID:OPH986+r
ハルナック集合はカントール集合の2次元版だから3次元版も作れるな
0233132人目の素数さん
垢版 |
2021/02/01(月) 13:01:24.40ID:cq4NiitP
>>228>>231
どうもありがとうございます。助かりました。
ルベーグ積分論をあまりやっていなかったので、微積の話でつまづいているものだと思っていました
ちなみにカントール集合のルベーグ測度は0だから違うような。。。
0234132人目の素数さん
垢版 |
2021/02/01(月) 15:27:24.35ID:xeODCXjX
>>233
> ちなみにカントール集合のルベーグ測度は0だから違うような。。。

こういうひとこと多いのは止めた方がいいよ
0235132人目の素数さん
垢版 |
2021/02/01(月) 15:47:30.76ID:cq4NiitP
>>234

気分を害されたなら申し訳ありません。
ただ間違ってると思ったので確認したかっただけなのですが、
それとも私が間違っているんでしょうか
0236132人目の素数さん
垢版 |
2021/02/01(月) 15:57:27.36ID:oMU5Sdem
そんなことを気にするより
>>227みたいな勘違いで人を貶した事を気にしなさい
0237132人目の素数さん
垢版 |
2021/02/01(月) 16:07:32.54ID:cq4NiitP
>>236
違うのではないかと言っただけで貶したわけではありませんし、
貶されたと思ったならそれこそ勘違いです。
ちなみに、特性関数をf(x)=0 if x ∈ [0,1]∩Q
f(x)=1 if x ∉ [0,1]∩Qと定義しても同じで閉集合になりません。
0238132人目の素数さん
垢版 |
2021/02/01(月) 16:19:57.20ID:iPzjkvA6
>>237
君、本当に神経がおかしいぞ?
リアルで対人トラブル起こしまくりだろ。
0239132人目の素数さん
垢版 |
2021/02/01(月) 16:30:37.76ID:cq4NiitP
うーん他人の感情より数学的な事実の方が大切だと思ってしまうのは良くないことなのか?
例えば>>225の段階で私の質問の目的に沿わない回答が来てしまった時、
私は閉集合でないということを指摘せざるを得なかったのですが、
何かもっと良い言い回しの仕方があったのでしょうか。
今後の参考にしたいので、「もっとこういう言い方をした方がいいよ」というのがあれば教えていただきたいです
0241132人目の素数さん
垢版 |
2021/02/01(月) 16:37:15.39ID:iPzjkvA6
>>240
君も無神経な人の1人か
0242132人目の素数さん
垢版 |
2021/02/01(月) 16:56:54.61ID:liMcVjwk
>>239
225とか227に悪意があるかどうかは知らないけど。
> うーん他人の感情より数学的な事実の方が大切だと思ってしまうのは良くないことなのか?
片方がより大切だからって他方を蔑ろにして良いなんてことないよね。
0244132人目の素数さん
垢版 |
2021/02/01(月) 19:13:24.69ID:OPH986+r
>>233 までは問題ないと思うが
>私が間違っているんでしょうか
てのは攻撃的意思を掩蔽してると解釈されるな
自覚してないと問題だね
0245132人目の素数さん
垢版 |
2021/02/01(月) 19:16:08.80ID:ScbrgHG6
あまり説得力は感じないがその指摘は間違ってはいなさそうだ
そして当然であり、承知の上で言ってるのかもしれないが、「攻撃的意思を掩蔽してる」と突飛な解釈をする側に「も」問題がある
0246132人目の素数さん
垢版 |
2021/02/01(月) 19:32:05.61ID:+QZW+o0j
>>245
馬鹿はレスしない方がいいよ
0248132人目の素数さん
垢版 |
2021/02/01(月) 20:02:34.12ID:oKIgF8Wx
……ごめんな
>>224を書き込むときに閉集合という条件が頭から抜け落ちてたんだ
どうしてこうなった
0249132人目の素数さん
垢版 |
2021/02/01(月) 20:15:17.69ID:xeODCXjX
ID:oKIgF8Wxさんは「ごめんな」が言えるが、ID:cq4NiitPは謝ることが出来ない
0251132人目の素数さん
垢版 |
2021/02/01(月) 22:48:31.94ID:dsgCCGE7
数学に対する愛があふれるここがいちばん相応しそうなので、ここで質問させて下さい。
私は高専卒なので大学へは行ってないのですが
今「数学であそぼ」というマンガを読んでいて授業の中で「自然数、整数、有理数、実数」の説明で
「切断」を使用した説明になって、主人公が「なんじゃこりゃ〜」となるシーンがあるのですが、
これを見てても主人公は「理解しよう」と努力していて他の人物は「丸暗記」しているように見えます。

私の考える「道具」に対する「理解」の判断基準の1つは「その道具を使用しての応用ができること、できない事の判定ができる」
「それはどういう理由で応用できるのかできないのかが説明できる」(厳密で無くてもいい)みたいな感じなのですが、
授業を受けて、この「理解」の状態になっているようにはとても見えません。

どういう事を言っているつもりなのかと言うとマンガの中でトイレットペーパを使って円の面積を求める考え方
を説明するシーンがあるのですが、例えばこれを例に使って説明するなら、「面積を変えずに図形を変形して
考えやすい形に変形する」という道具は、ピタゴラスの三平方の定理には応用?できて、
「余弦関数1周期とy=0で囲まれた面積を求める」には応用できない。その理由は「考えやすい形に変形するのが不可能だから」
こういう感じです。

実際どうなんでしょ?
授業を受けて上記のような感じの「理解」の状態になってるもんなんでしょうか?

以上よろしくお願いいたします。
0252132人目の素数さん
垢版 |
2021/02/01(月) 22:56:17.65ID:dsgCCGE7
「数学であそぼ」は小学館のサイトの無料お試しにあって
主人公が切断で苦悩する場面は、無料で読める範囲にあります
https://comics.shogakukan.co.jp/book?isbn=9784098702817
0253132人目の素数さん
垢版 |
2021/02/01(月) 22:59:24.20ID:F+FgG1wK
切断なんて応用が効く話じゃない。
位相概念がない時代に実数を定式化しようとした偉大なる先人の苦闘を鑑賞するもの。
0254132人目の素数さん
垢版 |
2021/02/01(月) 23:22:29.94ID:EyrHS+qF
間違った事を指摘される事に以上に嫌悪感持つ人間は数学辞めた方がいいよ
議論がマトモに成立しない
0255132人目の素数さん
垢版 |
2021/02/01(月) 23:31:46.67ID:xeODCXjX
その通り
このスレの流れとは関係ない話だけど
0256132人目の素数さん
垢版 |
2021/02/01(月) 23:36:57.29ID:EUMluB9v
>>251
そこらへんは高校まででも同じじゃないですかね
そういう高みまで行ける人もいればいけない人もいるというだけです
0258132人目の素数さん
垢版 |
2021/02/02(火) 00:44:43.52ID:mY0AvcbO
理解度をはかるのに応用ができるかどうか、なぜその定理や定義が有効か必要かを説明できるってのは確かに有用な指標ではあると思う


個人的には理解には大まかに3段階あって

一番上は定理や定義が表してる内容を理解できるし、そうした定義や定理の応用や成り立つ仕組み、有用性や条件の必要性なども説明できる

真ん中は
定理や定義が表してる内容は直感的に、イメージや意味で理解できているが、なぜ必要なのか、何に応用できるのかがよくわからない

一番下は
文字や論理記号でしか覚えていない


に分かれると思う
数学科でも一番上理解をちゃんと出来てる人間は少なくて、大半は真ん中

有名なεδとかでも、まあすんなり受け入れて覚えられる人は割と少なくないんだが
大半は真ん中か下の理解

ましてや切断なんか本当に定義の妥当性や有用性を理解できている人間は殆どいないだろうし大半は理解したといっても真ん中か下の意味合いなのだろう
しかしいきなり完全に理解しなければいけない、という事でもないので数学に慣れない新入生のうちはある程度受け入れながら進めて行く事も必要かなとは思う

ここで理解のレベルが、一番下、文字と論理構造だけの理解であればこれは丸暗記といっても過言ではないし、勉強が辛くなるんじゃないかな

せめて真ん中、イメージや意味を理解しているのであれば、これはまあ軽くは理解したといってもいいんじゃないか
これが出来ると一々文字列を丸暗記してなくても、その場で(同じ)定義を自分で作れたりできるし、簡単な性質であれば自分でその場で導けたりできる
やりながら理解度を一番上に上げていけばいいわけだし

この真ん中のレベルの理解であれば、一部の天才肌の頭のいい人間はいきなりでもそこそこ出来るよ
一方で下の理解だけで理解したと思って進めて行く人もいる
そのマンガの登場人物がどっちかはわからん
0259132人目の素数さん
垢版 |
2021/02/02(火) 01:03:17.38ID:zLyIbBYk
内容の無い長文
0260132人目の素数さん
垢版 |
2021/02/02(火) 05:39:53.04ID:tTWcmhnH
そうだな、論理100%脳のお前には無内容だな
人間には含蓄ある長文だ
0261132人目の素数さん
垢版 |
2021/02/02(火) 06:44:45.34ID:wNQv70U+
>>238
別にそうは思わんな
>>239
それでいいと思うよ
0262132人目の素数さん
垢版 |
2021/02/02(火) 06:45:39.42ID:wNQv70U+
>>240
俺もそうとしか思えん
>>241
無神経というのとはまた違うのでは?
0263132人目の素数さん
垢版 |
2021/02/02(火) 06:50:45.18ID:wNQv70U+
>>252
四畳半神話体系の登場人物みたいじゃないのか
0264132人目の素数さん
垢版 |
2021/02/02(火) 06:55:33.10ID:wNQv70U+
>>251
一辺切断勉強してみたらどう?理解できたらそういう感じの理解かどうか理解できるかも
0265132人目の素数さん
垢版 |
2021/02/02(火) 07:10:10.66ID:z5nsINww
定義の話とかだと、「なんでこんな定義が出てきたんだ」「この定義に何の意味があるんだ」という事がわからなくて混乱する人間は多い
>>258でいえば1番上の理解に到達していないって事だろう
もちろんいきなりそのレベルで修得できないのは学部1年生であれば自然な事だから大半の人間はその何故や何を飲み込んでとりあえず受け入れて進めていくのだが
それが出来ない人間がたまにいて、そういう人は不幸にもそこで躓いてしまうのだろう
しかしこの漫画の主人公はそういうレベルではなくて、単に理解できてないだけのように見える

理解した組は丸暗記でもおかしくないし、普通にまあ起こってることも理解できてるよ、ってのもそこそこはいる
0266132人目の素数さん
垢版 |
2021/02/02(火) 07:13:21.37ID:z5nsINww
やっぱ説明でもなんでも図は書いた方がいいよ
微積分や解析の定理で、内容を図と日本語で説明できない奴は理解してないと言える
説明できれば理解できてる
シンプルに言えばこれだな
0267132人目の素数さん
垢版 |
2021/02/02(火) 10:11:53.42ID:ApdwQ769
イメージとか図とか、昭和の頃の数学者の考え方が未だに残ってるんだよね
とりあえず知識を身に着けてればそれで良く、何が研究を成功させるかなんてエビデンスもないわけで、イメージが大事だという結論を導くことなんて出来ないんだが、過去の成功者が「数覚」とかを後任に説いたから、それが未だに残ってるんだろう(日本の数学界は門戸が狭い故に多様性が低く、考え方が統一されてしまっていると思われる)
例えば斎藤毅さんによれば、グロタンディークは「スキームXといえば、ただXと思っていたのかもしれない」ようだが、上の人によればグロタンディークは数学を理解できていないのだろうか
だとすれば、苦笑せざるを得ない
0268132人目の素数さん
垢版 |
2021/02/02(火) 10:22:16.62ID:zLyIbBYk
グロタンディークはブルバキに操られたパワー系なんちゃら
0269132人目の素数さん
垢版 |
2021/02/02(火) 10:50:17.27ID:wNQv70U+
>>267
>過去の成功者が「数覚」とかを後任に説いた
誰それw
イメージは大切だよ
スキームはスキームというイメージで
0270132人目の素数さん
垢版 |
2021/02/02(火) 10:50:39.64ID:wNQv70U+
イメージは別に幾何的なものに限らん
0271132人目の素数さん
垢版 |
2021/02/02(火) 11:44:16.67ID:PQAuxIK1
イメージ=視覚的図形と短絡しているから>>267のようなトンチンカンなことが言えるのだろう
0272132人目の素数さん
垢版 |
2021/02/02(火) 11:48:53.02ID:NtTRUcTw
>>270
グロタンディークはスキームXに対して「X」をイメージしているが、
それで良いならそれが出来ない人は存在しないので取り立てて言うことではないな
0273132人目の素数さん
垢版 |
2021/02/02(火) 11:51:48.99ID:zLyIbBYk
>>272
斎藤毅の推定に過ぎないものを論拠に、そこまで強い主張ができるのが不思議だ。
0274132人目の素数さん
垢版 |
2021/02/02(火) 11:56:44.44ID:NtTRUcTw
>>273
逆にグロタンディークはもちろんのこと、斎藤毅先生より立派な人もこのスレにはいないのに、
なんの根拠もなく先生の解釈を否定しにかかるのが不思議だ
0275132人目の素数さん
垢版 |
2021/02/02(火) 12:04:39.98ID:EO++lLHL
数学の定理なら1人が一回証明したらそれでいいが、グロタンディークに会ったこともない数学者による思考過程の推定を資料の裏付け無しに論拠にするのは厳しい。
グロタンディークと学問的交流のある数学者の証言が欲しいね。
0276132人目の素数さん
垢版 |
2021/02/02(火) 12:41:16.88ID:NtTRUcTw
>>275
数学を深く理解した結果としてグロタンディークはこう考えていたとするものをエビデンスなしに否定するほうが難しいと思うけど

ちなみにグロタンディークと学問的交流のあったデイヴィッド・マンフォードによると、
グロタンディークは具体的に考えていない。
私は例を通じて物事を理解し徐々にそれらをより抽象的にするが、少なくとも例を見ることが彼を助けたとは思わない。
と述べている。
斎藤毅先生も同ペーパーで述べているとおり、抽象数学は記号はただの記号であるということが大事であって、マンフォードや先生から見てグロタンディークはその権化に思えたんだろう
(ちなみに斎藤毅さんはただの記号であることが大事だが、そう思ってはいけないとも述べており、その上でグロタンディークはただXだと思っていたのではないかと述べていることからも、自分の都合のいいようにグロタンディークを解釈しているわけではないことが読み取れる)
0277132人目の素数さん
垢版 |
2021/02/02(火) 13:08:10.92ID:ApdwQ769
ちなみにマンフォードの談を付け加えると、
ザリスキは詰まったときによく曲線を黒板に描き、そこから代数へ入っていたが、
グロタンディークはこれを決してしなかっただろうし、極端に簡単でほとんど自明なものを除いて実例から研究しなかったし、ホモロジーの図式を除いてほとんど絵も描かなかった、とも述べている
しかもグロタンディーク自身も、数学で他の何よりも私を魅了する一つのことがあるとすれば、それは数でも数量でもなく、常に『形式』である、と述べている

こういう方法で『も』数学はできる(しかも歴史上トップクラス)という事実は、多くの人に理解されないかもしれないが、間違いなくある
0278132人目の素数さん
垢版 |
2021/02/02(火) 13:15:52.27ID:zLyIbBYk
やっぱりイメージ=図形と短絡してるんだな
ポントリャーギンだって図形では考えてないよ
0279132人目の素数さん
垢版 |
2021/02/02(火) 13:16:37.87ID:hlE32Q8a
ホモロジーの図式も立派なイメージだと思う
ホモロジー論最初に勉強した時はなんだかよく分からんかったけど圏論の簡単な本読んだらかなり分かる様になったし
0280132人目の素数さん
垢版 |
2021/02/02(火) 13:23:18.27ID:ApdwQ769
>>278
ポントリャーギンの歴史を知らなくて申し訳ないが、それを説明してもらえるとありがたい
0281132人目の素数さん
垢版 |
2021/02/02(火) 13:27:38.26ID:PQAuxIK1
>>280
ポントリャーギンは視覚を失っている
0286◆QZaw55cn4c
垢版 |
2021/02/02(火) 20:37:07.67ID:YQvA8W+C
>>253
私の解析の教科書はデデキントからスタートするのですが、これってやっぱり古いのですか?
0287132人目の素数さん
垢版 |
2021/02/02(火) 22:08:52.89ID:XLuDKoRL
よく言われる
(1階or2階)同次微分方程式 と 同次型微分方程式
って全く別のものですよね?
0288132人目の素数さん
垢版 |
2021/02/02(火) 22:11:32.78ID:wNQv70U+
>>286
いろいろ読んでみては?
0292132人目の素数さん
垢版 |
2021/02/03(水) 15:03:55.76ID:eFzNl/GF
彡(^)(^)「数学分からんなーせやっ!数学板で聞いたろ!」
0296132人目の素数さん
垢版 |
2021/02/03(水) 17:37:26.15ID:9jqw3TAl
289です
ありがとうございました

続けざまで申し訳ないのですが
半順序集合が完備半束ならば完備束である
ことに対して質問です
自然数全体から0を除いたものN\{0}は整除関係のもとで順序を考えたとき、任意の部分集合が下限を持つが完備束では無いと思うのですがどこがおかしいのでしょうか
0297132人目の素数さん
垢版 |
2021/02/03(水) 17:47:39.30ID:nA8Fu5x0
>>287は自己解決。やっぱ違うよね
でもいくらなんでも紛らわしすぎるよ 前者は斉次微分方程式と言うことにして区別したほうがよさそうだな
英語でもどっちもhomogenousと言うようだがなぜそんなことに
0298132人目の素数さん
垢版 |
2021/02/03(水) 18:22:49.53ID:O02t1RNg
複素解析です、よろしくお願いします
四行目からの一様収束の証明で、zを与えたあとにnの下限をzに依る形で定めて収束することを示していますがこれでは各点収束しか言えていないのではないですか?

https://i.imgur.com/1ADbNGo.jpg
0299132人目の素数さん
垢版 |
2021/02/03(水) 18:37:04.08ID:9jqw3TAl
>>296
自分で考えたんですけど、空集合の下限は整合的に定義しようとするとNの整除関係における最大値である0であるから、そもそもN\{0}では空集合に下限が定義できないということでしょうか
0303132人目の素数さん
垢版 |
2021/02/03(水) 18:57:36.71ID:O02t1RNg
>>300
すみません、もう少しくわしくお願いします……( ; _ ; )
0304132人目の素数さん
垢版 |
2021/02/03(水) 19:08:01.51ID:ckjStsau
吉田洋一著『ルベグ積分』に以下の問題とその解答があります。

「GがRにおける有界な開集合ならば、Gは開区間の列の直和として表わされることを証明する。」

解答:
https://imgur.com/OBnjcB9.jpg

解答に、

(α(x_1), β(x_1)) ∩ (α(x_2), β(x_2)) ≠ 空集合



(α(x_1), β(x_1)) = (α(x_2), β(x_2))

と書いてありますが、その理由は以下でOKですか?


y ∈ (α(x), β(x)) とする。

(1) y = x の場合

(α(x), β(x)) = (α(y), β(y)) である。

(2) y > x の場合

(α(x), β(x)) ⊂ G である。

α(x) < x < y < β(x) だから (y, β(x)) ⊂ G

よって、

β(x) ≦ β(y)

(y, β(y)) ⊂ G である。
また、 (x, y] ⊂ G である。
よって、(x, β(y)) ⊂ G である。

∴β(y) ≦ β(x)
∴β(y) = β(x)

α(x) < x < y < β(x) だから (α(x), y) ⊂ G
よって、α(y) ≦ α(x) である。

(α(y), x) ⊂ (α(y), y) ⊂ G である。
よって、α(x) ≦ α(y) である。
∴α(x) = α(y)

以上より、(α(x), β(x)) = (α(y), β(y)) である。

(3) y < x の場合

(2)と同様にして、(α(x), β(x)) = (α(y), β(y)) である。


y ∈ (α(x_1), β(x_1)) ∩ (α(x_2), β(x_2)) とする。
上で示したことから、
(α(x_1), β(x_1)) = (α(y), β(y)) = (α(x_2), β(x_2))
である。
0305132人目の素数さん
垢版 |
2021/02/03(水) 19:08:32.40ID:ckjStsau
解答がスマートでないと言われてしまったのですが、スマートな解答はどんな感じになりますか?
0306132人目の素数さん
垢版 |
2021/02/03(水) 19:41:40.39ID:ByqtypZk
つまんないとこばっかりで引っかかってるなぁ
相変わらず
とっとと先進めよ
0308132人目の素数さん
垢版 |
2021/02/03(水) 23:45:11.44ID:U9t5evhX
3点(0,0),(0,2),(4,2)を頂点とする三角形領域上における曲面z=x+y^2の曲面積を求めよ
0309132人目の素数さん
垢版 |
2021/02/04(木) 01:31:30.93ID:bKgsh2v0
>>307
8行目のn>…の右辺がzに依っていて、そのnに対してしか成り立つことを言えていないと思いました
右辺はzの値によりいくらでも大きくなるので、やはり一様収束が示せていないのでは……?
0310132人目の素数さん
垢版 |
2021/02/04(木) 02:11:24.47ID:UhQYF54l
>>308
マルチはともかく、大学生ならもうちょっとまともに書かないと
0312132人目の素数さん
垢版 |
2021/02/04(木) 06:14:03.49ID:LXiG8o2I
>>309
zとε>0があたえられたとき、ある数値より大きいnに対し、Ω上でfn(z)とzの差の絶対値がε未満、と言えてるんだからOK
0313132人目の素数さん
垢版 |
2021/02/04(木) 06:30:49.47ID:35+zx1Il
書き直し

絶対値が1未満のz_0とε>0があたえられたとき、ある数値より大きいnに対し、|z|<|z0|上でfn(z)とzの差の絶対値がε未満、と言えてるんだからOK
0314132人目の素数さん
垢版 |
2021/02/04(木) 13:55:14.81ID:z5BMvlW9
廣中・森の「代数幾何学」のP121について教えて下さい
E^{p,q}_∞ = Z^{p,q}_∞/B^{p,q}_∞ + Z^{p+1,q-1}_∞とあるのですが、+ Z^{p+1,q-1}_∞の部分はなぜ必要なのでしょうか?
E^{p,q}_∞ = Z^{p,q}_∞/B^{p,q}_∞だと思うのですが。。。
0315132人目の素数さん
垢版 |
2021/02/05(金) 01:05:15.63ID:bLOEuYHC
ある数学の講義の期末試験で広義二重積分の問題が出て変数変換してもうまくできずそのまま逐次積分しようにも原始関数がなかなか求まらないのでwolframalphaにぶち込んで不定積分求めさせようとして
xで積分すると「標準的な数学関数での結果が見つかりません」と出て
yで積分すると結果のところにerfiとかEiが出てきたんですけどこれは作問ミスでしょうか?
特定が怖いので問題は載せられませんがよろしくお願いします
0316132人目の素数さん
垢版 |
2021/02/05(金) 01:42:33.78ID:8Ki8/0Nl
俺たちはエスパーじゃない
0318132人目の素数さん
垢版 |
2021/02/05(金) 08:10:06.30ID:krlBaLLZ
>>315
不定積分が初等関数で表されなくても積分区間によって積分値が具体的に求まることはある。
ましてや、重積分の広義積分だろ。
累次積分での積分順序や変数の置換によって、途中の計算が簡単だったり難しかったり初等関数で表せなかったりいろいろ。
erfやEiが出てきたからって作問ミスとは言えない。

というか、正規分布の分布関数でも積分させているんじゃないの?
そんなのいくらでも教科書に載ってるだろ。
0320132人目の素数さん
垢版 |
2021/02/05(金) 12:24:44.75ID:X1QFZZBd
>>315
問題ぐらい書けや
0321132人目の素数さん
垢版 |
2021/02/05(金) 12:28:27.19ID:sGjxiyyu
レポートの問題書いたらカンニングがバレちゃうじゃん
0323132人目の素数さん
垢版 |
2021/02/05(金) 14:33:12.81ID:bLOEuYHC
>>318
回答ありがとうございます
参考にさせてもらいます
0324132人目の素数さん
垢版 |
2021/02/05(金) 14:39:49.00ID:/7ErvALH
線形代数の質問なのですが、ベクトルから座標を取り出すのってどうやって計算するんでしょうか.
具体的にはV: K線形空間, b=(b_1, ..., b_n): Vの基底 のとき,
写像φ_b: K^n -> V; (v_i)_{i=0}^{n-1} -> Σ_{i=0}^{n-1} v_i b_i
の逆写像を計算する方法を知りたいです
Vに内積*があれば, bを正規直交基底eにして, eからbへの基底の取り替え行列Pを求めて,
(v_i)_{i=0}^{n-1} = P^(-1) ((e_i * v)_{i=0}^{n-1}) を計算すれば良いのですが, Vに内積が定義されてないときどうすれば座標が求まるのでしょうか.
0325132人目の素数さん
垢版 |
2021/02/05(金) 16:03:55.53ID:X1QFZZBd
>>324
解けよ
0327132人目の素数さん
垢版 |
2021/02/05(金) 17:08:23.00ID:UmsnBxvg
v=Σvjbjと表されるとすれば(bi,v)=Σvj(bi,bj)だから
(bi,bj)を成分に持つ行列Bの逆行列B^(-1)を使って
vi=Σ(B^(-1))ij(bj,v)と解ける
0328132人目の素数さん
垢版 |
2021/02/05(金) 17:10:06.09ID:X1QFZZBd
>>326
解けよ
0329132人目の素数さん
垢版 |
2021/02/05(金) 17:12:14.43ID:X1QFZZBd
だいたい
内積あれば解けるって自分で言ってるんなら
内積入れろよ
内積は普通は入れなくて解くけどな
0332132人目の素数さん
垢版 |
2021/02/05(金) 17:49:01.28ID:UmsnBxvg
ごめん、ちゃんと読んでなかった
vやbiが数ベクトルとしての表示があると仮定していいならbiたちを並べた行列使って同じように解くことは可能だけど、それなら結局、数ベクトルとしての内積を使って解いたのと変わらない気もする
0333132人目の素数さん
垢版 |
2021/02/05(金) 18:01:26.51ID:/7ErvALH
φ_bの定義が明示的なので逆写像も具体的に記述出来るかと思ったんですが無理そうですね
内積とか何かプラスアルファが無いといけなさそうですね
0334132人目の素数さん
垢版 |
2021/02/05(金) 19:00:38.24ID:UmsnBxvg
たしかに言われてみれば有限体上とかだとどうするんだろうね
そもそも抽象的なベクトルを基底使って表示するのは計算で与えられるというより定義と思うべきなのか
数ベクトル表示にせよ内積(bi,v)を既知とするにせよ、どこかで既にベクトルvの基底に対する情報は分かっているわけで、いきなりv=vibiと書いてしまうことと大差ないのかも知れない(係数viが逆写像の定義としてしまう)
0336132人目の素数さん
垢版 |
2021/02/05(金) 21:16:00.81ID:X1QFZZBd
>>335
それだとトートロジーで意味なし
0340132人目の素数さん
垢版 |
2021/02/06(土) 19:43:11.16ID:oOR93MJx
>>339
双対基底を習っていたら >>333 であることが分かっているはずだから
習っていないのかな、と思っただけ
0342132人目の素数さん
垢版 |
2021/02/06(土) 21:39:41.95ID:LR53yx4D
ここの回答者は表現行列の概念すらわかってないレベルの低い方しかいないということがバレてしまいましたね
0343132人目の素数さん
垢版 |
2021/02/06(土) 22:07:50.66ID:JPcxMNTU
そもそも内積なければ足し算とスカラー倍ぐらいしか基本演算はないんだから
それで座標を出すのは無理だよ
別でそれ用の演算定義すればいいけどそれは結局内積定義してやってるのと大差ないし
0348132人目の素数さん
垢版 |
2021/02/07(日) 08:30:30.58ID:+FSt6Wwe
>>345
解くしかないから
0349132人目の素数さん
垢版 |
2021/02/07(日) 11:39:36.06ID:8cQcFgLj
何を解くか知らないけど、もし行列の方程式を解けって言う話ならまず数ベクトル表示が得られてることが前提でしょう
0350132人目の素数さん
垢版 |
2021/02/07(日) 18:39:07.19ID:M8aNtWvU
第二形式の帰納法に慣れたいので第二形式の帰納法を利用して証明する
問題が載った問題集を買いたいのですが、どういうのがありますか?
0353132人目の素数さん
垢版 |
2021/02/09(火) 16:02:04.79ID:ahNZu6s5
可微分多様体の上の滑らかで単射な曲線γがあって
dγ/dt=0となる点が離散的なときγは可微分な埋め込みである事を示せという問題がわかりません
微分が0である点での逆写像の滑らかさはどのようにして言えるのでしょうか
0357132人目の素数さん
垢版 |
2021/02/09(火) 18:39:15.25ID:1HroZ2ho
>>353
>単射な曲線
0362132人目の素数さん
垢版 |
2021/02/11(木) 00:29:17.25ID:MPXzTgUc
F"= - ξ F'/2 → F"/F' = - ξ/2 → log(F') = - ξ^2/4 + C1 → F' = C2 exp(- ξ^2/4)
→ F = C2 ∫ exp(- ξ^2/4) dξ + C3 = C4 ∫ exp(- s^2) ds + C3
0364132人目の素数さん
垢版 |
2021/02/11(木) 11:37:49.34ID:3FqAnyM9
京大一般講座の普遍代数のプリント
http://www.kurims.kyoto-u.ac.jp/~kenkyubu/kokai-koza/H30-terui.pdf
の、p.11真ん中あたり、直既約な代数の直積に分解しない代数の例が思い浮かびません。
例えば何がありますか?
0365132人目の素数さん
垢版 |
2021/02/11(木) 12:07:44.73ID:xfcUeXRF
>>364
R[x]のコピーRi無限個用意しといて
A = { (ri) ∈ Π Ri | 有限個を除いてRi∈R }
とかでいけるのでは?
因子としてはRiしかないけど全部かけるとAを超えてしまう
0367132人目の素数さん
垢版 |
2021/02/11(木) 22:59:48.62ID:3FqAnyM9
>>365
366です
この場合、A自体が直既約な代数で、自明な代数とA自身との直積に分解されるので、私の求めている代数と違うかなと思いました

それ自身が直既約でなく、その自明で無い直積分解の因子となる代数にも自明で無い直積分解が存在して…と、直既約な因子が現れずに際限なく自明でない分解が行える代数の例を探しています
0368132人目の素数さん
垢版 |
2021/02/11(木) 23:04:08.26ID:xfcUeXRF
>>367
直規約じゃないでしょ
添字の集合をI=I1∪I2とか分けて
A1=A∩Π[i∈I1]Ri,A2=A∩Π[i∈I2]Ri
とでもすればA=A1×A2でしょ?
0370132人目の素数さん
垢版 |
2021/02/14(日) 19:16:47.82ID:+GF8x0MQ
wikipediaのモノイドの記事について、モノイドの構造の可換モノイドのところ

https://ja.wikipedia.org/wiki/%E3%83%A2%E3%83%8E%E3%82%A4%E3%83%89

「任意の可換モノイド M は
x≤y⇔x+z=y ∃z∈M
として定まる代数的前順序 "≤" を持つ」
とあるんですが、非可換モノイド(M',・,1)でもx≤y⇔x・z=y ∃z∈M'と定めれば、
x・1=xよりx≤x、x・c=yかつy・c'=zならば
(x・c)・c'=x・(c・c')=zよりx≤yかつy≤zならばx≤zなので前順序になると思います。

やはり可換でないと代数的前順序を持ちませんか?
0371132人目の素数さん
垢版 |
2021/02/14(日) 23:00:10.06ID:0T1voGBJ
>>370
可換→持つ

非可換→持たない

同値?
0372132人目の素数さん
垢版 |
2021/02/14(日) 23:19:33.40ID:Mq/JFvLD
>>371
いいえ、私は
モノイドが可換→持つ

モノイドが非可換→持つ
だと思ったので
モノイド→持つ
であって、その順序の定義に可換性は必要ないと思いました。

それでも、可換モノイドの欄に書かれているので
可換→持つ

非可換→持たない
であって、自分がどこか間違っているのかと思いました。
0373132人目の素数さん
垢版 |
2021/02/15(月) 06:18:10.68ID:hR8IspM6
>>372
例の無神経な奴だ。
「いいえ」から始める失礼さがわからない。
0374132人目の素数さん
垢版 |
2021/02/15(月) 06:50:39.47ID:yIXO9tvg
言うほど失礼か?
質問への回答を真っ先にするのはむしろ潔いじゃないか
0376132人目の素数さん
垢版 |
2021/02/15(月) 07:24:11.38ID:JKD78jir
>>370
代数的順序の定義が不明確だが、「a≦bならばac≦bcかつca≦cb」だとすると、非可換の場合ac≦bcは一般には言えない。
なお、群ならすべてのa,bに対しa≦bなので非可換でも自明な代数的前順序が入る。
0377132人目の素数さん
垢版 |
2021/02/15(月) 09:28:51.17ID:HUOgFNZ4
単に今んとこそんな定義にしても面白い応用例がないだけちゃうの?
数学的概念は定義域が広ければいいというもんでもない
結局その概念定義してその後面白い話展開するなら、そこから「非可換モノイド」なるものの知られてる性質を利用することになるけど、それでは大したことができないなら、結局話を可換の場合に限るしかない
すると実質話に可換の場合の話しか出てこないのに毎回propositionやtheoremの仮定の中に“可換の場合には”と書かされることになる
使われることもない非可換の場合のために毎回そんなウザい記述をさせられるのは意味ないから元から話に入ってないんやろ
0379132人目の素数さん
垢版 |
2021/02/16(火) 09:36:57.45ID:Pz6Vs65U
>>376
>代数的順序の定義
なるほど単なる順序じゃ無いんだ
定義はそれかな?順序関係が左右からの演算で保存
0381132人目の素数さん
垢版 |
2021/02/16(火) 20:43:14.55ID:AvFYwScL
ちなみに、πへ収束するn項和f(n),g(n)に対して、
f(n)はg(n)より収束が早い :⇔ ∃N、N≦∀n |f(n)-π|≦|g(n)-π|
っていう意味です
0383132人目の素数さん
垢版 |
2021/02/17(水) 14:48:28.61ID:82KILvW7
それだとf(n)=πが一番早いことに…
f(n)!=πとしても
f(n)=π+h(n) where h(n) = if n==0 then 1 else 0
が一番早そう
というツッコミは良いとしてどういうハードウェアに計算させるかにかなり依存すると思う
0385132人目の素数さん
垢版 |
2021/02/18(木) 16:18:31.26ID:F7gcKB9O
双曲平面のガウス曲率を求めよという問題で自分は定義に従って計算したのですが
もっと楽な方法があるのでしょうか

自分は円盤モデルの計量からレビ・チビタ接続を定義である計量と両立する事と対称性から求めて
そこから曲率テンソルを調べてガウス曲率を計算しました
ただ問題のヒントに1点でのガウス曲率を計算すればよく円盤モデルの原点で計算すると楽だという記述があり
自分の方法ではそのように考えても全然楽にならないので、もっと良い計算法があるのかと疑問に思っています
0386132人目の素数さん
垢版 |
2021/02/18(木) 18:42:34.84ID:jsvclIk2
円盤モデルって負の定曲率のやつでしょ?
双極面って定曲率じゃないんじゃないの?
0387132人目の素数さん
垢版 |
2021/02/18(木) 19:35:21.37ID:F7gcKB9O
>>386
双曲平面とは円盤モデルやら上半平面モデルやらで表される空間の意味です
なので言い換えると「(2次元の)円盤モデルが負の定曲率であることを示せ」
という問題を考えています
0388132人目の素数さん
垢版 |
2021/02/18(木) 20:10:00.06ID:jsvclIk2
>>387
イヤ、定曲率であるのは当たり前やん
等長の自己同相の全体が可遷に作用してるんやから
なので等曲率なのは当たり前なのでまさに原点での曲率計算すれば良い
0389132人目の素数さん
垢版 |
2021/02/18(木) 20:16:05.97ID:F7gcKB9O
>>388
そこは分かっています
ただ上に書いた方法で計算すると原点での値を求めるまでもなく
計算でそのまま任意の点で曲率が負の一定値であることが出てしまうので
わざわざ原点について考える必要性がないのです

なのでもっと別の方法で原点の曲率だけなら簡単に計算が出来るのかというのが疑問です
0390132人目の素数さん
垢版 |
2021/02/18(木) 20:26:12.78ID:jsvclIk2
>>389
それは>>385でいってるのと話ちゃうやん
>>385のヒントで言われてる事は>>388でオレが書いた事でしょ?
等長写像が可遷に作用してるから当然定曲率、その曲率を求めてください、もちろん原点に限って計算すれば終わりですよ、がヒントでしょ?
もちろんそれより楽な方法なんかないやろ
それが気に食わなくてわざとしんどいやり方やりたいならしんどいのは覚悟してやるしかないでしょ?
0391132人目の素数さん
垢版 |
2021/02/18(木) 20:35:52.71ID:F7gcKB9O
>>390
原点に限って計算するという部分が疑問なのです
原点に限って計算するということが可能なのですか?
気に食わないとかそういった話ではなく、>>385の後半で書いたような方法で計算すると
全体での曲率が求まってしまうのです
ずっとその事だけを質問しているのですがどうもうまく説明できていないようで
0392132人目の素数さん
垢版 |
2021/02/18(木) 20:40:14.72ID:jsvclIk2
>>391
実際やってみればいい
原点だと対称性でx座標についてコレコレのあたいなら当然y座標についてやった場合の値はコレコレが同じ作業になるので半分になる
何せ計量テンソル16個出てくる
色々対称性あるからもちろんそんなにないけど原点のありがたみ使わないなら止めはしない
やってみて平面の場合ですらどれだけ大変か味わってみるといい
0393132人目の素数さん
垢版 |
2021/02/18(木) 20:43:58.11ID:tQOxJHma
例の失礼な奴だ
下手に相手をするからどんどん増長していく
0394132人目の素数さん
垢版 |
2021/02/18(木) 20:49:51.17ID:F7gcKB9O
>>392
自分でやってみたのですがその課程の概略を書いたらどこが簡単になるか教えて貰えるでしょうか
(そんなには大変じゃなかったのでもしかしたらそもそもどこかで間違えている気もしてきました)
0395132人目の素数さん
垢版 |
2021/02/18(木) 21:04:49.00ID:F7gcKB9O
円盤モデルの計量はg=4/(1-x^2^y^2)^2{dx^2+dy+2}なので
<∂x,∂x>=<∂y,∂y>=4/(1-x^2-y^2)^2
<∂x,∂y>=0
これらの式の共変微分を考えて(r=1-x^2-y^2と略記します)
2<∇x∂x,∂x>=∂x<∂x,∂x>=16x/r^3=∂x<∂y,∂y>=2<∇x∂y,∂y>
0=∂x<∂x,∂y>=<∇x∂x,∂y>+<∂x,∇x∂y>
0=∂y<∂x,∂y>=<∇y∂x,∂y>+<∂x,∇y∂y>
2<∇y∂y,∂y>=∂y<∂y,∂y>=16y/r^3=∂y<∂x,∂x>=2<∇y∂x,∂x>
これらの式から接続は
∇x∂x=2/r(x∂x-y∂y)
∇x∂y=∇y∂x=2/r(y∂x+x∂y)
∇y∂y=2/r(-x∂x+y∂y)と求まる
長いのでいったん切ります
0396132人目の素数さん
垢版 |
2021/02/18(木) 21:18:18.42ID:F7gcKB9O
計量テンソルRmについて[∂x,∂y]=0を使って計算すると
Rm(∂x,∂y,∂y,∂x)=<∇x∇y∂y-∇y∇x∂y-[∂x,∂y]∂y,∂x>
=<∇x{(2/r)(-x∂x+y∂y)}-∇y{(2/r)(y∂x+x∂y)},∂x>
=<-∂x(2x/r)∂x-(2x/r)∇x∂x+(2y/r)∇x∂y-∂y(2y/r)∂x-(2y/r)∇y∂x-(2x/r)∇y∂y,∂x>
=…
=-16/r^4となり
ガウス曲率の定義はこれを
|∂x|^2|∂y|^2-<∂x,∂y>=16/r^4
で割ったものなので曲率は-1
として求めました

この計算は原点ではなく一般の点での計算になっていると思うのですが
原点に限るとこれよりも簡単になるのか(どこかが省略できるのか)が疑問なのです
よろしくお願いします
0397132人目の素数さん
垢版 |
2021/02/18(木) 21:19:06.69ID:XZSUxj8X
>>395
だから書いたやん?
そうやって計算して出てきた式にx=0,y=0での値を求める
つまり
Rxxxx(0,0), Rxxxy(0,0),‥,Ryyyy(0,0)
を計算していくけど、原点ならRxxxx(0,0)=Ryyyy(0,0)は当然同じ値になる
そもそも

R(0,0)=cが成立する



R(x,y)=cが(x,y)の恒等式になる

なら作業の量当たり前に違うやん?
(x,y)=(0,0)ほりこんで値比較するだけなのと、関数計算してそれを整理して定数関数になるのを確認する作業量が同じなわけないやん?
0398132人目の素数さん
垢版 |
2021/02/18(木) 21:28:53.32ID:F7gcKB9O
>>397
上で書いた課程ではどの段階で(x,y)=(0,0)を入れるのでしょうか?
ガウス曲率は勝手に取った基底について計算すればいいので
曲率テンソルの計算としてはRxyyxという1つだけの値が分かればいいはずです
なので
>原点ならRxxxx(0,0)=Ryyyy(0,0)は当然同じ値になる
という部分のありがたみがどこにあるのかがわかりません
0400132人目の素数さん
垢版 |
2021/02/18(木) 21:39:42.37ID:F7gcKB9O
>>399
やっぱり微分した後でしか代入できないですよね
>>396で…と省略しましたが
そこの微分をやって接続のとこに上で計算した式を代入すると
その段階でほとんど項が消えてしまって
…の部分はその後はほぼ計算量が0なのです
なのでほぼ計算量が0のとこで値(0,0)を代入してもぜんぜん計算量変わらなくて
原点考えてもそのまま計算してもこの方法だと全然変わらないのです
0401132人目の素数さん
垢版 |
2021/02/18(木) 21:43:44.58ID:XZSUxj8X
>>400
もちろんそうなるわな
だって定数であるのは最初からそうなんだから
今回は整理する作業が大していらないケースだったんでしょ?
もちろん一般にはそんな都合のいいことは起こらないのでまともに関数計算して整理するのとある点での“微分係数”計算するのとでは作業量変わってくると思うのが感覚
本の著者もそう思ったんでしょ?
それに文句言ってどうすんの?
0402132人目の素数さん
垢版 |
2021/02/18(木) 22:04:38.81ID:F7gcKB9O
>>401
なるほど
つまりこの問題の具体的な解答に対するヒントというよりは
この手の問題を考える上での一般的な方針みたいなつもりで書いていたということでしょうか

なんか大いに誤解されてるようですが、決して文句を言ってるわけじゃなくて
ヒントと自分の計算とで実感がずれていたので
自分が非効率的な方法を取ってるのじゃないかと思って質問した次第です
(そもそも本当に正しく計算できているのかも自信がなく)
0403132人目の素数さん
垢版 |
2021/02/18(木) 22:13:41.87ID:XZSUxj8X
>>402
まぁその本の著者も実際どこまで自分でやって見たのかわからんし
やってみたらわかったと思うが計量テンソルの計算は2次元の場合ですら殺人的
オレが学部の時出されたレポートは球面の測地線の方程式出して大円が測地線になる事確かめよだったけど2次元でもクリストッフェル記号だけで原理的には8個出てくる
最初極座標でやろうとして挫折
よくよく考えると普通に(x,y,√(1-x^2,y^2))で計算すると作業量が半分になる事に気づいて解決した
何せ√が入ってて分母に回り込んで行くので鬱陶しい事この上ない
しかし測地線なので“原点だけ計算してなんとかする”などという事はできるはずもない問題
だからまぁ直感的に原点だけに絞れるなら楽やんと思った
まぁそうでもなかったんかも知らんがそんなもんやってみんとわからん
せっかくそこまでやったんなら測地線もやってみるといい
ピタッと0になるとすげぇ感動した覚えがある
0404132人目の素数さん
垢版 |
2021/02/18(木) 22:22:23.13ID:AS3mLgYp
>>402

> なんか大いに誤解されてるようですが

質問する側がこういう口の聞き方をするのは駄目
0405132人目の素数さん
垢版 |
2021/02/18(木) 23:27:08.40ID:F7gcKB9O
>>403
そうですね確かに計算は非常に重かったです
球面や双曲平面の測地線を求めるのも対称性をうまく使って簡単な場合に絞って〜
という議論は読んでる本で読みましたがそうせずに計算だけでやるとなると
確かに非常にしんどそうですね…訓練だと思って試してみます

長いことお付き合いいただいて本当にありがとうございました
0406132人目の素数さん
垢版 |
2021/02/19(金) 00:17:03.23ID:HTY2MKnL
>>403
あるある!
n次元球面の曲率計算したことあるけど、直交座標に心射投影してやると簡単だった
0407132人目の素数さん
垢版 |
2021/02/20(土) 09:35:56.73ID:23wkqJNE
ちなみに定曲率である事がわかってるならガウスボネでも曲率計算できるな
上半平面モデル
ds^2=1/(y^2)(dx^2+dy^2)
でvolume formは(1/y^2)dxdy
領域はD:-1/2≦x≦1/2、y≧√(1-x^2)にとると内角は2つあって共にπ/3
曲率をKとするとガウスボネより
∫[D]K (1/y^2)dxdy = π/3 + π/3 - π
(π/3)K = -π/3
∴K = -1
0408132人目の素数さん
垢版 |
2021/02/20(土) 19:53:19.45ID:Bd4w6+jc
ガウス-ボンネの定理かー名前しか覚えてねーや
上を見ると積分と曲率の関係だったんか
0410132人目の素数さん
垢版 |
2021/02/21(日) 18:47:07.29ID:RTZ9nhZg
線形空間の公理は通常8つあると思います。

零ベクトルの存在を保証する公理を除いた7つの公理から零ベクトルの存在を保証する公理を導けるでしょうか?
0412132人目の素数さん
垢版 |
2021/02/21(日) 20:14:28.03ID:uRwNF1k4
wikiの公理なら無理じゃない
0の存在のところを抜いたら V={ } としても問題ないし
0413132人目の素数さん
垢版 |
2021/02/21(日) 20:49:22.42ID:RTZ9nhZg
>>412
ありがとうございました。

>>411
Vを空でない集合とし、Wikipediaのベクトル空間の8つの公理のうち、「加法単位元の存在」を除いた7つの公理から「加法単位元の存在」が導けるかどうかが知りたいです。
0415132人目の素数さん
垢版 |
2021/02/21(日) 20:52:12.82ID:bisAjwLZ
0・vがベクトル空間の零元になるから、零元の存在は元の存在を言っているのと同じだな
>>412さんが正しいな
0416132人目の素数さん
垢版 |
2021/02/21(日) 20:55:31.35ID:RTZ9nhZg
>>414
間違えました。ありがとうございます。

Vを空でない集合とし、Wikipediaのベクトル空間の8つの公理のうち「加法単位元の存在」の公理と「加法逆元の存在」の公理を除き、
「任意のベクトル u, v ∈ V に対し、 u + x = v を満たすような V の元 x が存在する。」を付け加えた7つの公理から
「加法単位元の存在」の公理は導けるかどうかが知りたいです。
0417132人目の素数さん
垢版 |
2021/02/21(日) 20:59:47.54ID:bisAjwLZ
>>416
「任意のベクトル u, v ∈ V に対し、 u + x = v を満たすような V の元 x が存在する。」も要らないね
0体でなければ体には1も-1もあるから
0418132人目の素数さん
垢版 |
2021/02/21(日) 21:04:02.58ID:RTZ9nhZg
実はある本でのベクトル空間の定義が以下だったので質問しました:

Vを空でない集合とし、Wikipediaのベクトル空間の8つの公理のうち「加法単位元の存在」の公理と「加法逆元の存在」の公理を除き、
「任意のベクトル u, v ∈ V に対し、 u + x = v を満たすような V の元 x が存在する。」を付け加えた7つの公理を満たす集合Vを
ベクトル空間という。(スカラーは実数です。)

そして、その本には、「u + x = v を満たすような V の元 x を v - u と書く」とも書かれています。

x を v - u と書くということは、まず、そのような x の一意性を言わなければ駄目ですが、それについても何も書かれていません。

さらに、「任意の v ∈ V に対して、 v - v を 0 と書き、零元と呼ぶ」と書かれています。

任意の u, v ∈ V に対して、 u - u = v - v であることを証明しなければならないはずですが、それについても何も書かれていません。
0419132人目の素数さん
垢版 |
2021/02/21(日) 21:06:01.99ID:bisAjwLZ
ぼかさずに書名を明示して
0420132人目の素数さん
垢版 |
2021/02/21(日) 21:09:04.60ID:RTZ9nhZg
>>419
金谷健一著『これなら分かる応用数学教室 - 最小二乗法からウェーブレトまで』(共立出版)です。
0422132人目の素数さん
垢版 |
2021/02/22(月) 00:30:04.97ID:8uyyaY8k
>>418
ベクトル空間の公理の話じゃなくて群の公理の話をしているように見える。
0424132人目の素数さん
垢版 |
2021/02/22(月) 12:40:26.16ID:ZA1BxG4s
>>423
u, v の逆元が存在することを証明するには、まず零元が存在することをいう必要があります。
零元の存在はどうやって示すのでしょうか?
0425132人目の素数さん
垢版 |
2021/02/22(月) 12:44:44.64ID:ZA1BxG4s
あ、 u - u は、 u + x = u を満たすような元 x でしたね。

いずれにしても零元の存在が言えないと証明できないと思います。
0426132人目の素数さん
垢版 |
2021/02/22(月) 13:15:07.54ID:+MFi2cAF
0・vが零元だよ
0427132人目の素数さん
垢版 |
2021/02/22(月) 13:17:59.89ID:ZA1BxG4s
任意の u, v ∈ V に対して、

u + 0*v = u

はどうやって示すのでしょうか?
0429132人目の素数さん
垢版 |
2021/02/22(月) 14:05:48.41ID:F7yd8XIG
松坂くんに聞きたいんだけど、得意な分野は何かあるの?
他の人と比べてできるという意味ではなく、あくまで自分の中で比較的得意だと思う分野は?
0430132人目の素数さん
垢版 |
2021/02/22(月) 15:00:56.64ID:ZA1BxG4s
>>428

ありがとうございます。

u, v を V の任意の元とする。

v + 0*v = 1*v + 0*v = (1 + 0)*v = 1*v = v

∴ 0*v = v - v

とできそうだなと一瞬思いましたが、

u + x = v を満たすような V の元 x が一意的に存在することをまず示さなければならないはずです。

それはどうやって示すのでしょうか?
0431132人目の素数さん
垢版 |
2021/02/22(月) 15:08:25.25ID:Hwp3wiaJ
自分で考えろ
バカ
0432132人目の素数さん
垢版 |
2021/02/22(月) 15:20:20.48ID:ZA1BxG4s
やはり、零元の存在は示せませんね。
0433132人目の素数さん
垢版 |
2021/02/22(月) 15:22:33.87ID:ZA1BxG4s
Vを空でない集合とし、Wikipediaのベクトル空間の8つの公理のうち「加法単位元の存在」の公理と「加法逆元の存在」の公理を除き、
「任意のベクトル u, v ∈ V に対し、 u + x = v を満たすような V の元 x が存在する。」を付け加えた7つの公理から

「任意のベクトル u, v ∈ V に対し、 u + x = v を満たすような V の元 x が一意的に存在する。」は証明できないと予想します。

反例をお願いします。
0434132人目の素数さん
垢版 |
2021/02/22(月) 15:57:03.56ID:ZA1BxG4s
「任意のベクトル u, v ∈ V に対し、 u + x = v を満たすような V の元 x が一意的に存在する。」が仮に証明できたとすると、

u, v を V の任意の元としたとき、

(u + v) + 0*u = (u + 0*u) + v = (1*u + 0*u) + v = (1 + 0)*u + v = 1*u + v = u + v
(u + v) + 0*v = u + (v + 0*v) = u + (1*v + 0*v) = u + (1 + 0)*v = u + 1*v = u + v

(u + v) + x = u + v の解 x は一意的に存在するから、 0*u = 0*v

----------------------------------------------------------------------

z を V の任意の元とする。

u を V の任意の元とする。

u + 0*z = u + 0*u = 1*u + 0*u = (1 + 0)*u = 1*u = u だから、 0*z は零元である。
0435132人目の素数さん
垢版 |
2021/02/22(月) 15:59:27.03ID:ZA1BxG4s
そして、実際には、「任意のベクトル u, v ∈ V に対し、 u + x = v を満たすような V の元 x が一意的に存在する。」は成り立たないので、

>>420
金谷健一さんの公理系はベクトル空間の公理系とは異なる。


というのが本当のところではないでしょうか?
0437132人目の素数さん
垢版 |
2021/02/22(月) 16:39:07.91ID:Q5jYCxr3
てか正解はどっちなんやろね?
実際単位元の存在が証明できるのか、持たない反例が存在するのか
まぁ気にはなるけどどうでもいい気もする
0438132人目の素数さん
垢版 |
2021/02/22(月) 17:22:24.35ID:0e+8iyNk
あれ?
そんなに難しくないやん?
任意にu,vをとるときv=u+xとなるxをとれば
v+0u=u+x+0u=u+x=v
vを0vに置き換えて0u+0v=0v
コレが任意のu,vについて言えるから0u=0v (∀u,v)
悩むとこないやん
0439132人目の素数さん
垢版 |
2021/02/22(月) 19:45:12.36ID:ZA1BxG4s
>>438
ありがとうございます。

零元の存在が言えたので、零元の一意性、 v = u + x の x の一意性も言えますね。
0440132人目の素数さん
垢版 |
2021/02/22(月) 19:46:42.03ID:ZA1BxG4s
意外にも金谷健一さんは間違っていなかったんですね。

一意性を証明せずに、 u + x = v を満たす x を v - u と書くというのは問題がありますが。
0441132人目の素数さん
垢版 |
2021/02/22(月) 20:34:06.56ID:Q5jYCxr3
てかそれが間違ってたとしてなんなん?
教科書の間違い探すの趣味なん?
悪趣味やで?
0442132人目の素数さん
垢版 |
2021/02/22(月) 20:45:27.64ID:Hwp3wiaJ
キチガイの相手をするな
0443132人目の素数さん
垢版 |
2021/02/22(月) 21:51:00.70ID:B7LQrn1c
>>418
存在と一意性を合わせて公理にしてるんでしょ
0444132人目の素数さん
垢版 |
2021/02/22(月) 21:51:31.03ID:B7LQrn1c
>>420
応用数学なら仕方ない
0447132人目の素数さん
垢版 |
2021/02/23(火) 22:46:30.92ID:zonNd7Qo
今更だけど要は二項演算×が
∀g,h ∃x,y gx=h, yg=h
∀g,h,k (gh)k=g(hk)
を満たすなら群になるんだな
ベクトル空間がどうたら関係ない
0448132人目の素数さん
垢版 |
2021/02/23(火) 23:08:55.78ID:TUB/p8pJ
∃!が要るかどうかの話だね
考える必要はある
0450132人目の素数さん
垢版 |
2021/02/23(火) 23:12:11.06ID:pBM0fkVw
ge=g
he=yge=yg=h
e'g=g
e'h=e'gx=gx=h
e'=e'e=e
gg'=e
g''g=e
g''=g''e=g''gg'=eg'=g'
0451132人目の素数さん
垢版 |
2021/02/23(火) 23:16:23.38ID:pBM0fkVw
>>448不要
>>449いい
0452132人目の素数さん
垢版 |
2021/02/23(火) 23:31:22.61ID:sWb9Yl6m
>>451
不要なことはない
唯一成を示す必要はある
0453132人目の素数さん
垢版 |
2021/02/24(水) 00:04:54.46ID:7sFMZBYW
>>452
>>450
および通常の一意性証明
0454132人目の素数さん
垢版 |
2021/02/24(水) 00:07:10.33ID:7sFMZBYW
>>453
>通常の一意性証明
ge=eg=g
ge'=e'g=g
e=ee'=e'
gg'=g'g=e
gg''=g''g=e
g'=g'e=g'gg''=eg''=g''
0455132人目の素数さん
垢版 |
2021/02/24(水) 00:15:55.23ID:u9u1THcY
だから∃から∃!が導かれることは示しておく必要はあるよね?って話
0457132人目の素数さん
垢版 |
2021/02/24(水) 01:00:27.58ID:DCUn/KgJ
公理として十分か(それから通常の群の定義を満たすことが示せるか)どうかの話じゃないの?
0458132人目の素数さん
垢版 |
2021/02/24(水) 01:13:42.78ID:7sFMZBYW
>>448
>∃!が要るかどうか
不要
>>452
>唯一成を示す必要はある
必要
0459132人目の素数さん
垢版 |
2021/02/24(水) 01:26:36.27ID:u9u1THcY
ID:7sFMZBYW
この人のレス、文章が書いてなくて気持ち悪い
0461132人目の素数さん
垢版 |
2021/02/24(水) 13:32:41.39ID:YBKQ0wNi
おまいら
レベルの低い話は盛り上がるな
0462132人目の素数さん
垢版 |
2021/02/24(水) 14:18:58.20ID:e3rk6lLz
>>460
よく言われてるけど嘘っぱちだよね。
0463132人目の素数さん
垢版 |
2021/02/24(水) 14:36:43.36ID:k01yBJ5B
量化子を全く含まない言語体系で一階の述語論理と同じ論理体系が構成できるの?
0465132人目の素数さん
垢版 |
2021/02/24(水) 17:33:46.26ID:k01yBJ5B
いや、普通の述語論理では量化子は必須だし、量化子なしで一階述語論理と同等な議論ができる論理体系なんか見たことないから聞いてるんだけど?
その論理体系では量化子なしで証明かいとけば自動的に量化子が決まるようなクソ便利な理論があるん?
0466132人目の素数さん
垢版 |
2021/02/24(水) 18:00:04.28ID:F48bgLw1
量化子を書かないだけなら
(∀x)P(x) を x→P(x)
(∃x)P(x) を x∧P(x)
とすればスコープの記述をどうするか以外だいたいよくて、
スコープの制限を外せば動的論理になるんだと思った
0467132人目の素数さん
垢版 |
2021/02/26(金) 08:39:05.25ID:64AF3idO
>>466
>(∀x)P(x) を x→P(x)
>(∃x)P(x) を x∧P(x)
>とすれば

xて、いつ論理式になったん?
0468132人目の素数さん
垢版 |
2021/02/26(金) 09:38:39.27ID:WUBPqun+
>>467
論理式にするってことよ
0469132人目の素数さん
垢版 |
2021/02/26(金) 11:18:05.53ID:64AF3idO
>>468
どうやって?

※勝手に「論理式だ」と宣言するのはダメ
0471132人目の素数さん
垢版 |
2021/02/26(金) 11:58:37.33ID:J3QOsf/v
>>470
1行目は妥当と思えるけど
2行目は意味がよくわからない
なぜだろう、ただ論理式を変換しただけなのにね
0473132人目の素数さん
垢版 |
2021/02/26(金) 14:57:25.05ID:/FnssZzf
>>471
1行目の意味がわかれば2行目もわかろうに
∀がand
∃がorの拡張だとは知ってるんだろ?
0475132人目の素数さん
垢版 |
2021/02/26(金) 15:06:50.79ID:J3QOsf/v
>>470 の違和感の原因
>(∀x∈A)P(x) を x∈A→P(x)
というのは
(∀x∈A)P(x) を ∀x(x∈A→P(x))
のことを指している。これは正しい

>(∃x∈A)P(x) を (x∈A)∧P(x)
というのは
(∃x∈A)P(x) を ∀x((x∈A)∧P(x))
のことではないから1行目と同じように考えることはできない

結局のところ限定子を明示しないとおかしなことになる
0478132人目の素数さん
垢版 |
2021/02/26(金) 16:51:27.76ID:sjg7fm2z
>>469 勝手に「論理式だ」と宣言してから意味論を与える

>>470
普通の述語論理だと変数の動ける範囲である対象領域Uが解釈のときに与えられるので
x だけで x∈U を意味すると思ってください
x だけが変に見えるなら [x] としてもいい

∀x(P(x)→Q) ≡ (∃xP(x))→Q

[x]→(P(x)→Q) ≡ ([x]∧P(x))→Q
になって
R→(P→Q) ≡ (R∧P)→Q
という命題論理と同じ形になったりする
0479132人目の素数さん
垢版 |
2021/02/26(金) 19:44:45.38ID:J3QOsf/v
>>476
いやそれはわかるが

x∈A∧P(x) と書かれているものを見て∃x(x∈A∧P(x)) と同じだと言われても違和感しかない
0480132人目の素数さん
垢版 |
2021/02/26(金) 20:32:16.39ID:QZNYt4/a
>>479
全然?
>>474は?
0481132人目の素数さん
垢版 |
2021/02/26(金) 20:34:51.28ID:QZNYt4/a
多分君は
P(x)は∀xP(x)の略記だとしか認識してないんだろうな
0486132人目の素数さん
垢版 |
2021/02/26(金) 23:20:02.23ID:Bmqpqbe8
>>485
しょむ無いネ君
0487132人目の素数さん
垢版 |
2021/02/26(金) 23:21:40.66ID:Bmqpqbe8
>>484
>ID:64AF3idO
理解もしようとしない人だね
0488132人目の素数さん
垢版 |
2021/02/26(金) 23:26:03.11ID:Bmqpqbe8
大学学部レベルというか
問題を解くだけしかしてなくて
数学に向き合うということをしたことない人たちが居るようね
それって数学の本質を実戦してないんだけどね
0489132人目の素数さん
垢版 |
2021/02/27(土) 00:30:15.48ID:/RZPp9I7
本質に向き合った(向き合おうとした)結果ただ教科書なぞるだけしかせずに具体的な問題一切解けない人間になったら目も当てられない、そしてそのような人間は割と多くいる
それに比べたらまだ問題解ける分マシだよ
0490132人目の素数さん
垢版 |
2021/02/27(土) 00:38:33.19ID:wMfESIt3
んじゃあ
卒研じゃ無くて卒試で済ませるような学問もどきに堕するんだな
0492132人目の素数さん
垢版 |
2021/02/27(土) 11:46:21.04ID:4igMNdbq
本質は知らんが議論は論理的に正しいほうが正しくないよりマシ
ただ日本人のクリティカルシンキング力は低いから、どうしても「問題が解けるほうがいい」という「プラクティカルな主張」に傾く確率が高い
0493132人目の素数さん
垢版 |
2021/02/27(土) 12:11:00.93ID:mVDpi0+T
以下の命題の以下の証明は合っていますか?



{u_n} は集合 D 上の函数列とする。

|u_n(x)| ≦ a_n (x ∈ D), Σa_n < ∞

となる正数 a_n が存在すれば、級数 Σu_n(x) は一様収束する。

何となれば、

|Σ_{n=M}^{N} u_n(x)| ≦ Σ_{n=M}^{N} a_n

M, N → ∞ のとき右辺は x に無関係に 0 に収束する。
0495132人目の素数さん
垢版 |
2021/02/28(日) 00:40:45.87ID:rfu+VzsY
位相空間Xの各点において連結な近傍をもつが局所連結でないものの例ってどんなものがありますか?

例えばコンパクト性を局所的に考えるときはそのような近傍が存在することとして局所コンパクト性が定義されていたのに対して、連結性では基本近傍系で定義されているので上のような例を教えて欲しいです
0498132人目の素数さん
垢版 |
2021/02/28(日) 04:19:22.51ID:vAztcVF2
連結は2点への連続写像を作ろうとすると上手くいかない、何かが邪魔をする、ということだから、(2点以上の)部分集合の方が非連結になりやすい。
0499132人目の素数さん
垢版 |
2021/03/01(月) 09:15:12.13ID:xZz6CGzJ
b n が次で与えられている
b0 = 0, b1 = 1
bn = 2b(n-1) + 2 b(n-2)
vを二進付値とする時
v(bn) = n + v( [ n/2 ] )
を示せ
0500◆QZaw55cn4c
垢版 |
2021/03/01(月) 20:50:03.07ID:mVM2M1hV
>>447
いろいろ考えてみたんですが、ガバガバユルユルな定義ですね
群の公理としては、二つ目の結合則は必要として、一つ目の式はちょっと贅肉が多いのでは?
群の公理は、結合則の他には ∃e∀g ge = g, かつ ∀g∃h gh = e で十分かと
0501132人目の素数さん
垢版 |
2021/03/01(月) 21:03:03.43ID:b6EQ2gzc
アーベル群ならそれでいいよ
0502132人目の素数さん
垢版 |
2021/03/02(火) 00:25:49.52ID:s/RK2FsU
他の代数系との関係もあるから
共通のは独立に書いた方がわかりやすい
0503132人目の素数さん
垢版 |
2021/03/02(火) 07:57:38.62ID:RHwb3U7G
>>500
そのhでhg=eを示せ
0504132人目の素数さん
垢版 |
2021/03/02(火) 07:58:41.72ID:RHwb3U7G
eg=g示すのが先か
0505◆QZaw55cn4c
垢版 |
2021/03/02(火) 20:50:17.83ID:9lTkOgWK
>>501
アーベル群=可換群でなくても、群の公理は@結合則とA∃e∀g ge = g, かつ ∀g∃h gh = e で十分であることを、これから示しましょう
以下、量化子を省略します

>>504
gh = e ‥‥@
ge = g より @の g に ge を代入して
geh = e
すなわち g(eh) = e ‥‥A
Aと gh = e を辺々比べて eh = h
すなわち eg = g ‥‥B
これが証明すべきことであった.

>>501
gh = e を ge = g に代入するが、gh = e より gh を左辺の g に、 e を右辺の g に代入してよく
ghe = e ‥‥C
Cの両辺に g を右からかけて
gheg = eg
Bを使って
ghg = g ‥‥D
Dと ge = g を辺々比べて hg = e
これが証明すべきことであった.
0506132人目の素数さん
垢版 |
2021/03/02(火) 20:54:03.77ID:qHnIfoFs
>>505
>すなわち g(eh) = e ‥‥A
>Aと gh = e を辺々比べて eh = h
これなんで?
0507132人目の素数さん
垢版 |
2021/03/02(火) 21:03:00.58ID:qHnIfoFs
>>505
>∃e∀g ge = g, かつ ∀g∃h gh = e
このeについて
∀g (ge=g∧ge'=g) → e=e'
を示してくれますか
端的には
>∀g∃h gh = e
のeが指すものを確定できますか?あるいは公理は
∃! e∀g ge = g
ですか?
0508132人目の素数さん
垢版 |
2021/03/02(火) 21:04:07.46ID:qHnIfoFs
>>505
>Aと gh = e を辺々比べて eh = h
こちらも
∀g∃! h gh = e
ということですか?
0509132人目の素数さん
垢版 |
2021/03/02(火) 21:08:29.17ID:qHnIfoFs
>>505
>∃e∀g ge = g, かつ ∀g∃h gh = e
∃! e∀g ge = g, かつ ∀g∃! h gh = e
なら問題ないということですか?
!は取れませんか?
0510◆QZaw55cn4c
垢版 |
2021/03/02(火) 21:12:42.21ID:9lTkOgWK
>>506
g(eh) =e であり、かつ、
g h  =e ならば
この g は任意にとることができるのだから
eh = h なのでは?

>>507
ge = g かつ gg^-1 = e から ge = eg = e を導きだす >>505 の推論を認めてくださるのならば次のように証明できます
今、ge = eg = e ‥‥@
かつ e' も単位元だから ge' = e'g = e' ‥‥A
であったとする、@に e' → g を代入して e'e = ee' = e ‥‥B
Aに e → g を代入して ee' = e'e = e ‥‥C
BCより e = e' すなわち単位元は、存在すればただ一つ、QED
0511132人目の素数さん
垢版 |
2021/03/02(火) 21:13:14.72ID:qHnIfoFs
∃!は∃に比べてかなりキツイ条件なので
!が外れないなら>>447の方がよりよいかな
あと
!が外れたとしても
eの存在を規定するよりは>>447の方が気持ちいいかな
0512132人目の素数さん
垢版 |
2021/03/02(火) 21:14:59.79ID:qHnIfoFs
>>510
>この g は任意にとることができるのだから
>eh = h なのでは?
任意に取れたら何でeh=h?
0513132人目の素数さん
垢版 |
2021/03/02(火) 21:17:29.65ID:qHnIfoFs
>>510
>ge = eg = e を導きだす >>505 の推論
ge=eg=gね?それが分からない
もちろん
ge=eg=gなら単位元の一意性は通常の証明です
0514◆QZaw55cn4c
垢版 |
2021/03/02(火) 21:25:17.71ID:9lTkOgWK
>>512
>g(eh) =e であり、かつ、
>g h  =e ならば
>eh = h

ここでの演算子を * とします
任意の元 g に対して
g * X = e
g * Y = e
だったら X = Y ‥‥うーん、確かに逆元は今の段階では単位元も逆元も一つとは限りませんね‥‥


考え直します‥‥
0515132人目の素数さん
垢版 |
2021/03/03(水) 00:24:56.08ID:vCAyKp+X
長い直線とおんなじ感じで長い点線てあるんでしょうか?
任意の二つの点は有限の点列でつながっているけど、全体は整数と順序同型ではない、みたいな
0516132人目の素数さん
垢版 |
2021/03/03(水) 01:06:45.11ID:RlseFId7
すみません方程式についての質問です
数学を専攻したことはありません(物理学科) ググって知ったことについてです。

(1)5次方程式の数値解法があるらしいのだから、一般の5次方程式の解の小数表示を有限回の四則演算で1桁ずつ確定させることもできるんですよね?

(2)「一般の3次方程式の解を有限回の実数の冪根の計算と四則演算で求めることはできない」というのを今知ったのですが
では数値を求めるには(どういうのが効率が良いか分かりませんが)逆三角関数値や三角関数値をマクローリン展開を使って計算していくなどする必要がありますよね?
(そんなことを言ったら実数の冪根もそもそも四則演算も1桁ずつ数値を確定させていくことしかできなくて似たようなものだが)

(3)という事は3次方程式の解の公式について、複素数の冪根を認めて「代数的解法」と称するのは、「実数の冪根を認めるなら対称的に複素数の冪根を認める方が妥当」という専ら形式的なものでしょうか?
それとも、非代数的な解(?)と違って、解の累乗の計算が有機的にできたりと代数の枠組みで有意義な取り決めだとされるんでしょうか?


(3)に関しては、(「解法があるかないか」のような)字面に惑わされて起こる感覚的な思い込みを捨てて、単に論理的に言えることだけを理解しようとすれば意味のない質問と化す類の愚問かもしれませんが
もしかしたら「虚数にはこういう有機的な意味があるんだ!」と言われる文脈のように、多数で共有される感覚としての答えが存在する可能性もあると思って聞きました。
0518132人目の素数さん
垢版 |
2021/03/03(水) 01:19:55.46ID:CGS3YUU8
>>516
まぁこの話は昔のヨーロッパでやってた方程式の解の公式の懸賞問題の取り決めとかも関わってるから純数学的にどうこうの問題も絡んでて微妙
“複素数のルート”を三次方程式の解の公式で使うのがありかなしかは「ありにするのが正解」とか「そんなのはイインチキ、ダメ」とか一概に言えるもんでもなく微妙
しかし確実に言えるのは
「五次以上の方程式だと複素数のルートを認めたとしても解けない」
「四次、三次は複素数のルートを認めれば必ず解ける、ルートは実数のルートとiしか認めないなら作図可能性と可解性が同値になる」
0519132人目の素数さん
垢版 |
2021/03/03(水) 01:24:16.42ID:RlseFId7
>>518
ありがとうございます
全体的なニュアンスは伝わってきますし具体的な部分でも作図可能性〜のところだけ分かりませんが2次方程式以下のことであるのは分かります
一応>>516の(1)と(2)にもYes,Noでいいので答えてくれないでしょうか。両方yesですよね?
0520132人目の素数さん
垢版 |
2021/03/03(水) 02:07:09.29ID:RlseFId7
にわか知識でなんJにスレ立てて楽しんできた

【クイズ😆】一般のn次方程式で、解を有限回の四則演算で求められるのは○次以下である
https://swallow.5ch.net/test/read.cgi/livejupiter/1614702665/

こういう感じで答えが1次→2次→(3次→)…と変わっていってしかも7次とか大きい数まで問題が作れたら面白い気がする😆
5次方程式の解を構成できる超冪根とかいう奴、6次方程式に通用する同様の概念は作れないのかな?(ほぼ独り言😆)
0522132人目の素数さん
垢版 |
2021/03/03(水) 10:05:14.21ID:RlseFId7
>>521
ありがとうございます!
0524132人目の素数さん
垢版 |
2021/03/03(水) 12:59:07.74ID:zG/lqqO3
正解は○=1だよな
0525132人目の素数さん
垢版 |
2021/03/03(水) 13:13:14.76ID:nTI+qAnq
と思うじゃん?
「係数から」有限回の四則演算〜とは書かれていない、これは罠だったんだ
0529132人目の素数さん
垢版 |
2021/03/03(水) 17:11:15.29ID:RlseFId7
>>528
その解自体はどう求めるの?ってなったらどうやっても一般の(任意の)方程式から解を求めてるとは言わんよね
0530132人目の素数さん
垢版 |
2021/03/03(水) 22:47:30.05ID:c6oCII/v
>>515
なに言ってんのか知れン
0532132人目の素数さん
垢版 |
2021/03/03(水) 22:49:04.12ID:c6oCII/v
>>516
多分君は数値を特定するということとは10進法の桁をひとつひとつ決めていくモノだと思っているのだろう
0534132人目の素数さん
垢版 |
2021/03/03(水) 23:26:16.39ID:c6oCII/v
>>533
やはりね
小数表記を求めることを以て解法とするのであれば何でも解ける
代数的解法はべき乗根と四則を(小数表記では無く)確定した値を表すモノとして使って一般の方程式の解を表すということであって
べき乗根が実数のべき乗根に限るとはしないのが通常の解釈
小数表記なら実数であれ複素数であれやることは同じだし
もちろん実数のべき乗に限ればどうなるかというコトを考えても良い
結論的には>>518で尽きているだろう
0538132人目の素数さん
垢版 |
2021/03/04(木) 07:36:13.72ID:DY7dzUfk
>>536
アホーか
>>516でそうではないかと尋ねているからそうだと言った
自分が尋ねていてそうだよと答えられて当たり前じゃんと返すとはな
もちろんそれは>>518で尽きていることだからすでに答えられている
0539132人目の素数さん
垢版 |
2021/03/04(木) 08:05:38.61ID:zRYLg81o
テンソルって概念が分からん

スカラー→ベクトル→行列 という拡張の流れは分かるんだが
テンソルって出てくるときはたいてい行列のような形で表記されている
行列は「2次のテンソル」なんて話を聞いたことがある

じゃあテンソルってのは
スカラー(0次元)→ベクトル(1次元)→行列(2次元)→?(3次元)→??(4次元)→…
という拡張の流れの中にあるものを総称したものなのか?
0540132人目の素数さん
垢版 |
2021/03/04(木) 08:21:05.61ID:4Nhg0Q4J
多重線形を線形したものがテンソルです
物理に出てくるテンソルは数学のそれの成分表示したものです
2次のテンソルを成分表示すれば添字が2個だから行列の形で書いてるだけで、もちろん行列そのものではないです
演算(積)が行列のものとは異なります
0541132人目の素数さん
垢版 |
2021/03/04(木) 09:17:24.07ID:zRYLg81o
「物理で出てくるテンソル」「数学のそれ」
って言い方も分からん
行列なら物理で出てこようが数学で出てこようが数値が2次元に並んだもので演算は共通だけどテンソルだと違うの?
0542132人目の素数さん
垢版 |
2021/03/04(木) 13:42:03.67ID:bKHikyXz
同じさ
数学では色んな見方をするだけだ
物理でも用途によって使うが
0544132人目の素数さん
垢版 |
2021/03/04(木) 15:17:48.77ID:CUIXoIWf
>>541
数学のテンソルはg_ab dx^a dx^b

物理のテンソルは g_ab

本来のテンソルの一部分しか見てないので、わけのわからないこと言い始めるわけですね

多様体論の座標変換の話で終わるだけなのに、変換性がどーたらこーたらを満たす行列だーとか言い始めるわけです
0545132人目の素数さん
垢版 |
2021/03/04(木) 15:42:46.05ID:T6XhnBRF
物理系が対称性を持つとき、物理量もその対称性を持つ
多くの場合、その対称性は行列で書かれたリー群やリー環で表現される(表現Vを1つ固定する)
物理系を変換したとき、変換しないものがスカラー(自明表現)、Vとして変換するのがベクトル、Vのテンソル積表現として変換するのがテンソル
時空に対する対称性の場合、符号表現がテンソルされてるものは「擬」という接頭語がついたり、元の群の被覆群の表現に属するものはスピノルと呼ばれたりする
0548132人目の素数さん
垢版 |
2021/03/04(木) 19:28:07.95ID:zRYLg81o
テンソルがちんぷんかんぷんだから、テンソルを「テンソル積」で説明されてもわからん
テンソルってのはあくまで数学における概念で、物理学はそれを利用してるだけ、ってとこはいいんだよな?
それとも量子力学の変な記号みたいに数学とは別に物理学で独自に発展しちゃってる部分もあるわけ?
そう言えばテンソルって言葉は物理学でしか聞いたことがない
0549132人目の素数さん
垢版 |
2021/03/04(木) 19:36:04.48ID:zRYLg81o
量子力学の変な記号ってのは「ディラックのブラケット記法」っていう、> とか < とか|使うやつのことね
0551132人目の素数さん
垢版 |
2021/03/04(木) 20:06:23.73ID:6wZGTihV
>>548
テンソル代数をイデアルで割って個別の具体的なナンチャラ代数を定義していく。
0552132人目の素数さん
垢版 |
2021/03/04(木) 20:29:55.23ID:T6XhnBRF
>>548
上にも書いたように考えてる物理系の対称性とそれの自然な表現Vを1つ固定して考えている
Vのテンソル積表現というのは数学的にはっきりしてて、それはV⊗V⊗…⊗V(ただしいくつかはVの反傾表現V'とすることもある)として作られていて
いわゆるテンソルとはこの線形空間の元(もしくはそれを基底で展開したときの係数)
0553132人目の素数さん
垢版 |
2021/03/04(木) 21:07:56.59ID:4Nhg0Q4J
>>541
例えば、ベクトル解析における作用素div,rotとかも実はテンソル(正確にはある次数の微分形式からなる線形空間上の外微分)
ただし物理では普通微分作用素を並べた「ベクトル」として扱っている、これは数学でいうと基底を固定して成分表示したものになっている

根本のところの「もの」は同じだけど、物理では計算面から見て実用しやすい形で定義している(と思う)
0554132人目の素数さん
垢版 |
2021/03/04(木) 22:20:40.61ID:bKHikyXz
>>549
ブラケットは数学での2次形式と同じだけど
転置記号もいらないし複雑な式も入れるし便利に魔改造されてるな
0555132人目の素数さん
垢版 |
2021/03/04(木) 22:23:14.68ID:bKHikyXz
>>548
テンソルって経済でも使えそうだがなー
言語学でも多重相関を調べるのに使わんかしらん
0556132人目の素数さん
垢版 |
2021/03/04(木) 23:48:01.75ID:VEVIDWr+
>>554
>転置記号もいらないし
代わりに右と左がいるやん
0557132人目の素数さん
垢版 |
2021/03/04(木) 23:53:19.78ID:VEVIDWr+
>>555
ディープラーニングで
0559132人目の素数さん
垢版 |
2021/03/05(金) 02:20:05.10ID:9AQyOZRj
>>545
ベクトルバンドル、ベクトル束
スピンバンドル、スピン束
なら言うけど

テンソルバンドル、テンソル束

言わないよなー。
0561132人目の素数さん
垢版 |
2021/03/05(金) 02:38:44.78ID:motBJKwN
>>559
それってカニやサソリやクモが節足動物だという話をしてるところに、蟹座や蠍座はあるけど蜘蛛座はないよなー、と言い出してるようなもの

>>560
テンソル積の普遍性を言いたいんだろうけど、ここで質問されてるテンソルとは話がずれてる
0565132人目の素数さん
垢版 |
2021/03/05(金) 03:14:46.48ID:motBJKwN
>>563
ベクトル束のテンソル積は取れるからね
でも、それも結局はベクトル束
本筋ではないことを言うのは混乱の元だからやめよう、という話
0567132人目の素数さん
垢版 |
2021/03/05(金) 08:03:53.48ID:SWWZv1v8
>>558
だから別に便利にってこと無いってこと
0573132人目の素数さん
垢版 |
2021/03/06(土) 12:59:32.14ID:PmxUoqJo
>>572
特性方程式p(x)求めて
x^5+9x^4+30x^3+36x^2+20x+9÷p(x)のあまりax^2+bx+c求めてaA^2+bA+cIを求める
0576132人目の素数さん
垢版 |
2021/03/06(土) 22:02:58.52ID:NFvwQDpA
数学のフィルトレーションだの自然なフィルターだのは
物理や工学のフィルターとは明らかにことなる。
ちなみに確率微分方程式の参考書で上の意味のフィルターを使う場合には
カルマンフィルタは絶対登場しない
明らかにフィルタの意味が違うから
位相にしてもなんで日本数学界の馬鹿たれどもはこういう紛らわしい訳語を選択するんだろ。
0577132人目の素数さん
垢版 |
2021/03/06(土) 22:26:50.31ID:nbSOpobn
それ日本の学会が決めてないと思う
0578132人目の素数さん
垢版 |
2021/03/06(土) 22:29:26.13ID:nbSOpobn
ultrafilterを検索すると濾過装置ばっかり出てくるよね
0581580
垢版 |
2021/03/07(日) 05:06:21.00ID:b7zzL7Gj
>>576じゃなく
>>577へのアンカー
0582132人目の素数さん
垢版 |
2021/03/07(日) 05:11:36.94ID:b7zzL7Gj
>>577
Filtrationは日本と関係ない話だったな。
情報の増大系をFilterというのは日本の話じゃない。

でもTopologyを位相と訳したのは日本の学会だろ
0583132人目の素数さん
垢版 |
2021/03/07(日) 08:06:39.76ID:ZsNGtVKl
本当かどうか知らんが、位相の訳語は「位置」と「形相」に由来しているらしい
ソースは加藤十吉『位相幾何学』
0584132人目の素数さん
垢版 |
2021/03/07(日) 08:26:11.68ID:ZsNGtVKl
>>580
数学ではCGPMやIUPACのような規格標準化のための組織は存在しないよ
どこどこの組織が「〇〇は〜とする」なんて定めるわけではないです
0585132人目の素数さん
垢版 |
2021/03/07(日) 10:15:38.83ID:UrORCwc2
文脈で分かるじゃん
0588132人目の素数さん
垢版 |
2021/03/07(日) 17:04:26.07ID:Ax93y6LC
趣味で数学をしてるアホ大学生です。

(X,B(X),μ)という測度空間を考えることにして
{fn}という関数列が
(1)fn≦Mを満たし、
(2)fnがfにL2で収束する

|f|≦M a.e.-μはなりたちますか?

δ>0として、
成り立たないと仮定すると、
∃A;μ可測集合,μ(A)>0,f|A≧M+δ

すると,
∫_X |fn-f|^2dμ
=∫_A |fn-f|^2dμ + ∫_A^c |fn-f|^2dμ
≧∫_A |fn-f|^2dμ
≧δ^2μ(A)
>0
なので矛盾

と考えました。
0589◆QZaw55cn4c
垢版 |
2021/03/07(日) 20:43:45.61ID:+cRDkVEs
>>513,512 (>>514 の解消)
>>500 という主張の証明を考え直しました

>>500
>群の公理は、結合則の他には ∃e∀g ge = g ‥‥@, かつ ∀g∃h gh = e ‥‥Aで「必要十分」(修正)

証明のツボ:Aの h に対してAを再適用すれば ∀h∃H hH = e ‥‥B

>>504
eg = ege (∵@)
= eghH (∵B)
= eeH (∵A)
= eH (∵@)
= ghH (∵A)
= ge (∵A・B)
= g (∵@)

>>503
hg = hge (∵@)
= hghH((∵B)
= heH(∵A)
= hH (∵A・B)
= e
以上が証明すべきことであった
0590132人目の素数さん
垢版 |
2021/03/07(日) 22:14:51.64ID:u7YtHJ1m
>>589
2⃣にはeの一意性が要るのでは?
具体的には
gh=eのeとhH=eのeが同じである理由を教えてください
0591132人目の素数さん
垢版 |
2021/03/07(日) 22:20:48.38ID:u7YtHJ1m
こうしたらどうでしょう?
∃e∀g∃h ge=g, gh=e
これならeとして複数考えられたとしてもそのうちの1つのeで>>589の考察が出来ますよ
0592◆QZaw55cn4c
垢版 |
2021/03/07(日) 22:29:38.18ID:+cRDkVEs
>>590
そういう読み方ではありません
∃e∀g ge = g ‥‥@,
で少なくとも一つの e が存在すれば、その e に対して
∀g∃h gh = e ‥‥A
が成立すると定義します。これらは定義・公理です、したがって
∀h∃H hH = e ‥‥B
はAのh の対してAを再適用しただけですから、Bの e は Aの e と同一の単位元です、すなわち
「gh=eのeとhH=eのeは同じ」

@Aが同時に成立すれば、ge = eg = g で e の一意性、続いて gh = hg = e から逆元の一意性を示せます
0594132人目の素数さん
垢版 |
2021/03/07(日) 22:50:26.59ID:u7YtHJ1m
>>592
>そういう読み方ではありません
それならばやはり>>591
0596◆QZaw55cn4c
垢版 |
2021/03/09(火) 16:04:09.61ID:+5fKxQbl
>>594
さらに考え直しました
∃e∀g ge = g
かつ
∃e(∀g ge = g → ∀g∃h gh = e)
あたりが私の気持ちに近いかもしれません…
0597132人目の素数さん
垢版 |
2021/03/09(火) 16:06:54.68ID:SKEI5bO2
松坂和夫著『解析入門上』の複素整級数のところに以下の記述があります。

C の部分集合 S で一様収束する複素連続関数列の極限関数が複素連続関数になるという命題の証明について、
R の区間 I で一様収束する実連続関数列の極限関数が実連続関数になるという命題の(この本での)証明を
そのまま用いるわけにはいかないということを言っています:


「さらに、一様収束する連続関数列の極限はまた連続である。(厳密にいえば、実変数の場合の9.1節の定理4は定理3に依拠しており、定理3の記述は
やや実変数に“局限”された形になっているから、証明には多少の補正を要しよう。しかしそれは容易であるから、ここではあらためて述べない。実際には
この定理は、後の距離空間の位相の章でみるように、もっと一般的な状況のもとに直接かつ簡単に証明することができる。)」


「定理3の記述はやや実変数に“局限”された形になっている」という箇所が何を言いたいのか分かりません。

定理3を見てみても実変数に“局限”などされていないと思います。

定理3で登場する x_0 は R の区間 I の任意の点ですので、かならず I の集積点になります。
一方、 z_0 を C の任意の空でない部分集合 S の任意の点とすると、 z_0 はかならずしも S の集積点にはなりません。(S の孤立点になる可能性があります。)

ですが、孤立点においては、関数はかならず連続ですから、証明に「多少の補正を要」するとは思いません。
0598132人目の素数さん
垢版 |
2021/03/10(水) 00:00:18.94ID:p8VP92mb
>>596
そもそも群の公理の同値な条件を云々しているので
どれが良いか(より弱いか)は個人の好み
ただし教育的な観点からの評価はあるか
自分は>>447
∀g,h ∃x,y gx=h, yg=h
か次点で>>591
∃e∀g∃h ge=g, gh=e
がいいかな
0599132人目の素数さん
垢版 |
2021/03/10(水) 06:57:24.90ID:X8F2vzLb
非可換のとき>>591で上手くいくとは思えない
0600132人目の素数さん
垢版 |
2021/03/10(水) 14:13:11.77ID:2VDr50wb
馬鹿が同じ場所を何年も回っているだけだ
相手にするな
0603132人目の素数さん
垢版 |
2021/03/10(水) 16:06:11.94ID:X8F2vzLb
eg = egeの論証が不十分
0604◆QZaw55cn4c
垢版 |
2021/03/10(水) 16:25:43.39ID:C23FnFRP
>>603
∵@

∃e∀g ge = g …@
g に eg を代入して
ege = eg
0605132人目の素数さん
垢版 |
2021/03/10(水) 17:31:33.78ID:X8F2vzLb
これは私の勘違いでした、すみません
0608132人目の素数さん
垢版 |
2021/03/11(木) 01:23:51.07ID:PJboiQw1
長い直線がパラコンパクトでないことはどのように証明できるのでしょうか?
0609132人目の素数さん
垢版 |
2021/03/11(木) 11:08:19.55ID:/hJkn62P
>>608
long linewikipediaの定義の第一の方法、すなわちord×[0,1)に辞書式順序入れたときの順序位相で定めたものとし、γを最初の非可算順序数(アレフ1)とすると(γ0)が可算な近傍基を持てない、すなわち第二可算ではない
0610132人目の素数さん
垢版 |
2021/03/11(木) 12:14:35.91ID:bxZbMBqv
>>609
ああなるほどパラコンパクトだと第二可算になることを使えばいいんですね
定義を直接示そうとばかりしてました
ありがとうございます
0612132人目の素数さん
垢版 |
2021/03/14(日) 08:40:26.67ID:NhPdW6zV
大学の専門とかじゃなくて、趣味で情報系数学ちょっとかじってるんですけど、

https://qiita.com/perrying/items/6b782a21e0b105ea875c

ここにある、
重みとバイアスの更新
の所で、
重みw[new,i]=w[i]-lr∂f(x)/∂w[i]
と表されるらしいのですが、

この説明は割愛されているのですが、なぜこのような式で表されるのか、感覚的でも良いので理解したいのですが・・・。

w[new]ってたぶんw[1]とかに対してw'[1]とかをさすんですよね?基本的な所ですが。
0613132人目の素数さん
垢版 |
2021/03/14(日) 08:44:08.87ID:NhPdW6zV
具体的な簡単な関数当てはめて実験していったら分かりますかね?
0614132人目の素数さん
垢版 |
2021/03/14(日) 10:43:15.73ID:6pTMOnGc
傾きが負なら重みを正の方向に動かしたい
傾きが正なら重みを負の方向に動かしたい
これだけならw_i^{new}=w_i - ∂f(x)/∂w_iで良いが、∂f(x)/∂w_iをそのまま使うと重みが動きすぎたりするので、lrをかけて調整する
0615132人目の素数さん
垢版 |
2021/03/14(日) 15:19:26.91ID:pDeYZQUi
おぉ!その論理は分かりました。
が、またよく理解していないがゆえ疑問が・・・

増加量がマイナスの場合について考えます。
f(x)は重みづけた和の事で、これが減っていっているなら、重みづけを増やして、
次の重みづけの和は増やそう、という事だと思うのですが、
なぜこのような「ちょうどよい値」にする必要があるんでしょうか・・・?
0616132人目の素数さん
垢版 |
2021/03/14(日) 16:46:18.35ID:pDeYZQUi
質問したのでageてみます・・・。
0617◆QZaw55cn4c
垢版 |
2021/03/14(日) 21:15:34.26ID:rI8Fpsaj
>>598
さらに性懲りもなく >>447 を考え続けています
>>447 ∀g,h ∃x,y gx=h, yg=h
で単位元の存在と一意性を説明できるのでしょうか?私は単位元の存在を >>447 からは証明することができないでいるのです
0618132人目の素数さん
垢版 |
2021/03/14(日) 21:55:31.83ID:Bv2d5rrE
>>617
>>450,454
0619◆QZaw55cn4c
垢版 |
2021/03/14(日) 22:33:12.91ID:rI8Fpsaj
>>618
>>450 をパラフレーズします
∀g,h ∃x,y gx=h, yg=h …@
@で h = g とおくと
∀g∃e ge=g …A
なおAの e は現時点では g に依存するから今後 e = e(g)(e は g の関数)、と書く
h・e(g) = y(g, h)・g・e(g)
= y(g, h)g
= h
すなわち∃e∀f fe = f

うまいですね…∀∃から∃∀を導けるだなんて驚きました
0620132人目の素数さん
垢版 |
2021/03/15(月) 11:24:27.90ID:iZviCeAZ
あともう1つ質問があるのですが、、

大学の専門とかじゃなくて、趣味で情報系数学ちょっとかじってるんですけど、

https://qiita.com/perrying/items/6b782a21e0b105ea875c

ここに詳細が書かれているのですが、
このニューラルネットにより求められた値とは、いったいどんな意味があるんでしょうか・・・

たとえば、初期値が10,20,30だった場合、
平均をとって20、とかが理想とは限らないんですよね?
どういう処理を施した値なんでしょうか・・・重みとかが関係していて、いまいちよく分からないのですが。。。
0621132人目の素数さん
垢版 |
2021/03/15(月) 11:57:16.10ID:BTbC2l05
>>620
そもそも、ニューラルネットワークの学習の目的は誤差関数(正解と今の差)を、重みとバイアスを更新して最小化すること
ただこの記事では誤差関数について説明していないので、その辺がなあなあに済まされている
0622132人目の素数さん
垢版 |
2021/03/15(月) 20:23:44.76ID:Usc6SLBO
>>621
ん〜説明して頂いた事を元に自分で今考えてみたのですが、
つまり、例えば顔の画像のニューラルネットワークによる認識であれば、
(バイアスはややこしいので省略して)ある重みで複数の層を通して?各行の計算値を出して、
その値が、「一般的な顔が示す値」と差があれば、その差が縮まるように、
重みを更新し、再度計算値を出して・・・を繰り返すんですかね?

一度全部計算してから重み更新するのかな・・・?
0623132人目の素数さん
垢版 |
2021/03/15(月) 20:45:53.46ID:Rlb9KUR/
顔の何を認識するんだ?
「顔である事」か?
どんな入力も「顔である」と認識して終わりとか?
0624132人目の素数さん
垢版 |
2021/03/15(月) 22:10:48.76ID:B9v5y10k
イデアルを導入する意味ってなに?
0627132人目の素数さん
垢版 |
2021/03/16(火) 11:34:28.43ID:x0ceTPrE
>>623
顔が、A君かB君の、どちらに似ているかを識別する、で、どうでしょうか。
この場合、>>622に書いたような仕組みになっているんでしょうか。
0628132人目の素数さん
垢版 |
2021/03/16(火) 14:01:14.65ID:/1jiOv6B
まあつまり、この式におけるf(x)が誤差関数?(教師データと現実データの差、二乗したりしたやつ)
なんですかね?
0629132人目の素数さん
垢版 |
2021/03/16(火) 14:42:34.10ID:41+AbIl4
>>628
そう
Qiitaの記事より長いけど「ニューラルネットワークと深層学習」っていう翻訳プロジェクトのサイトとか分かりやすかった気がする
0630132人目の素数さん
垢版 |
2021/03/16(火) 14:48:06.47ID:wbILBVOq
ちょっとだけ読んだことがあります、
大学レベルの数学を使っているので、分からない部分が自分には多かったので・・・
重みの更新というのは、全ての計算を終えた結果f(x)に対し、再度行われるんですね。
つまり全ての計算を一度行った後で、重みを全て更新、再度計算・・・
という感じですかね。
0631132人目の素数さん
垢版 |
2021/03/16(火) 14:57:33.60ID:41+AbIl4
>>630
逆伝搬だからな
無理して高校数学で理論を展開するほうが、個人的には困難な気がするけど
0632630
垢版 |
2021/03/16(火) 18:02:59.66ID:s6eqSaOh
ttps://www.yukisako.xyz/entry/backpropagation

こことか結構分かりやすいと感じました。
0634132人目の素数さん
垢版 |
2021/03/17(水) 01:17:01.64ID:hTyN+201
>>633
ちょっと色々調べて詳しくなったんですが、ではA君の顔を教師データとしましょう。
すると、教師データと出力の各値の差の二乗和が、誤差になるわけです。
で、より教師データに近づけていくのですが、たぶんこの調整の回数を調整し、
ある回数での二乗和の値を見て、A君よりかB君よりかを決めるのではないのでしょうか・・・。

これ誤差は今書いたように、差の二乗を使うんですよね・・・?
0636132人目の素数さん
垢版 |
2021/03/17(水) 01:38:31.68ID:hTyN+201
最終的には、A君よりがYES,B君よりがNOの2値になると思いますが、
画像の1行目、2行目、3行目・・・をニューラルネットワークにぶち込んだ、
各行の計算結果が出力であると思います。
0638132人目の素数さん
垢版 |
2021/03/17(水) 03:45:35.99ID:z51481pe
数学の記号で質問です。
変数の上に波線(チルダ?)を付けるとどんな意味になるのでしょうか?
https://dotup.org/uploda/dotup.org2416453.png
0640132人目の素数さん
垢版 |
2021/03/17(水) 10:19:33.19ID:hTyN+201
>>637
修正をある程度加えないと、ちゃんと識別できないからではないでしょうか。
素のデータだと、識別不能になり、A、Bの確率がどちらも50%に近くなる、とか?
0641132人目の素数さん
垢版 |
2021/03/17(水) 10:28:19.08ID:unVFkzVS
>>638
そういう付加的な記号は結構ややこしい文化で、伝統的に使い方がある程度決まってるものと論文によって違う使われ方をしてるものが混在してる
チルダは後者だと思う
0642640
垢版 |
2021/03/17(水) 13:45:21.03ID:qNpqfy2u
あ、ちょっと間違ってるか・・・
「各行の値の出力値」ではなくて、「各行の値を1出力した、出力値(つまり1つの値)」をf(x)の値として、
この増加分を考えて重みの調整を行う、んですかね、
合ってますかね。
0644132人目の素数さん
垢版 |
2021/03/17(水) 15:18:51.51ID:mhPW5G/9
中間データというか、出力データを重み微分で調整するっぽいです、複数回。
0646132人目の素数さん
垢版 |
2021/03/17(水) 18:19:04.91ID:pX5iuLxQ
じゃ自分の説明、理解がどこか間違ってるんですね、
実際にこのシステムは色んな所で使われてるようなので。
0647132人目の素数さん
垢版 |
2021/03/18(木) 01:07:19.14ID:KsOoVh5F
あと、
誤差逆伝播法(バックプロパゲーション)
っていまだに何なのか分からないのですが・・・
合成関数の微分のことでは、、ないんですかね。
0649630
垢版 |
2021/03/18(木) 09:49:01.68ID:UMJDgrmw
勉強ちょいちょいしてて疑問が湧いたのですが、
重みを更新する際に必要な、重みで微分を行う関数F(x)って、
http://jump.5ch.net/?https://qiita.com/perrying/items/6b782a21e0b105ea875c
ここには、
ニューラルネットワークの関数、とだけあるんですが、
これは、各行の出力結果(複数)なのか、それとも各行を最終的に1出力した出力結果(1つ)
なのかどちらでしょう、今のところ後者だと思っていたのですが、二乗和誤差とか出てくるし、前者かも・・・。

w(new)=w(old)-lr∂F(x)/∂w(old)の、F(x)の元の関数の事です。
0650132人目の素数さん
垢版 |
2021/03/18(木) 10:00:57.60ID:iV7Uj5DA
>>649
w(new)=w(old)-lr∂F(x)/∂w(old)
のFに本来入るべきなのは誤差関数だが、この記事では誤差関数を説明していないので、それっぽい関数で誤魔化しているんじゃないだろうか(実際、著者も「f(X)=……のような一つの関数」と含みのある書き方をしている)

この記事は、活性化関数の説明さえないので、雰囲気を伝えるための簡単な記事で、これだけで完結することは想定していないと思う
というかむしろこの記事だけでニューラルネットワークを再発明できたらそれはそれで天才だと言える
0651132人目の素数さん
垢版 |
2021/03/18(木) 10:05:26.54ID:iV7Uj5DA
すまん、よく見たら活性化関数や誤差関数は中盤以降で説明してた
だから序盤での説明が非常にざっくりしてるんだな
0652132人目の素数さん
垢版 |
2021/03/18(木) 10:22:24.33ID:UMJDgrmw
誤差が関係してくるので、やっぱ1出力でなく
複数出力の誤差和をF(x)としてるんですかね?
0653132人目の素数さん
垢版 |
2021/03/18(木) 10:31:02.35ID:iV7Uj5DA
>>652
誤差関数の定義によるが、一つの例としてはベクトルの差のノルムの二乗
0654132人目の素数さん
垢版 |
2021/03/18(木) 10:34:51.03ID:UMJDgrmw
結局この例だと、多出力の二乗誤差和とかですかね?1出力じゃなくて。
0655132人目の素数さん
垢版 |
2021/03/18(木) 10:48:17.12ID:iV7Uj5DA
>>654
この例がどの例を指しているのか分からないが、
一般には出力層は1出力かもしれないし2以上かもしれない
0656132人目の素数さん
垢版 |
2021/03/18(木) 13:13:15.46ID:v5HTKLUU
https://qiita.com/perrying/items/6b782a21e0b105ea875c
ここの、
w(new)=w(old)-lr∂F(x)/∂w(old)
このF(x)ですが、これ、誤差であり、かつ誤差はΣを使っているため、
通常は複数の出力値と教師データの誤差を考えると思ったのですが、どうでしょうか。
1,2出力と教師データの誤差であれば、1,2個引き算して二乗するだけなので、
わざわざΣを使って表現する必要がないと考えたため。
0657132人目の素数さん
垢版 |
2021/03/18(木) 13:48:31.82ID:v5HTKLUU
ちょっと思ったのですが、この記事もしかして曖昧な部分があるんですかね?
最終の1出力結果を微分した値を用いて、重み更新を行っていますが、
1出力結果を教師データから引いた値(の二乗?)を微分する必要があると思ったのですが
(そうしないと誤差にならない)
0658132人目の素数さん
垢版 |
2021/03/18(木) 14:21:41.48ID:v5HTKLUU
後半の誤差関数の説明の所にも、
ここでxixiはネットワークのi番目の出力、yiyiはi番目の出力に対応した目標の値になります。ニューラルネットワークは設定された誤差関数の値を最小にすることを目標とします。なので平均二乗誤差を用いるとxixiとyiyiの値の差がゼロ、すなわち全く同じ値になるように重みが更新されていきます。以下、実装例になります。
と、ちゃんと、「i番目の出力」ってありますし・・・いくつもの層を通した後の、活性化後?の各行の出力ですかね?
0660132人目の素数さん
垢版 |
2021/03/19(金) 13:03:24.37ID:mswUQLQO
http://imepic.jp/20210319/465330

の(3)にあるような書き方は正規というか、標準的なものなのでしょうか?

  lim[z→0]{f(x+z)-f(x)}/z = f'(z)|z=x ……(3)

の右辺は結局のところf'(x)なのでしょうから

  lim[h→0]{f(x+h)-f(x)}/h = f'(z)|z=x

  lim[z→0]{f(x+z)-f(x)}/z = f'(t)|t=x

としてもいっしょですよね?
0661132人目の素数さん
垢版 |
2021/03/19(金) 13:36:08.42ID:Wc4rbPG1
f:M→Nが局所微分同相の定義は
Mの各点pに開近傍Uがあって「f(U)はNで開であり」f:U→f(U)は微分同相
ですがこのカッコの部分は必要なのでしょうか?
つまりカッコを省いた定義からカッコ内が言えたりはしないのかと気になっています
0662132人目の素数さん
垢版 |
2021/03/19(金) 13:50:07.54ID:WMoDLc0T
>>661
平面上の直線について考えたことがある?
0666132人目の素数さん
垢版 |
2021/03/19(金) 17:52:16.32ID:Q88Js0RH
2chのころから気違い掲示板
0667132人目の素数さん
垢版 |
2021/03/20(土) 10:06:50.31ID:E38IU9B1
気違いとまではいかんが、とかく数学の掲示板には奇人・変人が多い
0668132人目の素数さん
垢版 |
2021/03/20(土) 12:52:51.32ID:IlBcMt2O
掲示板関係なく数学好きは気違いとまではいかない奇人変人が多いだろ
0670132人目の素数さん
垢版 |
2021/03/20(土) 19:14:00.76ID:WSRR6ZLh
数式=数学というとんでもない思い違い

物理板、ぷ板、工学板あたりの機械学習関連スレで聞くべき
0671132人目の素数さん
垢版 |
2021/03/20(土) 20:08:02.77ID:LGIMdGFw
>>668
確かに日本ではそうだな
でもフィールズ賞受賞者とかの有能な数学者を見ると、むしろ普通な人が多いんだよね
日本だけが未だに変人奇人が多い
0672132人目の素数さん
垢版 |
2021/03/20(土) 20:16:43.46ID:fHAHXUvI
>>671
最近でもヴィラニとかいるじゃん
0673132人目の素数さん
垢版 |
2021/03/20(土) 20:22:20.23ID:LGIMdGFw
>>672
多いかどうかで言うとね
0675132人目の素数さん
垢版 |
2021/03/21(日) 10:31:45.36ID:v6Ofw5kH
>>674
そうだな
数学者は奇人変人だ、という偏見を言われ続けることによってそうなっていきやすいのかもな
血液型の性格の理屈と同じで
0677132人目の素数さん
垢版 |
2021/03/21(日) 21:54:19.11ID:og2+Sblp
I =(xy) ⊂C[x,y], A= C[x,y]/I, α=x+I ∈ A とする
A_α 同型C[x,1/x] であることを証明せよ

この問題がわかりません。教えてください。 A_αはAの素イデアル(α)による局所化という意味でしょうか。
どういう風に証明を進めれば良いかがわかりません。
この問題は雪江先生の代数学2の本で第一章の問題1.8.3で解答が書いてありませんでした。
0678132人目の素数さん
垢版 |
2021/03/21(日) 21:54:19.14ID:og2+Sblp
I =(xy) ⊂C[x,y], A= C[x,y]/I, α=x+I ∈ A とする
A_α 同型C[x,1/x] であることを証明せよ

この問題がわかりません。教えてください。 A_αはAの素イデアル(α)による局所化という意味でしょうか。
どういう風に証明を進めれば良いかがわかりません。
この問題は雪江先生の代数学2の本で第一章の問題1.8.3で解答が書いてありませんでした。
0679132人目の素数さん
垢版 |
2021/03/21(日) 22:43:56.09ID:wbOvg2wZ
>>677
少なくとも局所化ではない
spec C[x1/x] はspecC[x]から(x)が抜けてるだけだから無限に素イデアル持ってる
もちろん局所環ではない
A_αが局所化の意味なら同型になるはずがない
0680132人目の素数さん
垢版 |
2021/03/21(日) 23:06:20.16ID:8W4pwq2q
>>677
A=C[x,y]/(xy)=C+xC[x]+yC[y]
積はxy=0
何か夢も希望もない環だね
I=(xy-1)とかじゃないの?
0681132人目の素数さん
垢版 |
2021/03/21(日) 23:12:53.61ID:91XYKuZE
まぁしかしApが素イデアルpによる局所化の意味ならA_αはC[x,1/x]じゃなくてC[x]_(x)にはなるしC[x,1/(x-a)]_(a≠0)にはなるけどな
ま、作問ミスやな
0682132人目の素数さん
垢版 |
2021/03/21(日) 23:40:00.87ID:og2+Sblp
すいません 
どのようにすると A_α 同型 C[x]_(x) 同型 C[x,1/(x-a)]_(a≠0)になりましたか?
問題文自体は見間違えていませんでした。
0683132人目の素数さん
垢版 |
2021/03/21(日) 23:44:16.11ID:og2+Sblp
A_αが仮にAの素イデアルαによる局所化だとするとA_αはどのような感じになりますか?
見やすくなる書き方はあるのでしょうか
0684132人目の素数さん
垢版 |
2021/03/22(月) 01:01:51.66ID:7pFo4sNn
すいません問題が理解できました。
環Aに対し、元x∈Aによる局所化A_xとはAの積閉集合 S={x^n|n=0,1,2,…} による局所化A_Sのことを指します
A_αはこのことでした。すみませんでした。
0685132人目の素数さん
垢版 |
2021/03/22(月) 07:11:43.05ID:d1e0wvEa
>>684
1/xが加わるからxy=0からy=0でA_α=C[x,1/x]
0686132人目の素数さん
垢版 |
2021/03/22(月) 10:08:15.24ID:RxJWXm5O
ニューラルネットワークで使われる「逆伝播計算」ってこれつまり単純に、
合成関数の微分、なんですかね?
いくつも関数が合成されてる時に使う。

これをプログラムで組んでいるだけ?
0687132人目の素数さん
垢版 |
2021/03/22(月) 14:18:02.93ID:kFoOblBs
それは使ってる計算手段だけ
微分で得た感度を使って修正を計算する
カルマンフィルターみたいなもん
0688132人目の素数さん
垢版 |
2021/03/22(月) 16:23:59.98ID:YT6v6SoP
逆伝播計算は計算方法の事ですよね・・・?
調整も逆伝播計算に含むんでしょうか・・・。
0689132人目の素数さん
垢版 |
2021/03/23(火) 00:06:18.89ID:sIGCPLOB
バックプロパゲーションなら学習アルゴリズムだから修正も入れなきゃ意味がない
0691132人目の素数さん
垢版 |
2021/03/23(火) 22:48:24.75ID:aJXWsA+r
op,cl をそれぞれ開核作用素、閉包作用素とする

op(cl(A)∩cl(B)) ⊆ cl(A∩B)
って一般に成り立つ?
0693132人目の素数さん
垢版 |
2021/03/24(水) 10:53:09.87ID:CfOa493i
>>692
サンクス


一応、
∀A,B開集合[op?cl(A)∩op?cl(B)=op?cl(A∩B)]
の証明は自分で作ったんだが、皆もこれが成り立つことはどうかんだよな?
ちょっと不安になった
0696132人目の素数さん
垢版 |
2021/03/27(土) 12:02:18.05ID:47ViJMGN
完備束の定義って任意の部分集合が上限・加減を持つだけど、これって空集合を含んでる?
Yesなら常に0,1をもつって事になるが、ほんとにそれでいいのか?
0698132人目の素数さん
垢版 |
2021/03/27(土) 15:36:27.40ID:9y451ySD
それでいい
手元の本でもその定義だし、完備束は0,1を持つと明言してる
0700132人目の素数さん
垢版 |
2021/03/28(日) 22:25:41.47ID:8DBRouEC
ガンマ関数って何に使えるですか?
0702132人目の素数さん
垢版 |
2021/03/29(月) 03:47:27.63ID:a3977UC3
>>701
そんなに出てこなくないですか?
0704132人目の素数さん
垢版 |
2021/03/29(月) 17:45:26.86ID:PbpzhVDM
n! = Γ(n+1) = ∫[0,∞] dt t^n exp(-t)
= ∫[0,∞] dt exp( n*log(t)-t )
= ∫[-n, +∞] dη exp( n*log(n+η)- (n+η) )
〜 exp( n*log(n)- n ) ∫[-∞,+∞] dη exp(- η²/2n)
= (n/e)ⁿ √( 2π n) {スターリング近似}

いろんなところに出てくるよなー
0705132人目の素数さん
垢版 |
2021/03/29(月) 21:52:39.43ID:a3977UC3
ガンマ関数の「値」を使うことってないんですか?

三角関数の値は測量に使える、対数関数の値は3^50とかが何桁の数か知るのに使える

Γ(2.7)とかの値は何かに使えないんですか?
0707132人目の素数さん
垢版 |
2021/03/30(火) 02:46:59.96ID:1nCYsZUW
Γ(1/4)もレムニスケート周率に出てくるので時々使う
0708132人目の素数さん
垢版 |
2021/04/01(木) 01:06:22.39ID:/IRwbsjx
ガンマ関数はsin cos 混じった積分とかx^p (1-x)^q の積分とかの計算に使うイメージしかない
0711132人目の素数さん
垢版 |
2021/04/01(木) 16:51:29.96ID:YlwZ/aV7
>>710
いつも思うが逆だよな
(n,m)=(n+m)!/n!m!
B(x,y)=Γ(x)Γ(y)/Γ(x+y)
階乗とΓ関数も1ずれてるから
1/B(n,m)=(n+m-1)!/(n-1)!(m-1)!=(n+m-1)(n-1,m-1)
で(逆数が)2項係数に対応してるというのもイマイチ
0714132人目の素数さん
垢版 |
2021/04/01(木) 23:39:18.48ID:7Wu6nf5/
X:コンパクトハウスドルフ空間かつ完全不連結
x∈U:Xの開集合

y \not\in Uに対して、「Xが完全不連結ゆえ開閉集合Hが存在してx \not\in Hかつy∈Hとなる」らしいのですが、これはどうしたら証明できますか?
0715132人目の素数さん
垢版 |
2021/04/02(金) 01:35:27.42ID:VfnaXHgm
wikiによると
完全不連結⇔一点集合が連結成分
らしいからH={y}でいいんじゃね?
0716132人目の素数さん
垢版 |
2021/04/02(金) 06:55:49.14ID:XVAiSe8C
>>715
それならそれがコンパクトって有限集合ってことか?
0718132人目の素数さん
垢版 |
2021/04/02(金) 08:00:22.54ID:XVAiSe8C
>>717
連結成分なら閉かつ開だろ
0721132人目の素数さん
垢版 |
2021/04/02(金) 13:27:15.46ID:PK4Jjv0b
>>720
局所連結ならそりゃ開閉ですけど、>>714は局所連結とは限らないですよね?
というか一点が開(かつ閉)なら離散位相だし、ここでコンパクトもハウスドルフも使わないなら完全不連結空間=離散空間になってしまうがそんなことはありえない

コンパクトハウスドルフでも同値にはならない、Qが反例
0726132人目の素数さん
垢版 |
2021/04/03(土) 04:55:35.52ID:uoyDCoCd
>>714
これがらみの日本語のwikiにはそれっぽい事がいっぱい書いてあるんだけど一つもリファレンスがついてない
完全にトウシロウが書いてる
正しいんかなコレ?
0727132人目の素数さん
垢版 |
2021/04/03(土) 05:38:55.43ID:8UGtc0Co
>>722
それコンパクトぢゃなくね
0728132人目の素数さん
垢版 |
2021/04/03(土) 08:46:32.11ID:ZZ81aGUe
>>725-726
ありがとうございます
>>714はとある本で、開閉集合からなる開基をもつことを示せという問題の解答に書かれてました

>>727
oh……
p進整数環Zpなら反例になるかな
0731132人目の素数さん
垢版 |
2021/04/03(土) 10:06:26.64ID:8UGtc0Co
wikipediaに書いてあるquasi componentとcomponentの一致が成り立てば証明できる
0732132人目の素数さん
垢版 |
2021/04/03(土) 10:30:34.58ID:uoyDCoCd
まだ眺めただけだけど>>729はself containedで証明自体は1ページ以内に収まるみたいだな
Theorem 2.1がそれだけどp2の2行目で証明終わっててそこまでで他の文献参照したりもしてない
まぁジェネラルトポロジーだからそりゃそうなんだろうけど
0733132人目の素数さん
垢版 |
2021/04/03(土) 12:37:51.22ID:YKM60Lf9
>>729みたいな読みづらい、うだうだ書いてる、がしかし、内容的には学部生でも読めるような奴を皆で読まないか?
勉強ノートみたいなので読みやすく纏めたら読者的には助かる
0735132人目の素数さん
垢版 |
2021/04/03(土) 12:39:50.55ID:YKM60Lf9
any second countable totally disconnected compact Hausdorff topological space X without isolated points
is homeomorphic to the Cantor middle-third set.
ってあるが、Cantor middle-third set.ってなんだ?
0736132人目の素数さん
垢版 |
2021/04/03(土) 12:59:32.50ID:H2a/+aEw
>>735
いわゆるカントール集合のこと
0739132人目の素数さん
垢版 |
2021/04/03(土) 17:46:10.97ID:YKM60Lf9
>>729を誰かきれいに勉強ノートとってアップしてくれたら俺がもっと綺麗に整えてデータ化してあげるぞ
0742132人目の素数さん
垢版 |
2021/04/03(土) 21:00:35.09ID:uoyDCoCd
>>740
thx
これ答え見たらわずか20行足らずのそんな難しい問題じゃなかったと言えるけど、初学者にやらせる練習問題としてはかなりむずいよな
0743132人目の素数さん
垢版 |
2021/04/03(土) 22:14:25.25ID:ZZ81aGUe
>>742
写真見てわかる通り、それまでの問題が割と簡単だったから「これも実は簡単なんかな……」と軽く落ち込みつつ解答見たら>>714で詰みましたww w w w w
0744132人目の素数さん
垢版 |
2021/04/04(日) 01:17:57.86ID:TZxEW7Ub
>> 714

ブルバキ数学原論, 和訳の位相第1巻 pp.177-178 に、証明があります。

一般にコンパクトハウスドルフ空間 X では、X の 任意の点 x に対して,
x の X における開かつ閉な近傍全体の共通部分が x を含む、
X における 連結成分 C(x) となります。

この場合は X の全不連結性より, C(x) = {x} となりますね.
0745132人目の素数さん
垢版 |
2021/04/04(日) 01:21:25.58ID:TZxEW7Ub
>>743 私もその回答ではわかりませんでした. この問題は少し難しいですよ.
0746132人目の素数さん
垢版 |
2021/04/04(日) 21:51:47.88ID:vSr9aCmj
https://i.imgur.com/ghaP4KM.png
https://i.imgur.com/AMUOntQ.png

2枚目の方の、「いままでのべたMについての条件I,II,III,IVはP∈MでMが集合論のモデルである時は容易にわかるように常に満たされる」
とあるが、この”満たされる”というのは、Pに対してなのか、B^Mに対してなのか、どっち?

マジで分からんのだが。。。。
0747132人目の素数さん
垢版 |
2021/04/05(月) 04:16:35.45ID:31QGLH8M
書名を明示せよ
0750132人目の素数さん
垢版 |
2021/04/05(月) 19:28:18.06ID:p9KUk2mI
竹内外史の現代集合論入門109ページ
Bをブール代数とした時、
   {b_i|i∈I} ∈ M ならば、Σb_iが存在する
時、BはM-completeという。


とあるが、Mに何の条件も課されていないので、通常のブール代数で成り立つ、
   Σb_i、Πb_iの一方が存在したら他方も存在して、これらは互いに-Σb_i=Π-b_iが成り立つ
っていう性質は持たないよな?
だから、M-completeだけの時は、Σb_iの存在だけからΠb_iの存在は何も言えないよな?
0754132人目の素数さん
垢版 |
2021/04/06(火) 08:47:26.85ID:XcOsplES
>>750
>とあるが、Mに何の条件も課されていないので、通常のブール代数で成り立つ、
xを実数とした時x<yであればxはy未満と定義する
に対して
yに何の条件も課されてないので通常の実数xで成り立つ
0757132人目の素数さん
垢版 |
2021/04/06(火) 15:49:27.75ID:6g8dbQlt
竹内外史の現代集合論入門113ページ
GがP-generic over Mであることの証明だが、
{-[p]^{-○} | p \in S }が∈Mである時に初めて、Π_{p \in S} -[p]^{-○}の存在が言えるんだが、
∈Mであることの証明がなされていない。

で、∈Mであることの証明が全く分からん
0758132人目の素数さん
垢版 |
2021/04/06(火) 18:23:00.00ID:6g8dbQlt
>>757
竹内外史のaxiomatic set theory p33,
倉田令二郎、篠田寿一の公理的集合論 p145
でもほぼ全く同じの議論してるけど、どこも「{-[p]^{-○} | p \in S }が∈M」の証明してないw
0759132人目の素数さん
垢版 |
2021/04/06(火) 19:26:23.33ID:D9cznXAr
コルモゴロフ、フォミーン著『函数解析の基礎原書第4版上』

定義:
空でない集合系 R に対して、 A ∈ R, B ∈ R ならばつねに A △ B ∈ R、 A ∩ B ∈ R となっているとき、 R を(集合)環という。

定理:
任意の空でない集合系 S が与えられたとき、 S を含み、かつ、 S を含む任意の環 R^* に含まれる環 R(S) が、一つしかもただ一つ存在する。

この定理ですが、
>>631
の2分木で表せるような集合全体の集合を考えると、明らかに、 △、∩ について閉じているので、 R(S) が一意的に存在するのは明らかだと思いますが、
コルモゴロフらは、 S を含むような環たちの共通分をとって、それが R(S) であるなどと長い議論をしています。

無駄に複雑な証明をしているのはなぜでしょうか?

△、∩ の演算子を有限回使って、表わされるような集合全体の集合が求める環であると書けば、一行で済む話です。
0761132人目の素数さん
垢版 |
2021/04/06(火) 20:29:48.05ID:D9cznXAr
コルモゴロフ、フォミーン著『函数解析の基礎原書第4版上』ってよくあるルベーグ積分の本での測度論のところに登場する有限加法族とかσ加法族とかよりも一般的
な環、半環、σ環について書いてあるんですね。

有限加法族とかσ加法族しか書いていないほうが確かに分かりやすいと思いますが、一般的に書いてあるのも魅力的ですね。

コルモゴロフらの本では、有限加法族は代数、σ加法族はσ代数と読んでいます。
0762132人目の素数さん
垢版 |
2021/04/06(火) 23:18:20.03ID:XcOsplES
>>755
あららw
0763132人目の素数さん
垢版 |
2021/04/07(水) 03:59:53.34ID:VI4TCFlo
テンソル積って、要は、「行列同士の要素の総当たり積」ってことでOK?
0766132人目の素数さん
垢版 |
2021/04/07(水) 11:40:06.96ID:2+tGg55i
で読み進めてまたわからないとこが出てくると本のせいにして文句言って投げ出す
の無限ループ
いつまでだっても一歩も進まない
0768763
垢版 |
2021/04/07(水) 14:19:02.73ID:VI4TCFlo
>>764
>>767
どっちなんだろう?
0769132人目の素数さん
垢版 |
2021/04/07(水) 16:53:06.58ID:3yLKAlGb
a_{ijk} と b_{nm} のテンソル積は a_{ijk}b_{nm} だ
要素の総当たり積で合ってる
0770132人目の素数さん
垢版 |
2021/04/07(水) 17:05:27.71ID:3yLKAlGb
>>768
多分 >>764 は座標変換で共変とか反変とか文句つけるんだろう
微分幾何で使うベクトルやテンソルならそういう性質を追加する必要があるが
線形代数のベクトルには関係ない
データベースに利用するなら演算さえ不要
0771132人目の素数さん
垢版 |
2021/04/07(水) 17:21:30.64ID:LZwL63FH
単にベクトル空間って数字が並んでるものだよねって言って
いいと思うか悪いと思うかだけの差
0773132人目の素数さん
垢版 |
2021/04/08(木) 04:24:09.28ID:rTVA1Wui
コルモゴロフ、フォミーン著『函数解析の基礎原書第4版上』

定義1:
B を空でない集合系とする。B が以下の(1), (2), (3)を満たすとき、B をσ代数という。

(1) a ∈ B, b ∈ B ならばつねに a △ b ∈ B、 a ∩ b ∈ B が成り立つ。
(2) a_n ⊂ B for n = 1, 2, … ならば、 ∪_{n=1}^{∞} a_n ∈ B が成り立つ。
(3) e ∈ B が存在して、任意の a ∈ B に対して、 a ∩ e = a が成り立つ。この e を B の単位元という。

定義2:
S を空でない集合系とする。
B を S を含むσ代数とする。
∪_{a ∈ S} a が B の単位元になっているとき、 B は S に関して既約であるという。

定理1:
空でない集合系 S に対して、 S を含む任意の S に関して既約なσ代数に含まれるようなσ代数 B(S) が存在する。

定義3:
f : m → n を写像、 N を n の部分集合からなる集合系とする。
f^{-1}(N) で集合系 N に属する集合 b の逆像 f^{-1}(b) の全体を表わすことにする。

定理2:
B(f^{-1}(N)) = f^{-1}(B(N)) が成り立つ。

------------------------------------------------------------------------------
定理2ですが、

f^{-1}(B(N)) が f^{-1}(N) に関して既約なσ代数であることは簡単に証明できました。
定理1により、 B(f^{-1}(N)) ⊂ f^{-1}(B(N) が成り立ちます。

B(f^{-1}(N)) ⊃ f^{-1}(B(N) が成り立つことが証明できません。

どう証明すればいいのでしょうか?

663 返信:132人目の素数さん[sage] 投稿日:2021/04/07(水) 20:58:42.88 ID:90BIMoih [2/2]
>>662
馬鹿アスペ二号

664 自分返信:132人目の素数さん[] 投稿日:2021/04/07(水) 21:04:34.09 ID:mYnipKIn [2/3]
>>662
この定理2ですが、この結果を後の章で可測函数を考察する際に必要になるそうです。
それにもかかわらず、証明が書いてありません。
0776132人目の素数さん
垢版 |
2021/04/08(木) 13:16:14.88ID:rTVA1Wui
>>774-775
証明してください。
0778132人目の素数さん
垢版 |
2021/04/08(木) 13:52:33.21ID:rTVA1Wui
>>773
あ、簡単ですね。

B(N), B(f^{-1}(N)) の元がどのような元からなるかを考えれば、以下の式から明らかですね。

f^{-1}(∪ a_n) = ∪ f^{-1}(a_n)
f^{-1}(a △ b) = f^{-1}(a) △ f^{-1}(b)
f^{-1}(a ∩ b) = f^{-1}(a) ∩ f^{-1}(b)
0779132人目の素数さん
垢版 |
2021/04/08(木) 13:54:34.55ID:rTVA1Wui
そして、コルモゴロフらがなぜこの命題の証明を書かなかったのかも推測できます。

B(N), B(f^{-1}(N)) の元がどのような元からなるかを正確に記述するのが面倒だからでしょうね。
0780132人目の素数さん
垢版 |
2021/04/08(木) 14:00:34.83ID:rTVA1Wui
自身の筆力・記述能力がないために、容易だから読者に任せるというパターンはよくありますよね。
確かに容易ではあるのですが、正確に記述するのは面倒というパターンです。

迷惑な話です。

そして、同じように容易な話でも記述するのが簡単な場合には喜んで書いていたりするんですよね。

松坂和夫さんとかによくあるパターンです。
0781132人目の素数さん
垢版 |
2021/04/08(木) 14:14:06.36ID:zXDjy5By
・NからB(N)をどうやって組み立てるか
・∩ や ∪ についての f^{-1} の性質
それ考えたらほぼ自明じゃね?
0782132人目の素数さん
垢版 |
2021/04/08(木) 14:17:19.83ID:rTVA1Wui
>>781
それでは、B(N)がどのような元から構成されるのか数学的に厳密に記述してください。
0783132人目の素数さん
垢版 |
2021/04/08(木) 14:27:37.77ID:rLZHI9gC





0784132人目の素数さん
垢版 |
2021/04/08(木) 14:28:52.95ID:zXDjy5By
順番に拡張しながら全部ぶっこむ
1. e
2. eについての補集合
3. 有限積 ∩ a_k
4. 無限和 ∪ a_k
終わり
0785132人目の素数さん
垢版 |
2021/04/08(木) 15:00:35.33ID:ODPkq44X
>>773
大学学部レベル質問スレ 15単位目
778 :132人目の素数さん[]:2021/04/08(木) 13:52:33.21 ID:rTVA1Wui>>773
あ、簡単ですね。

B(N), B(f^{-1}(N)) の元がどのような元からなるかを考えれば、以下の式から明らかですね。

f^{-1}(∪ a_n) = ∪ f^{-1}(a_n)
f^{-1}(a △ b) = f^{-1}(a) △ f^{-1}(b)
f^{-1}(a ∩ b) = f^{-1}(a) ∩ f^{-1}(b)

数学の本 第80巻
150 :132人目の素数さん[]:2021/04/08(木) 13:52:54.66 ID:rTVA1Wui>>148
あ、簡単ですね。

B(N), B(f^{-1}(N)) の元がどのような元からなるかを考えれば、以下の式から明らかですね。

f^{-1}(∪ a_n) = ∪ f^{-1}(a_n)
f^{-1}(a △ b) = f^{-1}(a) △ f^{-1}(b)
f^{-1}(a ∩ b) = f^{-1}(a) ∩ f^{-1}(b)
0786132人目の素数さん
垢版 |
2021/04/08(木) 15:19:55.57ID:AiTgWBpE
こういう議論って、集合系Sが、議論の土俵となっているような全体空間Xでの話なら証明はもっと簡単なんだが、
そういう全体の空間Xが定まってないんだろ?
すんげぇダルい話だよな
0787132人目の素数さん
垢版 |
2021/04/08(木) 16:30:57.17ID:zXDjy5By
>>784
よく考えたらボレル集合族みたく積み上げる必要あるから
有限段では終わらないな
0788132人目の素数さん
垢版 |
2021/04/08(木) 16:31:28.57ID:6ao9oBLQ
すげぇだるいが初学者はやらんといかん
やってるうちに
どっちの方が難しいのか
難しい方は大体どんな方法で示せばいいのか、示せるのか
の勘が養われてくる
今回ので言えばB(f^(-1)(S))⊂f^(-1)(B(S))はほぼ自明で反対がめんどくさい
可能なら⊂と同じ方法で示したいが無理だなぁというのがある程度勉強ができてる人間の感覚
松坂くんはその感覚が正反対
全く育ってない
しょうもない些細なことばかりに神経使ってそういう肝心の感覚がまるで育ってない
もう無理やろ
0791132人目の素数さん
垢版 |
2021/04/09(金) 23:07:03.13ID:jJy/FKfA
Xをブール代数
IをXのイデアルとする
x≡yをx・(-y),y・(-x)∈Iで定義する。
同値類の集合全体X/Iがまたブール代数になる

この時、Iが極大イデアルならば、X/Iは{0,1}に同型になる

この証明がわからない
0793132人目の素数さん
垢版 |
2021/04/10(土) 08:09:30.26ID:M0Yz4gOJ
ちょっとこれは、かなり難しく、力をお貸しいただきたいのですが、

自分でニューラルネットワークを作ろう
https://qiita.com/takahiro_itazuri/items/d2bea1c643d7cca11352#comment-a59cd26161ee56ea1220
の記事で質問があるのですが、

なんやかんやで大体ざっとは理解できたのですが、
重みの更新式
# 重みの更新
self.w_ho += self.lr * np.dot((e_o * self.daf(o_o)), o_h.T)
self.w_ih += self.lr * np.dot((e_h * self.daf(o_h)), o_i.T)
この式の意味が本当に分かりません。
※*は、成分同士を掛けて行列積を求めるもので、np.dotは普通の行列積になります。

一応

隠れ層から出力層への重みによる偏微分
入力層から隠れ層への重みによる偏微分

の部分は読んで、まぁそうなるんだろうなとざっと理解でき、

【深層学習】誤差逆伝播法|バックプロパゲーション
ttps://youtu.be/X8hK4ovQoLg
この動画を見たりしたのですが、

まず1つ目の
self.w_ho += self.lr * np.dot((e_o * self.daf(o_o)), o_h.T)
からよく分かりません。

可能であれば、複数の式になって良いので、スカラーによる計算式で示して頂きたいのですが・・・なぜこのようになるのか、を。
たぶん、
隠れ層から出力層への重みによる偏微分
入力層から隠れ層への重みによる偏微分
にある「重みの式」に代入していくような感じだとは思うのですが・・・。
0794132人目の素数さん
垢版 |
2021/04/10(土) 15:56:34.78ID:GIzYdAVL
竹内外史の現代集合論入門と竹内外史の「Axiomatic set theory」が書いてることほぼ同じだけど
和書の方では明らかって言ってたのが、洋書の方ではその行間の証明が8行ぐらい書かれてたわ

なんで和書ってこんなにも読者を馬鹿にするのか
0795132人目の素数さん
垢版 |
2021/04/10(土) 16:11:01.78ID:Z9sY9TKp
>>793
ニューラルネットワークの知識ゼロで読んだけど
そんな「かなり難しい」というほどの事はしてないよ

Sigmoid 関数 φ(x) := ...
φ'(x) = φ(x) (1-φ(x)) (★ φ'(x)=x(1-x) は間違い)
y = φ(x), φ'(x) = y (1-y) =: daf(y)

隠れ層
x_hᵢ := Σ{j} W_ihᵢⱼ o_iⱼ
o_h := φ(x_h)

出力層
x_oᵢ := Σ{j} W_hoᵢⱼ o_hⱼ
o_o := φ(x_o)

評価関数 E := Σ{k} 1/2*(tₖ-o_oₖ)² {極小値となるパラメータ W を求める}
e_oᵢ := -∂E/∂o_oᵢ = tᵢ - o_oᵢ
e_hᵢ := Σ{j} daf(o_oⱼ) W_hoⱼᵢ e_oⱼ (★プログラムの記述は間違い)

E値が小さくなる方向にWを更新
ΔW_hoᵢⱼ ∝ -∂E/∂W_hoᵢⱼ
= Σ{k,m}(-∂E/∂o_oₖ)(∂o_oₖ/∂x_oₘ)(∂x_oₘ/∂W_hoᵢⱼ)
= e_oᵢ daf(o_oᵢ) o_hⱼ

ΔW_ihᵢⱼ ∝ -∂E/∂W_ihᵢⱼ
= Σ{k,m,n,s} (-∂E/∂o_oₖ)(∂o_oₖ/∂x_oₘ)
 * (∂x_oₘ/∂o_hₙ)(∂o_hₙ/∂x_hₛ)(∂x_hₛ/∂W_ihᵢⱼ)
= Σ{m} e_oₘ daf(o_oₘ) * W_hoₘᵢ daf(o_hᵢ) o_iⱼ
= e_hᵢ daf(o_hᵢ) o_iⱼ
0796132人目の素数さん
垢版 |
2021/04/10(土) 16:36:11.13ID:oqfmB9cK
ちょっと順に読ませて頂きます、

φ'(x) の式ですが、これはxにφ(x)を代入したら問題ないですよね、
が、、実際そうなってないんですかね・・・?
0797132人目の素数さん
垢版 |
2021/04/10(土) 17:00:53.52ID:Z9sY9TKp
>> 786
とにかく φ’(x) = x (1 - x) の式は間違い、ただそれだけの話

φ(x) = 1 / (1 + e^{-x})
φ’(x) = e^{-x} / (1 + e^{-x})² = (1+ e^{-x} -1) φ(x)² = (1 - φ(x)) φ(x) =: daf( φ(x) )

コード上では daf(x) ではなく daf( φ(x) ) 相当の扱いなので問題ない
0798132人目の素数さん
垢版 |
2021/04/10(土) 17:07:18.28ID:oqfmB9cK
これ、本来はΣi(i=0から最大値まで?)ではなく行列を使うんですよね、
それを行列の幅?を最大値として、Σを用いスカラー式で示して頂いていると・・・

e_hᵢ := Σ{j} daf(o_oⱼ) W_hoⱼᵢ e_oⱼ
このe_hって、そもそも、何を意味するんですかね?
0799132人目の素数さん
垢版 |
2021/04/10(土) 17:31:43.39ID:Z9sY9TKp
>>798
例.
(A v )ᵢ = Σ{j} Aᵢⱼ vⱼ = Σ_{j=1}^{3} Aᵢⱼ vⱼ = Aᵢ₁v₁ + Aᵢ₂v₂ + Aᵢ₃v₃
( B C v )ᵢ = Σ{j,k} Bᵢⱼ Cⱼₖ vₖ = Σ_{j=1}^{n} ( Bᵢⱼ Σ_{k=1}^{m} Cⱼₖ vₖ ) = ...
説明は不要かと...

e_o , e_h の定義は、そう置くと
 ΔW_hoᵢⱼ ∝ e_oᵢ daf(o_oᵢ) o_hⱼ
 ΔW_ihᵢⱼ ∝ e_hᵢ daf(o_hᵢ) o_iⱼ
お揃いの形になって気持ちいいから、たぶんそんな感じ
0800132人目の素数さん
垢版 |
2021/04/10(土) 17:54:06.75ID:oqfmB9cK
なるほど、なんとなくわかりました。

ΔW_hoᵢⱼ ∝ -∂E/∂W_hoᵢⱼ
= Σ{k,m}(-∂E/∂o_oₖ)(∂o_oₖ/∂x_oₘ)(∂x_oₘ/∂W_hoᵢⱼ)
= e_oᵢ daf(o_oᵢ) o_hⱼ

これなんですが・・・。
2行目、Σと添え字無視すれば、ただの合成関数の微分なので分かるのですが、、
Σがついているのはこれ、合成関数の偏導関数の公式を使っているからですか???
0801132人目の素数さん
垢版 |
2021/04/10(土) 18:13:08.91ID:Z9sY9TKp
> 合成関数の偏導関数
それです。それと
 ∂o_oₖ/∂x_oₘ = δₖₘ do_oₘ/dx_oₘ = δₖₘ φ’(x_oₘ) = δₖₘ daf(o_oₘ)
 ∂x_oₘ/∂W_hoᵢⱼ = δₘᵢ ∂x_oᵢ/∂W_hoᵢⱼ = δₘᵢ o_hⱼ
この辺りも省略しました
0803132人目の素数さん
垢版 |
2021/04/10(土) 19:37:16.91ID:Z9sY9TKp
ベクトル変数(多変数), ベクトル値の関数 A, B, C を合成して
  y = A( B( C(x) )) とおく

δyᵢ = (∂Aᵢ/∂B₁) δB₁ + (∂Aᵢ/∂B₂) δB₂ + ...
 = Σ{k} (∂Aᵢ/∂Bₖ) δBₖ
 = Σ{k} (∂Aᵢ/∂Bₖ) Σ{h}(∂Bₖ/∂Cₕ) δCₕ
 = Σ{k} (∂Aᵢ/∂Bₖ) Σ{h}(∂Bₖ/∂Cₕ) Σ{j}(∂Cₕ/∂xⱼ) δxⱼ
 = Σ{j} ( Σ{k,h}(∂Aᵢ/∂Bₖ)(∂Bₖ/∂Cₕ)(∂Cₕ/∂xⱼ) ) δxⱼ
∴ ∂yᵢ/∂xⱼ = Σ{k,h}(∂Aᵢ/∂Bₖ)(∂Bₖ/∂Cₕ)(∂Cₕ/∂xⱼ)
何段になっても同様
0804132人目の素数さん
垢版 |
2021/04/10(土) 20:49:10.70ID:eYoFZYDx
コルモゴロフ、フォミーン著『函数解析の基礎原書第4版上』

p.50 演習
ρ_1 > ρ_2, B(x, ρ_1) ⊂ B(y, ρ_2) なる二球 B(x, ρ_1), B(y, ρ_2) をもつ距離空間の例をつくれ。

X を離散距離空間とし、 x, y をその任意の元、 ρ_1 = 3, ρ_2 = 2 とすればよい。

この問題の著者らが想定している模範解答は何ですか?

まさか、こんなつまらない解答を想定してはいないですよね?

もし、こんな解答を想定しているとしたら、物凄い小物数学者のようですよね。
0805132人目の素数さん
垢版 |
2021/04/10(土) 21:04:02.18ID:ezuf2A0N
>>804
君の解答はそのままでは間違っている
xとyの距離が4だったら?
ちょっと訂正したら良いだけだが
0807132人目の素数さん
垢版 |
2021/04/10(土) 21:58:51.03ID:oqfmB9cK
>>803
これってつまり、

1行目は、
∂yᵢ/∂x = (∂Aᵢ/∂B₁) ∂B₁/∂x + (∂Aᵢ/∂B₂) ∂B₂/∂x + ...
という事ですかね?
これ、なんで/∂xをつけない場合は、どういう理屈で∂yᵢ、∂B₁ᵢでなくてδyᵢ、δB₁になるんでしたっけ・・・。
なんか大学の化学の授業ですごく適当にではあるが、やった記憶はあるんですが・・・。
で、2、3、4、5行目は、
 = Σ{k} (∂Aᵢ/∂Bₖ) ∂Bₖ/∂x
 = Σ{k} (∂Aᵢ/∂Bₖ) Σ{h}(∂Bₖ/∂Cₕ) ∂Cₕ/∂x
 = Σ{k} (∂Aᵢ/∂Bₖ) Σ{h}(∂Bₖ/∂Cₕ) Σ{j}(∂Cₕ/∂xⱼ)
 = Σ{j} ( Σ{k,h}(∂Aᵢ/∂Bₖ)(∂Bₖ/∂Cₕ)(∂Cₕ/∂xⱼ) )
であって、

1行目、例えばyᵢがB,CについてB[1],B[2],B[3],C[1],C[2],C[3]のみの関数であれば、
∂yᵢ/∂x = (∂Aᵢ/∂B₁) ∂B₁/∂x + (∂Aᵢ/∂B₂) ∂B₂/∂x + (∂Aᵢ/∂B₃) ∂B₃/∂x
であり、5行目は、
 = Σ{j} ( Σ{k=1,3}Σ{h=1,3}(∂Aᵢ/∂Bₖ)(∂Bₖ/∂Cₕ)(∂Cₕ/∂xⱼ) )
とか書いたりもできるんでしょうか。
0808132人目の素数さん
垢版 |
2021/04/10(土) 23:24:01.90ID:Z9sY9TKp
>>807
その辺の根拠を知りたいなら適当な微積の教科書を読んだほうが早いです
5chやQiita はアテにならないので
0810132人目の素数さん
垢版 |
2021/04/11(日) 21:39:51.22ID:tCRmiMbP
>>793
ぷ板の意見

338 デフォルトの名無しさん (ブーイモ MMff-fxd7 [210.138.177.206]) sage ▼ 2021/04/11(日) 11:31:48.34 ID:cX1p0N8YM [1回目]
>>337
そのQiitaの記事のコードの上の方に数式は具体的に書かれている訳だけども、
まずそっちは理解しているのかな?
理解できてないのであればまずは線形代数をしっかり学ぶ必要があると思う

339 デフォルトの名無しさん (アウアウウー Sa47-hzJq [106.133.47.168]) sage ▼ 2021/04/11(日) 21:23:07.14 ID:J8YGJLtEa [1回目]
>>337
dWの微分を行列で表すとそうなる
ほとんどの本ではそこは省略されてることが多い

341 デフォルトの名無しさん (アウアウウー Sa47-hzJq [106.133.47.168]) sage ▼ New! 2021/04/11(日) 21:29:25.29 ID:J8YGJLtEa [2回目]
スカラーから行列に手計算で直すのが良い
あとミニバッチ対応だと行列以外では表現できないから
行列は必須
0811132人目の素数さん
垢版 |
2021/04/11(日) 22:33:04.76ID:4bCCzhlV
ぷ板だってアテにならないのは同じなのに丸々同じ質問で投げてんのかよ
0812807
垢版 |
2021/04/11(日) 23:16:30.62ID:kmI6gdiU
いや、これは自分がした質問ではなく、>>810さんが質問してくれたものかと。
【統計分析】機械学習・データマイニング30で
0814132人目の素数さん
垢版 |
2021/04/12(月) 01:30:04.59ID:MVTQfLPH
>>795の定義の元で、
 ∂o_oₖ/∂x_oₘ = δₖₘ do_oₘ/dx_oₘ = δₖₘ φ’(x_oₘ) = δₖₘ daf(o_oₘ)
 ∂x_oₘ/∂W_hoᵢⱼ = δₘᵢ ∂x_oᵢ/∂W_hoᵢⱼ = δₘᵢ o_hⱼ
これについてなのですが・・・いまいち記号の意味がよく分かっていないのですが、
まず一行目について、
o_oはx_oをシグモイドに代入したものですよね、それをx_oで微分するのだから、
シグモイドが導関数となって、daf(x_o)とかが出てきそうなのですが、なぜこうなるのでしょうか・・・。
また、δₖₘ はなんなんでしょうか。。

二行目は、重みづけ和をある重みで微分しているので、その重みに掛け合わされた成分が残るという事だと思いますが、
これもδₘᵢはなんなんでしょうか。。
0815132人目の素数さん
垢版 |
2021/04/12(月) 09:37:46.47ID:iFX3F0ya
自分より年齢だいぶ下の人が自分より全然高度な数学をやってホームページ上にPDFを次々と上げてる様子を見たらマジで自分の無能さを痛感する
0816132人目の素数さん
垢版 |
2021/04/12(月) 10:50:41.21ID:yPK2H072
コルモゴロフ、フォミーン著『函数解析の基礎原書第4版上』

x, y ∈ (α, β)
y, z ∈ (γ, δ)



x, z ∈ (α, δ)

が成り立つなどと書かれています。

α < γ < δ < x < β

のとき、 x は (α, δ) に含まれません。

論理的に考えず、なんとなく開区間のイメージを思い浮かべてそれに頼って証明を書いているのがバレてしまいましたね。
0817132人目の素数さん
垢版 |
2021/04/12(月) 11:32:45.59ID:d/DzSP/c
また投げ出す為の準備に入りましたな
そしていつものごとく何も得ず撤退
相変わらずの般教入門レベルで足踏み
結局数学という学問そのものに対する畏敬の念が1ミリもない
それを守り育て次世代に引き継ごうとしている先人の偉業にも1ミリも敬意を持たない
そのクソみたいな精神性で≦という学問に挑めるはずもない
0819132人目の素数さん
垢版 |
2021/04/12(月) 15:33:12.70ID:Hvjjy0ec
>>814
δₖₘ はクロネッカーデルタ: k=mなら 1, k≠mなら 0 の値をとる

o_oₖ = φ(x_oₖ)
k=m ⇒ ∂o_oₖ/∂x_oₘ = φ’(x_oₖ) = daf( φ(x_oₖ) ) = 1 daf( o_oₖ ) = δₖₘ daf( o_oₘ )
k≠m ⇒ ∂o_oₖ/∂x_oₘ = 0 = δₖₘ daf( o_oₘ )
0821132人目の素数さん
垢版 |
2021/04/14(水) 13:05:43.80ID:XG40KOs9
コルモゴロフ、フォミーン著『函数解析の基礎原書第4版上』

Cantorの集合を F とおく。

F + F = [0, 2]

であることを示せ。
0825132人目の素数さん
垢版 |
2021/04/15(木) 16:05:14.36ID:EJgthEHB
リー群の問題でn>1のときU(n)がSU(n)×U(1)と同型でないことを示せという問題があり
リー群として同型でない事は群の中心がそれぞれS^1、Z/nZ×S^1と同型であり
位相空間として同相でないのでとして言えましたが、単なる群として同型でないことも言えるのかが疑問です
S^1とZ/nZ×S^1が群として同型でない事は簡単に言えるのでしょうか?
0826132人目の素数さん
垢版 |
2021/04/15(木) 16:40:40.52ID:X9qcKTqm
それ中心が群として非同型じゃないの?
A:=Z(Un)=U(1)
B:=Z(SU(n)×U(1))=Cn×U(1)
でAには位数がnの約数である元がn個しかないけどBにはn^2個ある
0828132人目の素数さん
垢版 |
2021/04/16(金) 08:02:16.19ID:mIcaOQyb
>>825
>S^1とZ/nZ×S^1が群として同型でない事は簡単に言えるのでしょうか?
n倍が全射かそうじゃないか
0829132人目の素数さん
垢版 |
2021/04/16(金) 12:00:50.87ID:zyJjMz4t
現代集合論入門(竹内外史) p128 定理2
この証明のG2の方の証明で、
ultrafilterの定義の中のfilterの定義を使う場面が出てくるんだが、本書で言うところのfilterじゃなくて強filterを使わないと議論が進まない気がするんだが?
0830132人目の素数さん
垢版 |
2021/04/16(金) 12:22:13.70ID:zyJjMz4t
この本やっぱりおかしいな

本の定義どおりすすめると止まってしまう点が2個あった
多分↓これが正解
1 順序集合上のフィルタの定義はφを除外する
2 順序集合上のフィルタの定義において強フィルタの定義そのものをフィルタの定義とすべき
0831132人目の素数さん
垢版 |
2021/04/16(金) 14:29:20.03ID:ln9vDyfb
竹内外史の本は「間違いだらけの名著」が多い、と森毅が書いていた
0832132人目の素数さん
垢版 |
2021/04/16(金) 14:47:57.37ID:jd5oFRy+
竹内外史の「現代集合論入門」には増補版がでてて
初版ではfilterと強filterを取り違えて色々間違ったことを書いてしまった
みたいなことが書いてあった気がする
0833132人目の素数さん
垢版 |
2021/04/16(金) 15:27:41.87ID:wToAlXH7
自分のてきとうな理解で本を書いているんじゃない?
量子論理の本もあやしそう
0834132人目の素数さん
垢版 |
2021/04/16(金) 15:33:08.13ID:NiDHgcDH
そんな程度の間違いなんかあって当たり前
こんなのでとやかく言ってる奴は受験数学の“1ミリも間違ってはいけない”と言うクソみたいな哲学に染まり過ぎ
むしろその手のミスをなにもしない方が非現実的
まぁその手の松坂くん流はどこかで潰れるから勝手にすればいいが
0836132人目の素数さん
垢版 |
2021/04/16(金) 18:48:20.09ID:ln9vDyfb
>>834
本に書かれてあることを確かめるのは何も悪くない
0837132人目の素数さん
垢版 |
2021/04/16(金) 18:52:16.47ID:NiDHgcDH
>>836
当たり前やん?
むしろ本に書いてあることなんて疑って当然
間違ってることが書いてある本がダメだという評価をするのがバカだと言ってるんだよ
0838132人目の素数さん
垢版 |
2021/04/16(金) 19:48:51.08ID:ln9vDyfb
間違いが多い本はダメ
0839132人目の素数さん
垢版 |
2021/04/16(金) 21:48:53.16ID:VPNEuP5B
間違いが多いのが名著
0840132人目の素数さん
垢版 |
2021/04/16(金) 22:19:21.76ID:SMl20eo3
間違いにも色々ある
・単なる誤植レベルの記述ミスや(修正可能な)計算ミスならあって当たり前
・計算の方針が多少おかしい(けど結果には影響ない)レベルのミスもあって当たり前
・定理の仮定が少し変な程度のミスもまあ普通
・定理が成り立たない(けどその本を読み進める上では特に障害にならない)のは無かったものとして読めばいいだけ
・未定義のものが出てくるのはやめてくれ
・定義が間違い?初学者には厳しすぎるからちゃんと推敲してくれ
・事実上修正不可能な致命的ミスはくたばれ

記述ミスだと思っても、後から見ればちゃんと読めてなかっただけで間違いではないものだったりするし(※個人の感想です)、致命的なミスを犯してる本って言うほどないよね
昔の本ならともかく2000年以降に出た本だと一冊しか見たことない
0841132人目の素数さん
垢版 |
2021/04/16(金) 23:42:28.30ID:zyJjMz4t
>>832
知ってる人がこのスレにいた!

俺が持ってるのは初版第1冊(1973)の。
ちなみに、全くと行っていいぐらい同内容の洋書の「Axiomatic set theory](竹内外史)(1973)では>>830の点は修正されてた
0842132人目の素数さん
垢版 |
2021/04/18(日) 15:34:20.70ID:CNzNTMHH
>>840

>昔の本ならともかく2000年以降に出た本だと一冊しか見たことない

その一冊の書名は?
0843132人目の素数さん
垢版 |
2021/04/18(日) 16:02:41.21ID:N+6EnGHZ
f : R^n→R^m , g : R^m→R^p で f,gがC^2級のとき
g○f もC^2級になると証明なしに書いてあるのですが
やってみても複雑になりすぎてわかりません。
0846132人目の素数さん
垢版 |
2021/04/18(日) 17:39:24.06ID:EeS+MiHU
集合と位相(内田伏一)p.118のチコノフの定理の証明のところで質問です。

『もし、pλ1^(-1)(U1),...,pλn^(-1)(Un) がすべてMに属することが示されたならば、
性質(i)によって、Mに属する任意の集合Fに対して
F ∩ pλ1^(-1)(U1) ∩ ... ∩ pλn^(-1)(Un) ≠ Φ …(※)
となり、yの近傍NがMに属するすべての集合Fと交わることになる。』

とあるのですが、性質(i)をどこで使っているのか分かりません。
Mは有限交叉性を持ち、Fや pλ1^(-1)(U1),...,pλn^(-1)(Un)がMの元であれば、
性質(i)に関係なく(※)が成り立ちそうなのですが、違うのでしょうか?
0848sage
垢版 |
2021/04/18(日) 18:04:37.54ID:EeS+MiHU
>>847
すみません、性質(i)とは以下の(i)です。
『Mを集合Yの有限交叉性をもつ極大な部分集合族とすれば、Mは次の二つの性質を持つ。
(i) Mに属する有限個の集合 F1,...,Fnの共通部分F1∩ … ∩Fn はMに属する。
(A)(省略)』
0849132人目の素数さん
垢版 |
2021/04/18(日) 18:17:57.95ID:FmOnSyhV
ダメや
エスパーできん
全く方針もなんも見えん
Xλがコンパクトな空間
MはX=ΠXλの有限交差性を持つ閉集合の族
だと思うけどUλはなんかの開集合?
0851132人目の素数さん
垢版 |
2021/04/18(日) 19:05:26.47ID:50jkd+y5
>>846
内田の証明は読んでないが、その性質(i)は
有限交差性を持つってことを使って、そこに属する集合が非空だと分かる
0852132人目の素数さん
垢版 |
2021/04/18(日) 20:19:58.62ID:95VobW25
>>846
> 性質(i)に関係なく(※)が成り立ちそうなのですが、違うのでしょうか?
内田本、持ってたので読んでみました。
証明過程で pλ1^(-1)(U1) ∩ ... ∩ pλn^(-1)(Un) それ自体が M に属すかどうかは不要な情報なので
性質(i) は何処で?と疑問に思うのは当然かと思います。
ただし、pλ^(-1)(U) が M に属すのを示すために (ii) を使っています。
ある意味 (ii) は (i) に依存してるので 両方使ってると言ってもいいじゃないでしょうか。

(i) A₁, A₂, ... , Aₙ ∈M ならば
任意のF₁,F₂,...,Fₘ∈M に対して
F₁∩F₂∩...∩Fₘ ∩ A₁∩A₂∩ ... ∩Aₙ ≠ ∅ (有限交差 )
Mの極大性より (A₁∩A₂∩ ... ∩Aₙ) ∈ M

(ii) A が 任意のF∈M に対して, F ∩ A ≠ ∅ ならば
任意の F₁, F₂, ... , Fₘ∈M に対して,
F₁∩F₂∩... ∩Fₘ ∩ A ≠ ∅ { ∵ (i) より (F₁∩F₂∩... ∩Fₘ) ∈ M }
Mの極大性より A∈ M
0854132人目の素数さん
垢版 |
2021/04/19(月) 10:00:42.22ID:ak89FaSV
解析接続の意味がよくわかりません。
定義域が異なる2つの複素関数が或る共通領域で等しいという
単にそれだけのこと(たとえばf(z)=Σz^nとg(z)=1/(1-z))が、
なぜそんなに大したこととして扱われるのでしょうか?
(一致の定理などはわかっているつもりです)
0856132人目の素数さん
垢版 |
2021/04/19(月) 10:33:20.20ID:ak89FaSV
> パッと見のインパクトが大きい

これはどういうことでしょうか?
なにか例はありますか?
0857132人目の素数さん
垢版 |
2021/04/19(月) 10:59:39.23ID:wJ9Ijnpl
別になんでみんなが解析接続を重要な定理として扱ってるか実感が湧かないなら湧かないでいいんじゃないか?
面白みがわからなければ次へ進めないなら次へ進まなければいいだけの話だし
0859132人目の素数さん
垢版 |
2021/04/19(月) 11:07:30.47ID:BmA+KWsf
リーマンのゼータ関数
0860132人目の素数さん
垢版 |
2021/04/19(月) 11:13:15.76ID:BmA+KWsf
f(z)=Σz^nがzの絶対値が1以上のとき発散するのはわかりますね?
0862132人目の素数さん
垢版 |
2021/04/19(月) 11:30:48.26ID:ak89FaSV
たとえば、
 一致の定理によって、f(z)=Σz^n は g(z)=1/(1-z) に解析接続することによって
 複素数全体に定義域を拡大できる。
というような説明がよくあると思いますが、
この言明自体が全く虚偽だと思うのです。
f(z)の定義域は決まっていて拡大できるわけがないですから。
とすると、ここの「解析接続することによって」になにか言葉の綾があるのでしょうか?
0864132人目の素数さん
垢版 |
2021/04/19(月) 11:40:29.58ID:1v96v9Tb
>>862
拡大が一意に定まることが一致の定理の意味ですね
0865132人目の素数さん
垢版 |
2021/04/19(月) 11:56:33.11ID:ak89FaSV
f(z)=Σz^n は、|z|>1 では定義できませんよね?

> 拡大が一意に定まることが一致の定理の意味ですね
ここの「拡大が」が余計だと思います。
0866132人目の素数さん
垢版 |
2021/04/19(月) 12:09:34.62ID:3WME5C/y
工学部1年です
微分積分学の講義で参考書として
理工系の微分積分学
解析入門T(小平邦彦)
解析入門T(杉浦光夫)
のいずれかを買うように言われました
それぞれの特徴、おすすめなど教えてください
0867132人目の素数さん
垢版 |
2021/04/19(月) 12:25:32.46ID:LhSHwvNz
>>865
拡大の理解が間違っているのだと思います
0868132人目の素数さん
垢版 |
2021/04/19(月) 12:50:40.68ID:ak89FaSV
>>867

> f(z)=Σz^n は、|z|>1 では定義できませんよね?

これは、「もちろん定義できない」という答えでいいですか?
0870132人目の素数さん
垢版 |
2021/04/19(月) 13:02:45.35ID:ak89FaSV
「拡大できる以上は」ってどういうことですか?
f(z)=Σz^n は、|z|>1 では定義できない(つまり拡大できない)でしょう?
あるいは無理に定義するとしたら、|z|>1 では+∞とするしかないですよね?
0872132人目の素数さん
垢版 |
2021/04/19(月) 13:31:54.98ID:LhSHwvNz
>>868
私は>>860です。
同じことを書いています。
0873132人目の素数さん
垢版 |
2021/04/19(月) 13:37:56.38ID:LhSHwvNz
>>862
zの絶対値が1未満の領域でf(z)=Σz^n で定まる正則関数は、複素平面から1を除いたものを定義域とする正則関数に一意に拡大されます。
0874132人目の素数さん
垢版 |
2021/04/19(月) 13:56:15.78ID:ak89FaSV
>>873
>zの絶対値が1未満の領域でf(z)=Σz^n で定まる正則関数は、複素平面から1を除いたものを定義域とする正則関数に一意に拡大されます。
ここは、「zの絶対値が1未満の領域でf(z)=Σz^n で定まる正則関数は、|z|<1 で g(z)=1/(1-z) と一致します」なら、
もちろん納得です。
「一意に」も、一致の定理を言いたいのだろうと理解します。
しかし、「拡大されます」の所は納得できません。
一致の定理がそれを保証しているわけでもなく、
f(2)=+∞ならまだしも、f(2)=-1 は全く恣意的な定義です。
0875132人目の素数さん
垢版 |
2021/04/19(月) 14:07:14.90ID:qdTHPbhp
わざとトンチンカンな事を言って教えたがりな有識者から情報を引き出すタイプか
0877132人目の素数さん
垢版 |
2021/04/19(月) 14:17:18.80ID:LhSHwvNz
>>874
単なる連続関数の拡張ならば恣意性はありますが、正則関数の場合は違ってきます。

その理解のままだとリーマン予想の意味が全く分からなくなります。
0878132人目の素数さん
垢版 |
2021/04/19(月) 14:24:51.84ID:ak89FaSV
>>877
>単なる連続関数の拡張ならば恣意性はありますが、正則関数の場合は違ってきます。
一致の定理はわかっているつもりです。
いまのところリーマン予想を知りたいわけでもありません。

ここで皆さんが言う「拡大できる」というのは、
「必然的に拡大される(拡大しないと矛盾する)」という意味ではなく、
「拡大しても矛盾はしない」という意味なのでしょうか?
0879132人目の素数さん
垢版 |
2021/04/19(月) 14:31:47.56ID:LhSHwvNz
必然的に拡大される、という言い回しを使っているということは、岡潔の業績をご存知なのでしょうか?
今は、それとは違っていて後者の話です。
0881132人目の素数さん
垢版 |
2021/04/19(月) 14:43:36.33ID:ak89FaSV
>>879
「拡大できる」=「拡大しても矛盾はしない」という意味だということですね。
それならダメとはいえませんね。
もともと定義されていないところはどう追加定義しても矛盾はしないでしょう。
でも、f(2)=+∞でなく f(2)=-1 と定義するのは、元のf(z)=Σz^n の情報を無視した
不自然で乱暴な拡大定義ですね。
0882132人目の素数さん
垢版 |
2021/04/19(月) 14:49:58.83ID:LhSHwvNz
>>881
むしろ一番自然で乱暴さのない定め方
0885132人目の素数さん
垢版 |
2021/04/19(月) 16:58:33.98ID:LhSHwvNz
>>884
失われていません。
拡張した関数を元の定義域に制限して0を中心にべき級数展開したらΣz^n になります
0886132人目の素数さん
垢版 |
2021/04/19(月) 18:08:20.77ID:LhSHwvNz
多価関数からリーマン面の萌芽まで学ばないと解析接続の意義はわからんよね
0887132人目の素数さん
垢版 |
2021/04/19(月) 18:23:59.69ID:wJ9Ijnpl
大して意味もないものをみんながありがたがってるからなんとなくありがたがってるとでも思ってるんでしょ?
もちろんその可能性を疑いながら勉強したいならしてもいいし、意味がないもの勉強する気にならないなら勉強しなくてもいい
結局その辺の見極め、心構え全てが学門を学ぶ上での“才能”な訳だし
むしろ人間は心の奥底で“意味ない”と思ってるもの身につくわけないし
結局その辺の“なんか大切そう”と思えるかどうかの“嗅覚”がないなら数学なんて勉強しなけりゃいい
数学わからなくても今の日本で食っていくのに困らないわけだし
0890132人目の素数さん
垢版 |
2021/04/19(月) 18:57:21.50ID:+ikuDbnn
私は役に立たそうな例一つ知ってますけど、あなたは知らないんですね
0892132人目の素数さん
垢版 |
2021/04/19(月) 20:11:57.03ID:ak89FaSV
>>885

もちろんそれはそうなのでしょうが、それって、
f(z)=1/(1-z) where |z|<1
と言ってるに過ぎないですよね?

もとの f(z)=Σz^n からは、たとえば、
|z|>>1 ならば |f(z)|>>1 だろうなあ
なんていうことも妄想できるのですが、
f(z)=1/(1-z) からはそんなことは読み取れませんよね?

>>882

>むしろ一番自然で乱暴さのない定め方
とのことですが、なぜそれが一番自然と言えるのですか?
0893132人目の素数さん
垢版 |
2021/04/19(月) 20:23:20.58ID:5S2gubm7
コルモゴロフ、フォミーン著『函数解析の基礎原書第4版上』

「空間の完備化」についての定理ですが、ややこしいですね。

R を距離空間とする。R^* をその完備化空間とする。

ややこしいのは、構成した R^* が完備であることの証明の部分です。

こういう分かりにくい議論を嫌って、微分積分の本では、デデキントの切断を使った実数論ばかり書かれているんですかね。


p.69

「残るのは、空間 R^* が完備なことの証明である。まず、 R の点からなる基本列 x_1, x_2, …, x_n, … はすべて、 R^* においては、この基本列
で決定される R^* の点 x^* に収束する。このことは R^* の構成からただちに結論される。」

「R の点からなる基本列 x_1, x_2, …, x_n, …」に登場する R は定理の証明中で構成された R^* へのもともとの R の埋め込み R' です。
x_i はもともとの R の基本列が属する類で、その類に属する基本列がすべて R の同一の元に収束するようなものです。
「R^* においては、この基本列で決定される R^* の点 x^*」の一番目の R^* は証明中で構成された R^* で、二番目の R^* は R' の完備空間 R'^* のことです。
「R^* においては、この基本列で決定される R^* の点 x^*」の x^* は R'^* の元です。


このように階層のことなるものを安直に完全に同一視してしまっても問題はないのでしょうか?
0895132人目の素数さん
垢版 |
2021/04/19(月) 22:05:56.64ID:LhSHwvNz
>>892
何が言いたいのか分かりません。
「俺の気にくわない」は数学ではありません。
0897132人目の素数さん
垢版 |
2021/04/19(月) 22:31:48.72ID:ak89FaSV
>>895
なぜそれが一番自然と言えるかという質問も理解できませんか?
0898132人目の素数さん
垢版 |
2021/04/19(月) 23:03:32.36ID:LhSHwvNz
正則関数の正則関数による拡張が自然と思えないのならば自然だとは感じないでしょう。
0900132人目の素数さん
垢版 |
2021/04/20(火) 09:16:54.48ID:Fcf/+9+Z
>>898

ほとんど内容のないお答えですね。
0902132人目の素数さん
垢版 |
2021/04/20(火) 12:47:30.05ID:rjd7+aWR
>>892

もとの f(z)=1/(1-z) からは、たとえば、
f(2)=-1 ということもわかるのですが、
f(z)=Σz^n からはそんなことは読み取れませんよね?

原点を中心としたべき級数展開だけを神聖視する病気なら、それはしょうがない。
例えば1/zと1/(1-z)が平行移動でうつりあわないし、私は近づきたくないが、
既存の複素解析とは考え方が違うのだから、自分で理論構築でもすれば良い。
0903132人目の素数さん
垢版 |
2021/04/20(火) 14:30:11.84ID:um3o3lUE
空集合について質問です。

集合族 F の任意の元 a, b に対し、 a ∪ b ∈ F であるとき、 F はunion-closedであるという。

空集合もunion-closedでしょうか?

S = {F | F は有限集合の有限な集合族である。F はunion-closedである。}

空集合は S の元でしょうか?
0905132人目の素数さん
垢版 |
2021/04/23(金) 10:26:57.24ID:6wWbmJbC
Xを最大元1を持つ順序集合
X0=X\{1}
この時、Xの正則開集合全体の成すブール代数、X0の正則開集合全体の成すブール代数はブール代数として同型である

この証明がわからない
同型写像は何なんだ?
0906132人目の素数さん
垢版 |
2021/04/23(金) 11:55:48.48ID:4w3qqVRK
>>863
そうみたいね
0907132人目の素数さん
垢版 |
2021/04/23(金) 11:57:34.30ID:4w3qqVRK
>>878
必然って?
正則な関数で拡大できたら嬉しいってだけ
0908132人目の素数さん
垢版 |
2021/04/23(金) 12:13:52.15ID:aSwRq5XB
相乗平均の式の質問です

R=5√(1+〇%)×(1+〇%)×(1+〇%)×(1+〇%)×(1+〇%)−1×100=〇%

で合ってますか?
0909132人目の素数さん
垢版 |
2021/04/23(金) 18:04:17.99ID:Y3/1ak7C
>>905
最大元は順序位相の開集合基(開区間)に含まれないから内点ではない
だからXの正則開集合は最大元を含まない
つまりXの正則開集合とX0の正則開集合は同じ
0910132人目の素数さん
垢版 |
2021/04/23(金) 18:21:30.34ID:6wWbmJbC
>>909
いい忘れてたけど、
Xが順序集合の時、Xの開集合系:=({(-∞,x] | x∈X}によって生成される開集合系)として位相空間と考える。
ただし、(-∞,x] := {y∈X|y≦x}である。
0911132人目の素数さん
垢版 |
2021/04/23(金) 18:29:48.17ID:QgHfFtbm
>>907
ハルトークスの定理を知っていたら、必然というのはそんなにおかしな表現でもない
0912132人目の素数さん
垢版 |
2021/04/23(金) 19:13:48.46ID:v4fxJgW2
(-∞,1)は ∪(-∞,x] (xはX-{1}を走る)と書けるよね
0913132人目の素数さん
垢版 |
2021/04/23(金) 19:24:09.57ID:6wWbmJbC
>>912
既に間違ってる。
正則開集合(regular open)全体の成すブール代数RegOp(X)の単位元はX。1∈X
0914132人目の素数さん
垢版 |
2021/04/23(金) 19:33:53.75ID:v4fxJgW2
正則開集合の定義を書いてみてよ
0915132人目の素数さん
垢版 |
2021/04/23(金) 19:46:42.41ID:6wWbmJbC
>>914
A ⊆ Xとする。op,clはそれぞれ開核作用素、閉包作用素
op cl (A) = Aの時、Aは正則開集合
0916132人目の素数さん
垢版 |
2021/04/23(金) 19:48:02.10ID:6wWbmJbC
RegOp(X)において、積はA∩B, 和はop cl (A∪B)、マイナスはop(X\A)とすることでブール代数となる
0917132人目の素数さん
垢版 |
2021/04/25(日) 14:38:11.44ID:rdsF2370
何か見る見る内に数学を勉強・研究する体力が落ちてきたんだが、どうやったら集中力を持続・回復させれる?
気分転換に何か良いのないか?
0919132人目の素数さん
垢版 |
2021/04/27(火) 03:48:11.23ID:3hG/efYU
素数pの円分体Q(ζ_m)での分解についてなんですが
p|mのときってどうなるんでしょうか?
0922132人目の素数さん
垢版 |
2021/04/27(火) 14:44:15.25ID:3hG/efYU
m=m'p^k (m',p)=1として
Q⊂Q(ζ_(p^k))⊂Q(ζ_m)を考えると
たぶんQ(ζ_(p^k))では(p)は完全分岐ですよね?
そこからさらにQ(ζ_m)に上がるとどうなるかよく分からないんです
Gal(Q(ζ_m)/Q(ζ_(p^k)))=Gal(Q(ζ_m')/Q)
なので、そこの分解は通常のようにmod m'で判断できるんでしょうか
0923132人目の素数さん
垢版 |
2021/04/27(火) 15:24:06.91ID:DJx9gYQv
>>922
> Gal(Q(ζ_m)/Q(ζ_(p^k)))=Gal(Q(ζ_m')/Q)

(Q(ζ_m')/Q) はmod pで分岐指数0だから拡大指数e,上にあるイデアルの数をgとしてφ(m') = egでg = [ F_p(ζ_m') : F_p ] = Z/m'Zの乗法群におけるpの類の位数になるのではなかろか
0925132人目の素数さん
垢版 |
2021/04/27(火) 19:32:41.08ID:f9wypzjN
行列の質問です
1×3行列と3×1行列の積は1行1列の行列になると思い、結果を(-3)のような形で回答したところ教授から「-3はスカラーで、行列とスカラーは別物なので()はつけない」というような形で、減点をされました
これはそういうルールなのでしょうか?
0929132人目の素数さん
垢版 |
2021/04/27(火) 22:11:08.72ID:7jrUh7dD
行列とスカラーを厳密に区別するなら、行列の積は行列であるべき
だから、1行1列の行列と見たほうが正しいと思うよ
0931132人目の素数さん
垢版 |
2021/04/28(水) 01:46:40.45ID:Mu+6Sp1L
>>925
スカラーは1行1列の行列だから教授が間違ってる
ただし、1行1列の行列はスカラーだから -3 と書くのが正しい
0932132人目の素数さん
垢版 |
2021/04/28(水) 05:46:51.25ID:aGl8yxI2
んなどっちでもエエやン
3︎⃣でもエエで
0934132人目の素数さん
垢版 |
2021/04/28(水) 17:50:32.71ID:z2Ews3Sn
おまいら
レベルの低い話題は盛り上がるな
0936132人目の素数さん
垢版 |
2021/04/28(水) 19:04:49.82ID:+hT3FC8z
難しい問題には即座に煽りレスがつき何回も聞くとコピペ認定される
簡単な問題には即座に解答がつき解答者は大人ぶる

これが数学板の実力です
専門板なのに異常にレベルが低い
せいぜい数学の少しできる高校生レベル
0937132人目の素数さん
垢版 |
2021/04/28(水) 20:03:38.20ID:CpSmZ5gU
物理板も似たような感じだよ
SNSも5chもワイワイやるのが第一目的だから…
0940132人目の素数さん
垢版 |
2021/04/28(水) 21:11:43.39ID:+hT3FC8z
今日も「解いた側」の圧勝かぁ・・・。
毎日毎日、ラクラク解ける問題ばかりだから常勝なんだよね・・・。
たまには、解けない解けないっと悩んで負けてみたい、それが今の切実な悩み。
0943132人目の素数さん
垢版 |
2021/04/29(木) 02:20:01.62ID:Fzh+VbgJ
広田微分の演算子をD_xとして、
e^(D_x) f(x)・f(x) = f(x+1) f(x-1)
ってどんな場合でも成り立ちますでしょうか?
色々計算してみたのですが、どうしても分からずもしかしたら別の条件式から導くのかもしれません
0944132人目の素数さん
垢版 |
2021/04/29(木) 04:09:04.17ID:W4wiZagW
>>943
閉区間Iで言えれば良い
この区間で一様収束する級数
f(x)=Σak exp(kx)
が取れる( ∵ f(logt)にweirrstrass使う)
この時
exp(Dx)f(x)・f(x)
=exp(Dx) Σ akal exp(kx)・exp(lx)
=Σ(k-l)^m/m! akal exp((k+l)x)
=Σ akal exp((k+l)x + (k-l))
=f(x+1)f(x-1)
0947132人目の素数さん
垢版 |
2021/04/29(木) 16:57:05.84ID:6iarWeMY
>>946
自演の証拠は?何で補償する?お前の全資産?相手の全負債を受け持ちつつ?嘘じゃないなら担保できるだろ?
0949132人目の素数さん
垢版 |
2021/04/29(木) 18:38:16.35ID:Fzh+VbgJ
>>944
ありがとうございました!
0951132人目の素数さん
垢版 |
2021/04/29(木) 22:44:25.81ID:p7y4MBh1
f : X→Y
f(x) = 3x
のような写像fを考えた場合、

なぜ、
f({2})={6}
となり、
f({2})={{6}}
とならないのですか?

また、
f({5,7})={15,21}
となり、
f({5,7})={{15},{21}}
とならないのですか?
0952132人目の素数さん
垢版 |
2021/04/29(木) 23:39:58.79ID:hiF+fJig
>>951
f({2})={{6}}にも一理ある、と思わせるような理屈を知りたい
0953951
垢版 |
2021/04/29(木) 23:57:25.97ID:p7y4MBh1
>>952
f(x) = {3x}とすれば、

f(A) = {f(x)| x ∈ A}
A={5,7}

f({5,7}) = {f(x)| x ∈ {5,7}}={f(2),f(3)}
f(2) = {15}
f(3) = {21}
0954132人目の素数さん
垢版 |
2021/04/30(金) 00:00:21.02ID:Yx2j79tN
↑ 間違いありました。すみません。

>>952
f(x) = {3x}とすれば、

f(A) = {f(x)| x ∈ A}
A={5,7}

f({5,7}) = {f(x)| x ∈ {5,7}}={f(5),f(7)}
f(5) = {15}
f(7) = {21}

ご検討お願いいたします。
0956132人目の素数さん
垢版 |
2021/04/30(金) 06:08:21.13ID:dzGyJyyg
>>954
そう定義すればそうなる、としかいいようがない
0957132人目の素数さん
垢版 |
2021/04/30(金) 06:31:09.06ID:wQw+PGCf
aと{a}を区別して考えるのに、「f(x)={3x}とすれば」とはどういうことなの
それ違う写像じゃん>>951とは関係なくなってるじゃん
0958132人目の素数さん
垢版 |
2021/04/30(金) 07:18:48.32ID:dzGyJyyg
集合値関数はちゃんとした数学的対象
0960951
垢版 |
2021/04/30(金) 08:42:25.50ID:YUqlDPPN
>>956, 957

コメントありがとうございます。

岩波書店の「集合・位相入門」p.27に
「f(a)={b}と書くかわりに、単に、f(a)=bと書く。」
とあり、混乱しております。

f : X→Y
f(x) = 3x
の場合、

3x∈X
f(x)∈Y

と考えれば、一般的な写像の概念に、沿ったものとなりますか?
0961132人目の素数さん
垢版 |
2021/04/30(金) 10:05:50.99ID:dzGyJyyg
>>960




岩波書店の「集合・位相入門」p.27に
「f(a)={b}と書くかわりに、単に、f(a)=bと書く。」

もうちょっと前の方から引用してくれないと意味がわからない
0963951
垢版 |
2021/04/30(金) 10:23:48.32ID:YUqlDPPN
>>961
補足致します。

fをAからBへの写像とすれば,Aのどの元aに対しても,そのfによる像f(a)は
Bの1つの元bから成る集合{b}となっているわけであるが,
この場合は,通常,({b}のかわりに)bをfによるaの像といい,
また,f(a)={b}と書くかわりに,単に,f(a)=bと書く.
0964132人目の素数さん
垢版 |
2021/04/30(金) 10:46:57.60ID:np2wkfiR
相変わらずどうでもいいとこで詰まってる
なんでそんなとこで詰まれるんだよ
足踏みばっか
0965132人目の素数さん
垢版 |
2021/04/30(金) 10:47:49.06ID:dzGyJyyg
>>963
f({a})={b}なら正しいが、その記述のままなら間違っている
0967132人目の素数さん
垢版 |
2021/04/30(金) 12:32:56.60ID:np2wkfiR
本そのもの読んでないんでわからないがおそらくその本では
f(x) = { y ∈ Y | <x,y>∈f }
とか定義して任意のx∈Xに対しf(x)がsingleton {y} になるときfを関数と呼び、特例としてf(x)=yと書く
とかいう構成してるんだろ
まぁそうだとしたらあまりいい構成と思えないが、どのみちこんなとこ詰まるようなとこじゃない
こんなところで足踏みばっかりして馬鹿なんじゃないかと
0968132人目の素数さん
垢版 |
2021/04/30(金) 13:25:19.87ID:67YSvwa5
>>967
分からない所は付箋を付けておいて一旦スルーして勉強を進めるのも大事だけど、
徹底的に追求して自分の中で納得できるレベルに辿り着くことも大事なんだよな
0969951
垢版 |
2021/04/30(金) 14:02:29.21ID:YUqlDPPN
f: X → Y
f(x) = 2x
x∈X, y∈Y

f({3,5,7}) = {y|y=f(x),∃x∈{3,5,7}} = {6,10,14}

の∃とはどういう意味ですか?
∃がなかったら、答えが変わりますか?
0970951
垢版 |
2021/04/30(金) 14:25:40.63ID:YUqlDPPN
すみません。↑を訂正させて下さい。

f: X → Y
f(x) = 2x
x∈X, y∈Y

f({3,5,7}) = {y|∃x∈{3,5,7}, y=f(x)} = {6,10,14}

という表現は、

「それぞれのx∈{3,5,7}について、y=f(x)を満たすものの集まり」

という理解で大丈夫ですか?
0971132人目の素数さん
垢版 |
2021/04/30(金) 14:40:18.52ID:WcG2Oz7V
大丈夫でない
「それぞれのx∈{3,5,7}について、y=f(x)を満たす y の集まり」
0972132人目の素数さん
垢版 |
2021/04/30(金) 15:02:38.47ID:wuiRaabM
>>968
もちろんわからないところを追求するのは大切だけどコイツのは単なるイチャモンの域をでないような重箱のすみ突っつくだけの行為を延々と繰り返すだけなんだよ
挙句最後は著者が悪い、教科書が悪いに行き着く
そういうところどれくらいこだわるかは定量的な問題
最大の効率、あるいはそれに準ずるところを見定めないといけないのにコイツはくっだらないところをいつまでもいつまでもいつまでもこだわっておんなじところずーっと足踏み
実際学力全然上がってない
それで終わるだけなら自己責任でいいんだけど、それで失敗したことを他人のせいにして公共の掲示板でグチグチグチグチグチグチグチグチ文句だけ垂れ流してくるんだよ
0975951
垢版 |
2021/04/30(金) 15:36:03.56ID:YUqlDPPN
>>971
有難うございました。
0977132人目の素数さん
垢版 |
2021/04/30(金) 20:27:07.25ID:WQqCcSZW
p[n]をn番目の素数として、無限級数
 f(x)=Σx^n/p[n]
について何か調べられていることはありますか?
0978132人目の素数さん
垢版 |
2021/04/30(金) 20:32:22.32ID:3JJvFfx9
馬鹿アスぺ二号は ID:AykD014e だよ
「松坂君」の呼び名は松坂先生に失礼なのでやめてね
0979132人目の素数さん
垢版 |
2021/04/30(金) 23:19:32.11ID:Mv71lLwz
>>947
0980132人目の素数さん
垢版 |
2021/04/30(金) 23:21:39.23ID:Mv71lLwz
>>964
ホンソレ
重要性を見積もれないんだろうか
0982132人目の素数さん
垢版 |
2021/05/01(土) 01:36:44.03ID:A3c/N6CR
体Kの元からなる無限数列(a_[1],•••,a_[n],•••)
の全体のなす集合はKベクトル空間である
基底がわかりません。教えてください
0983132人目の素数さん
垢版 |
2021/05/01(土) 02:46:33.84ID:WyJRELtY
>>982
具体的に書けるわけない
0984132人目の素数さん
垢版 |
2021/05/01(土) 02:57:15.05ID:A3c/N6CR
集合で表せないということですか?
0985132人目の素数さん
垢版 |
2021/05/01(土) 03:14:56.00ID:WyJRELtY
集合で表すとは?
0986132人目の素数さん
垢版 |
2021/05/01(土) 03:17:02.28ID:WyJRELtY
君はRがQベクトル空間であることを示せると思うけど
基底は何だと思う?
0987132人目の素数さん
垢版 |
2021/05/01(土) 03:42:29.82ID:A3c/N6CR
確かにそうでした。ありがとうございます。
0988132人目の素数さん
垢版 |
2021/05/01(土) 11:57:43.38ID:khuIlfMx
初歩的な質問だったらごめんなさい
これの一番上なんですが、なんでNの補集合にNが含まれているのでしょうか? 意味がわかりません...
https://i.imgur.com/GRmv7Bw.png
0989132人目の素数さん
垢版 |
2021/05/01(土) 12:07:51.33ID:ViE9i5Yi
補集合ではなく閉包です。
0990132人目の素数さん
垢版 |
2021/05/01(土) 12:10:41.47ID:u8ptD1Mp
>>986
てかその酷い文章の出典は何?
とてもまともに数学わかる人間の書いた文章と思えん
0991132人目の素数さん
垢版 |
2021/05/01(土) 12:14:05.77ID:khuIlfMx
>>989
理解しました、ありがとうございます
0992132人目の素数さん
垢版 |
2021/05/01(土) 12:20:21.48ID:mM+NbUKn
そんなに酷いか?
0993132人目の素数さん
垢版 |
2021/05/01(土) 12:33:01.44ID:8lxYHkz2
>>988
それはNの閉包

補集合をバー表記する本で学んだから誤解してるだけだけど、位相空間論では閉包をバーで表記するのは標準的
0994132人目の素数さん
垢版 |
2021/05/01(土) 12:42:10.44ID:+Rae8XGu
>>990
酷くないじゃん
実数は有理数係数ベクトル空間で、
その基底の存在証明は非構成的である。
そういうことを言っている?
0997132人目の素数さん
垢版 |
2021/05/01(土) 13:21:55.21ID:06BSD7kJ
ホッケは包茎
ウグイスは包〜包茎きょ
仙台包茎専門学校
なるほど、ほうけ〜
0999132人目の素数さん
垢版 |
2021/05/01(土) 13:27:49.21ID:2D+Ak2Ng
違う
まぁわかるやつは一発で素人の文章とわかる
わからんやつはわからんので説明はせん
10011001
垢版 |
Over 1000Thread
このスレッドは1000を超えました。
新しいスレッドを立ててください。
life time: 123日 19時間 22分 54秒
10021002
垢版 |
Over 1000Thread
5ちゃんねるの運営はプレミアム会員の皆さまに支えられています。
運営にご協力お願いいたします。


───────────────────
《プレミアム会員の主な特典》
★ 5ちゃんねる専用ブラウザからの広告除去
★ 5ちゃんねるの過去ログを取得
★ 書き込み規制の緩和
───────────────────

会員登録には個人情報は一切必要ありません。
月300円から匿名でご購入いただけます。

▼ プレミアム会員登録はこちら ▼
https://premium.5ch.net/

▼ 浪人ログインはこちら ▼
https://login.5ch.net/login.php
レス数が1000を超えています。これ以上書き込みはできません。

ニューススポーツなんでも実況